Segment 1 Of 3     Next Hearing Segment(2)

SPEAKERS       CONTENTS       INSERTS    Tables

 Page 1       TOP OF DOC    Segment 1 Of 3  
MILITARY CONSTRUCTION APPROPRIATIONS FOR 1999

Thursday, February 12, 1998.

DEPARTMENT OF DEFENSE

WITNESSES

WILLIAM J. LYNN, UNDER SECRETARY OF DEFENSE (COMPTROLLER), ACCOMPANIED BY HENRY SODANO, DIRECTOR OF CONSTRUCTION

Statement of the Chairman

    Mr. PACKARD. We are a couple of minutes early but let's go ahead and get started. The main man is here. I am grateful to have Mr. Hefner with us early because I wanted to make a comment or two.

    In all my career here in the Congress, which is now in its 16th year, much longer than I anticipated, I have never worked with a person that I have been more grateful to have that opportunity than Bill Hefner. He has been truly a marvelous person to work with. We have been a good team on this subcommittee. His experience as chairman of the subcommittee for some time certainly has helped me and helped this subcommittee and I am extremely grateful.

    As most of you know, he has announced that he would be retiring at the end of this term and the loss to the country, the loss to the Congress and the loss to this subcommittee and to me personally is going to be huge. Bill, it has been a pleasure, a real pleasure working with you.
 Page 2       PREV PAGE       TOP OF DOC    Segment 1 Of 3  

    Mr. HEFNER. I appreciate it.

    Mr. PACKARD. We have done some things together outside of congressional work that we have enjoyed, too, and that has always been a pleasure. He and I have played some golf together. So Bill has been a real good friend. Sometimes that is getting to be more and more of a rare thing, to have members from the two sides of the aisle become dear friends but Bill and I have. I think it has been helpful to this subcommittee.

    It is a pleasure now to call this subcommittee to order. We are very pleased to have the Honorable William Lynn, the Under Secretary of Defense (Comptroller) as our first witness.

    This is the beginning of our hearings. We will conclude them in March, I believe, except for the break that we will be having in mid-February. We will be holding hearings regularly.

    We have a new member of this subcommittee, Mr. Bud Cramer from Alabama. He is also a good friend and will make, I think, a significant contribution to this subcommittee. He has recently been appointed to the Appropriations Committee and has been given this as one of his subcommittees. We will look forward to working with Mr. Cramer.

    Let me begin with some housekeeping comments and hopefully each member will get this message, even though they are not here now.

 Page 3       PREV PAGE       TOP OF DOC    Segment 1 Of 3  
    We will try to start on time. We have done that in the past. We will try to do that in the future.

    We will recognize members for questioning after the statements are made, but we will try to hold to the five-minute rule. If it is necessary, we will go into a second or even a third round of questioning. We will not shut the members down from questions. They will have all the time they need, but we would like to hold it in some orderly way. So we will try to adhere to the five-minute rule, the ranking member, of course, being excluded from that. I would never cut him off at any time.

    We also will welcome any member of the subcommittee to submit questions for the record and ask the witnesses, of course, to respond to that in writing and send it to the subcommittee. They are welcome to do that.

    With those few comments, let me then comment now on the substance of this hearing.

    We are a subcommittee that has been pinched for monies for years and the pinch is getting more and more severe. The President's budget this year is, I think—I will say it very bluntly—abominable. It really is. It shortchanges us far more than we ever expected.

    The President has made a strong commitment that he would certainly take care of our men and women in the services, their quality of life issues, and so forth. But we are being asked in this budget submission from the President, to absorb a 15 percent cut from last year's appropriated levels of $9.2 billion down to $7.8 billion. That is a cut that I just can't see how we can accept or live with. And frankly, I think the subcommittee will find ways to strengthen that budget because that is deeper than what we can absorb and still do what we think we are charged to do, particularly on top of two previous years that I am familiar. I think it goes back maybe even further than that, but certainly the last two previous years we have absorbed a $2 billion cut. And out of an $11 billion budget, you are talking about a 15, 16, 17, and 18 percent cut over those two years.
 Page 4       PREV PAGE       TOP OF DOC    Segment 1 Of 3  

    So if the President's budget prevails, we will have absorbed close to a 30 percent cut in the last three years, and that simply is not acceptable for any subcommittee, much less one of our Defense Subcommittees.

    So this is the lowest budget request since 1982 and I don't see a way that we can absorb it and still do what we have to do. This cuts into our commitment to take care of the quality of life and work issues for our men and women in the services; namely family housing, single member housing, barracks, day care centers, hospitals, dental centers, and all of the other quality of life areas.

    But there are also many quality of work areas at military bases that we have visited and we have seen the deplorable conditions that many of our men and women have to work in. Those accumulate to where, as Bill Hefner has been pointing out very strongly for several years now, that affects retention. We are losing good men and women in the services because of the quality of work or the quality of their living quarters and their other benefits for their families.

    We have tried to strengthen that area, but this budget will not only limit but also actually decrease our ability to address these life and quality of work issues. And I believe we are reaching the point, in this budget, where we are dealing with readiness. We don't believe that we can provide the physical facilities that will allow our men and women in the services to be ready for any military action and certainly right now we are facing the very strong possibility of military action that will test our ability for our men and women to fight and to be ready for such military action.
 Page 5       PREV PAGE       TOP OF DOC    Segment 1 Of 3  

    And when we reach the point where not only quality of life is being deteriorated but also our readiness is being threatened, I think we have reached a point where we are below any budget level that is acceptable.

    Those are strong comments and I am really concerned. We have before us the man that can address that concern, the Comptroller of our Defense Department, the Under Secretary, the Honorable William Lynn.

    Before I turn it over to you, Mr. Lynn, let me again add what I said earlier, how grateful we are to have Bud Cramer join us on this subcommittee. He is going to make a wonderful addition. He has good experience on military issues. He has a significant amount of military concerns and Defense concerns in his district and certainly in his state and we are extremely grateful to have you with us, Mr. Cramer, and hope that you will make a contribution to this committee. We fully expect that.

    Mr. CRAMER. Thank you, Mr. Chairman.

    Mr. PACKARD. Again I add some preliminary comments. We will hold to the five-minute rule when we reach the question and answer period, but we will have a second and a third round of questions, if necessary. We will not cut anybody off from a chance to ask all the questions that they need to ask. But we will try to have some order to it.

    We would invite you to submit questions for the record if you choose to do so. We will be as flexible, as we can in the way we operate.
 Page 6       PREV PAGE       TOP OF DOC    Segment 1 Of 3  

    Again, thank you for being here, Mr. Lynn. We will now turn to you.

    Oh, I apologize. I should have a statement from my colleague.

    Mr. HEFNER. I will be very brief. I just left a hearing upstairs with the Defense Subcommittee. General Rodeman made the statement that our most important and most critical weapon is the soldier or the marine or the sailor. That is what we have said in this subcommittee for many, many years.

    We are asking people to go out and operate the most sophisticated weapons that man could ever imagine and then we are asking them to go back to their families and themselves and live in facilities that were not very adequate during the Korean and Vietnam War. Some of these facilities are World War II vintage. We are asking these men to go back and live in those facilities and go out the next day and be gung-ho about their jobs and operate the most sophisticated weapons in the world.

    Now, this just flies in the face of what is reasonable. Over the years, we have found here and all the administrations that they're all guilty of it. When they put together a budget, it seems that the military construction budget is not a high priority because it is not real sexy to go to North Carolina or Alabama or Mississippi and cut the ribbon for some new barracks. Nobody comes except the people that built the barracks and some other folks. If you unveil a B–2 or a B–1 or a new tank or what have you, people come and they cheer and they carry on.

 Page 7       PREV PAGE       TOP OF DOC    Segment 1 Of 3  
    But this is, in my view, the most nonpartisan committee. It is the best committee in this House. It has the best staff. I believe we do the best work with what dollars we have to do the job for the men and women in the service.

    This budget is not adequate. It hasn't been adequate for the past several years. During that time we have had a total ''pause''. We didn't do anything. I believe in the budget cycle last year, we didn't do anything for the Marines to speak of—did we?

    This is an inadequate budget and we are called on year after year because they know that this committee is made up of people who have bases. They say, ''Well, the guys over there can do the weapons stuff and they'll take care of the military guys'' and then they give us a pittance.

    So it kind of gets frustrating but I think we have done a remarkable job under different leadership and under the excellent leadership of Mr. Packard to use the scarce dollars that we have to do the best that we can, especially now that we are in a situation where retention could get to be a problem for us in the years to come, and especially when people are seeing that commitments are not kept or being honored as far as health care that men have signed up for.

    So it is a real tough chore and I commend Mr. Packard, who will be fighting this battle hopefully for years to come. We hope that you will be able to take our message back and be our voice when they make these decisions. With that, I want to thank you for being here.

 Page 8       PREV PAGE       TOP OF DOC    Segment 1 Of 3  
    I also want to thank you for those kind words, Mr. Chairman. It has been a joy to serve on this committee. I will be able to look back with some pride on things we have been able to do. I wish all the committee Godspeed and good luck in doing what is right for our men and women in the services. Thank you, Mr. Chairman.

    Mr. PACKARD. Thank you very much.

    We don't normally have opening statements but if any member of the subcommittee would like to make an opening statement, we certainly would make room for you. We like to hold short hearings and I think that is better than opening statements usually, so we will turn now to Mr. Lynn.

STATEMENT OF THE HONORABLE WILLIAM J. LYNN

    Mr. LYNN. Thank you very much, Mr. Chairman. I appreciate the opportunity to appear before you in your first hearing of the year. This is indeed my first hearing, as well, as the Comptroller. I have testified with the Secretary but this is the first time that I have been the principal witness and I appreciate the opportunity.

    In the interest of preserving your time for the questions, which I think you would like to concentrate on, I ask that the full statement that I have submitted be included in the record. What I would like to do is just highlight four points out of that statement and then turn it over to you for questions.

    Mr. PACKARD. That will be fine, without objection.
 Page 9       PREV PAGE       TOP OF DOC    Segment 1 Of 3  

    Mr. LYNN. The first point I wanted to make is one that comes from the hearing you had last year with my predecessor, John Hamre. Mr. Chairman, as I understand it from reviewing the transcript, an important point you made there was the question of how the Department does its planning. We weren't focussed at that point on the budget year but on how we do long-term planning for military construction and family housing. I think you cited deficiencies there. I think that was fair and I think Dr. Hamre took that to heart. We have tried to do some things to address those deficiencies. Let me just name a few.

    We have started to set planning goals within these accounts. Dr. Hamre set a goal to eliminate all inadequate housing in the Department of Defense by the year 2010. With 345,000 family housing units—that is an ambitious goal. We hope to use some techniques out of the private sector, leverage private funding, and some other things to accomplish that goal because we are not going to be able to do it purely within our budget. We think we have a goal and we are developing the plans to accomplish that goal.

    Similarly, there is a goal to eliminate permanent party gang latrines and open bay berthing by 2008. There I think we are even a little further along. All of the services have plans that will do that—the Air Force by 1999 and the other services will hit their goal by 2008.

    Finally, as a consequence of the Defense Reform Initiative, the Secretary's initiatives to try and take infrastructure overhead out of the Department, be able to put funding against the highest priorities, and move money from tail to tooth. We are also looking towards privatization of utilities. We have set a goal to try to privatize all the utilities, where it is appropriate, by the year 2000. You obviously make national security exceptions or other exceptions, but the general goal in the system would be to privatize all those except where there were unique security needs or privatization was uneconomical.
 Page 10       PREV PAGE       TOP OF DOC    Segment 1 Of 3  

    Those, I think, are the things we are trying to address in response to your request and your direction to do more planning in this area. I think that is a foundation on which we can build, but I wouldn't say that we are done in that area. I think there are going to be other things that we are going to try and address.

    The second issue that came up in last year's hearing, and has certainly come up throughout the year, is the question of the integration of the guard and reserve. A number of issues came up as we did the Quadrennial Defense Review about the adequacy of how we review the guard and reserve funding requests, funding needs in the budget, and I think we have recognized that there have been problems in this area. We took steps this summer and fall to try to address those issues.

    As a consequence, you will find that the National Guard and reserve component leadership are now members of the Defense Resource Board where all the major long-term program decisions are made. They are in the major budget issue meetings, the MBIs, which is where the final decisions are made on the budget year, in this case the '99 budget. The Secretary, myself, and John Hamre have been meeting directly with the guard and reserve leadership to try to make sure we understand those priorities and we have, in general, tried to take steps to have a total force perspective on the budget priorities.

    As a consequence of some of that, you will see in some of the accounts the guard and reserve budgets have gone up over what we expected to request in '99, in some cases what the Military Departments had requested in '99. I would not say we have addressed all the issues that the guard and reserve leadership would raise. I can't frankly say that about any community in a constrained budget environment, but I think as a consequence of the improvements in the process, we have done better. We have improved things from their perspective and frankly, from the perspective of the whole Department by addressing their priorities.
 Page 11       PREV PAGE       TOP OF DOC    Segment 1 Of 3  

    Third, let me comment on the issue that you focussed on, Mr. Chairman. I understand your disappointment that this budget frankly does not match the increases that the committee was able to allocate and that Congress was able to allocate through this committee to the military construction and family housing areas. I understand that you do think more is needed here and I think we agree. We do want to do more here.

    We are working through, at this point, the first year of the Quadrennial Defense Review. In that first year we developed some savings that frankly went other places. We added $1 billion to $1.5 billion to the readiness accounts. In the hearing Mr. Hefner said he just came from, I don't know whether General Rodeman mentioned, but there was a $500 million increase in the defense health program account to meet must-pay bills. The account was underfunded and it was something that we had to do and something we really had to do in '99. There was no discretion.

    You are all familiar with the Secretary's commitment to getting on a modernization path up to $60 billion and we took the first step in that with the $3.5 billion increase in the procurement accounts this year.

    All of those things put pressure on the budget. It meant that this year we were not able to match the increases that you set. I understand, as I said, your disappointment in that. I think what I can offer you is that this is just the first year of the QDR.

    There will be additional savings. We are, over the course of the next two to three or four years, going to reduce the end strength by 60,000 people, with congressional approval. We are going to reduce reserve end strength by a commensurate amount. We are going to reduce civilian end strength, frankly, by even more—80,000 right now is what is projected.
 Page 12       PREV PAGE       TOP OF DOC    Segment 1 Of 3  

    All of those reductions will yield savings. Those savings, as I say, in the initial steps have gone into readiness, health, and modernization accounts. We have programmed in the out-years increases for family housing and military construction.

    Now, I understand programming them in the out-years is not the same as carrying through in the budget year and we are going to need to follow through and protect all or most of those increases in order to satisfy the committee and the Congress. And I would say to you now that I would hope to be coming back to you next year with something that would be improved.

    What has changed, as I said, is the QDR. This is the first year. We hope to do better into the second and the third year.

    The fourth and last point I wanted to raise in the testimony is one that I don't think is well received in Congress but the Secretary believes, as well as I and the Administration, is needed, is the question of additional base closure rounds. We think that the case is clear. We have not reduced the base structure commensurate with force structure and budget reductions.

    We need, we think, two additional rounds to get those in line and into balance. We have proposed that be done with two additional rounds. We have slightly shifted the proposal from the one that you saw last year. In order to address some of the concerns that were raised, we are proposing that it be done in the first year of the next two Administrations, so there would be one in 2001 and one in 2005.

 Page 13       PREV PAGE       TOP OF DOC    Segment 1 Of 3  
    That, of course, would mean that you would have different Congresses and different Presidents that would propose those rounds. We think that by doing it that far in advance and by putting in place now, a base closure round that far in advance, you will insulate it somewhat from the politics that I think are a concern to some people.

    The bottomline is, as the Secretary has said, that we need this in order to provide the funds for the modernization, the readiness, and the military construction and family housing needs. We are going to have to bring the base structure into line with the force structure.

    With those four points, and other points in the testimony, I am happy to explore whatever you would like in questioning. I conclude my statement with that.

    [Prepared Statement of William J. Lynn follows:]
    "The Official Committee record contains additional material here."

    Mr. PACKARD. Thank you, Mr. Lynn. We appreciate your statement. Very well done. Just a comment that ties in with what I mentioned earlier and what you commented on in regard to a reduction in this year's budget.

OVERALL FUNDING

    When there is a decrease in the overall DOD budget, then it is our subcommittee that generally picks up most of that decrease. In the past, about 75 percent of all decreases that the President has submitted in terms of the Defense budget, the MILCON Subcommittee has had to pick up about 75 percent of those decreases.
 Page 14       PREV PAGE       TOP OF DOC    Segment 1 Of 3  

    When the DOD budget is increased, as it has this year, then MILCON is usually looked upon as the offset because we are required to offset any increases. And so our subcommittee gets the short end of whichever way it goes. Whether we have a decrease in our overall DOD budget or an increase in our DOD budget, we seem to be the ones that have to pay for the cost of those increases or decreases and it always hurts us.

    I am not going to ask questions first. I am going to let the members. And what we will do, as we have done in the past years, is, we will recognize members in the order in which they arrived, of course with the exception of the ranking member. And the full committee chairman and ranking member, if they attend our meetings, will be recognized first, of course.

    Mr. Hefner, we would recognize you now for any questions.

HOUSING PRIVATIZATION

    Mr. HEFNER. Well, I have already made my statement.

    Are we moving ahead with the Secretary's recommendation that we—I don't know what the exact name would be, like the housing authority that would work with the military bases to have people in the private sector go in and construct or keep up. Are we still moving in that direction with an experimental program?

    Mr. LYNN. Yes, Mr. Hefner, we are. We are actually trying to accelerate the move. We already have in progress two projects with 2,000 units in this family housing privatization initiative.
 Page 15       PREV PAGE       TOP OF DOC    Segment 1 Of 3  

    Mr. HEFNER. And where is that?

    Mr. LYNN. Those are in Corpus Christi, Texas and Everett, Washington. We have two more. One at Fort Carson, Colorado has just been notified to the committee. That alone will be nearly 3,000 units. And we have another six or seven projects in the proposal stage which would have an additional 15,000 units that, as we develop them, we will be notifying the committee.

    So you will see a geometric increase, because we think there is enormous promise in the leverage provided by this initiative. We can leverage the dollars we have in order to try and meet that goal of eliminating all inadequate housing by 2010.

    Frankly, whatever the budget would be, if we don't go outside our normal means, we would not be able to meet that initiative. We need the three- to five-, even ten-to-one leverage that we get out of the privatization to turn those 345,000 units over that fast.

    Mr. HEFNER. Thank you, Mr. Chairman.

    Mr. PACKARD. If the gentleman would yield on that very point, is our effort to move into privatization delaying the execution of projects?

    Mr. LYNN. I have to say honestly, Mr. Chairman, in some cases, yes. Not long, I think weeks and months, but particularly as we start this up, we have to review which projects to privatize. We have to be careful. We have to pick the right projects. We have to make sure that we have the right relationship with the private sector to make this work. And in some cases it has delayed a few months. While we screen for this privatization it has delayed some projects by a few months.
 Page 16       PREV PAGE       TOP OF DOC    Segment 1 Of 3  

    As you can see, we are moving faster on this. Part of that reflects that the system is starting to come in place and I think you will see less and less delay. But I should say we think the delay is worth it. When we can get that kind of three-to-one, five-to-one, seven-to-one leverage, additional housing units for the troops and their families, it justifies us taking some additional time to get that leverage. We, however, hope to reduce that time to a minimum and start to do this more in the budget bill as we move forward, and make this part of the regular order.

    Mr. PACKARD. Does that create any problem in terms of being able to use your appropriated funds in a timely manner?

    Mr. LYNN. I am not aware of any problem that has created. I will look to Henry to see. I should introduce Henry Sodano, who is the real expert in terms of our military construction budget.

    Mr. SODANO. The appropriations are available for five years. Authorization is good for three years. By no stretch would reviewing the project to see whether it was an acceptable candidate for privatization, would affect the obligations.

    Mr. PACKARD. Mr. Parker.

BASE-REALIGNMENT AND CLOSURE

    Mr. PARKER. Thank you, Mr. Chairman.
 Page 17       PREV PAGE       TOP OF DOC    Segment 1 Of 3  

    Mr. Lynn, I don't want to hurt your feelings or anything and I appreciate the fact that you have guard and reserve, in the meetings. How many decades did it take to get them in the meetings?

    Mr. LYNN. Too many.

    Mr. PARKER. You didn't give them any money but they are in the room, so I will accept that.

    I am very much concerned with this whole concept of what we are doing. You are talking about having a BRAC-3 and a BRAC-4?

    Mr. LYNN. We have already had four rounds so it would actually be 5 and 6.

    Mr. PARKER. 5 and 6. Maybe I am missing something. We have had all these recommendations to close and realign these bases but I don't see us closing and realigning bases. And I know that is disingenuous, coming from Mississippi, when I have a majority leader on the Senate side who is saying we don't need another BRAC. Well, I am of the opinion that we don't quite need it if we do the first, second, third and fourth correctly.

    This whole thing, it is kind of like—do you remember how many times in the past we have spent lost food stamps? You know that issue? Well, what we would do is we would say we have $20 billion of lost food stamps. Now, that was some figure out there and we would spend it on something.
 Page 18       PREV PAGE       TOP OF DOC    Segment 1 Of 3  

    Somewhere along the way we have this base realignment and closure as being some kind of thing out there that we can utilize, but we never follow through with what we're doing. We have not done BRAC correctly yet, and I know it is because of political pressures. I know that in Mississippi it is politically unpopular to say we need to realign some of the bases in Mississippi because everybody from Mississippi that represents the state is going to say we don't need any changes; in fact, we need to expand the bases we have there. That is true in every state that you deal with.

    I am very much concerned about what we are doing in this country. It is not very logical to me how we are approaching it. And I may not be explaining my frustration too well to you but when I hear you talk about this modernization and new weapon systems and all and spending all this money, it is kind of like putting a red light on a dirt road. You know, you have it on this long, straight dirt road and there is not an intersection there. This red light is just sitting there, a traffic light. Well, people may come to look at it but it doesn't serve any useful purpose.

    And I see where we are not serving a very useful purpose in going in these directions and not funding what is really necessary.

    And on the military construction side, if you believe that with the guard and reserve, you get the biggest bang for the buck there, we are not doing what is necessary from that standpoint.

    I read this thing and I don't think we are accomplishing a whole lot with this, in what the Administration's proposal is. I have real problems. And I don't know what the answer is. If I could write it myself, I know exactly what I would do. But do you feel—and I know you are here to defend the President's budget but do you feel real comfortable in the direction we are going?
 Page 19       PREV PAGE       TOP OF DOC    Segment 1 Of 3  

    Mr. LYNN. Yes, sir, I do. Let me take your points in order.

    The Guard and Reserve point, I guess I disagree with your characterization that we added no money. Actually we did increase the optempo account by $100 million and the training account by $50 million and the investment accounts by over $300 million.

    Now, saying that, does that mean that we have addressed every priority that either the National Guard leadership inside the building or the various associations and your Adjutant General would identify? No, we have not and I wouldn't claim that.

    I do think that the process has improved. The dialogue has improved the trust and respect, and that is frankly where we have to start. We need everybody inside working together and we haven't always had that in the past. I think we have moved in that direction.

    It has also shifted some resources towards the guard and reserve priorities, not as much as they would like to see, but it is an improvement from where we were. I would amend your statement there.

    The second point you made was that we are not closing the bases. I am not quite sure what data you are using. I am happy to give you the overall data, to provide that for the record as to where we stand in each of the four closure rounds.

    [The information follows:]
    Offset folios 32 insert here
 Page 20       PREV PAGE       TOP OF DOC    Segment 1 Of 3  

    I can tell you from personal experience in the Department, the Secretary had a base closed in Maine, Loring Air Force Base, and that is now closed.

    I worked on the Senate side for Senator Kennedy from Massachusetts and Fort Devens, a major base in Massachusetts, was identified as a closure candidate in the second round. It was approved by the commission and that base is indeed closed.

    Mr. PARKER. Well, out of 152 major installations, how many of them have actually been closed?

    Mr. LYNN. I don't have that number in my head. We can provide that for the record. Do you have it, Henry?

    Mr. SODANO. Ninety-seven. Ninety-seven of the 152.

    Mr. PARKER. Have been totally closed?

    Mr. LYNN. I don't know how to qualify the word ''totally'' so I would like to take that for the record and we will come back to you with a description of what we mean by the 97.

    [The information follows:]

    When we say a base closes, we are referring to when the flag is lowered, active operations cease and caretaker operations start. There are some instances, however, where the flag has been lowered and active operations have ceased at closed sites that a reserve enclave and/or a Defense Financial Accounting Service (DFAS) activity have been allowed to remain. As of September 30, 1997, of the 77 bases closed, there were 22 sites where either a reserve enclave or DFAS activity remained.
 Page 21       PREV PAGE       TOP OF DOC    Segment 1 Of 3  

    Mr. PARKER. The reason I bring that up is because every time we turn around instead of closing it totally, what we do as a Congress is we turn around and allow them to change the mission or keep something at some of these places.

    Mr. LYNN. There are procedures in the law. You go through the McKinney Act. You go through other government departments that have first claim. Those things have happened. I do not think that they have undercut the basic thrust of the base closure effort and I would cite that we think we will get on the order of $5.6 billion in annual savings from the four rounds of closure we have had.

    We have been asked by the Congress to verify that number. That is a fair question. As an early indication in response to that, we asked the Inspector General of the Department, an independent auditor within the Department, to go and look. They have completed their review of the '93 round and they have concluded that actually we underestimated the savings and we overestimated the costs of implementation.

    So they would suggest that there are actually more savings out there and there was a little bit less investment up front.

    So I think this suggests that the savings are there from the actions that have been taken. As I said, our view is, when you ask the final point, am I comfortable that are we going in the right overall direction my answer is yes.

    Before this job, I had the long-term planning job. When you look at statistics where we have a budget that is down almost 40 percent, a force structure that is down almost 40 percent and a base structure that is down somewhere between 21 and 28 percent (depending on how you count it, whether you count foreign bases and so on) that gap is too large. You could expect some gap and those statistics are not absolutely precise and I wouldn't want to overuse them that way, but we can't only have a one-quarter reduction in the base structure relative to 40 percent force and budget reductions. We will be spending too much money on unnecessary bases if we do that.
 Page 22       PREV PAGE       TOP OF DOC    Segment 1 Of 3  

    Mr. PARKER. Thank you.

    Mr. PACKARD. Thank you, Mr. Parker.

    Mr. Cramer.

PRIVATIZATION

    Mr. CRAMER. Thank you, Mr. Chairman. I will attempt to be brief but I am a new member of this committee and am very interested in the issues presented to this committee.

    You are talking about a budget that commits funds for family housing, a budget that commits funds for barracks, for unaccompanied personnel, as well. I would assume that you have a priority list already of housing that needs to be constructed to replace old housing and housing that needs to be constructed to provide for personnel, essentially new housing.

    You also are experimenting, if that is the right word to use, with private developers and have a couple of demonstration projects. You have 50 additional family housing candidate projects under review. How do you determine your priorities there? Share with me some of the division of budget monies for those housing needs.

    Mr. LYNN. What we do is we try to do, on a case by case basis, a review of the potential of each of those 50 candidates for use of these privatization tools that we have been given by the Congress. These involve loan guarantees, leasing of property, guaranteeing some kinds of rent, and guaranteeing some kind of occupancy. There are a variety of incentives we can provide to the private sector that will cause them to invest their resources in the housing.
 Page 23       PREV PAGE       TOP OF DOC    Segment 1 Of 3  

    We have to do this right. We have to go through and make sure we have bond rating and we have protection against foreclosure. We have to review base closure potential and it is a complicated legal review. That is one of the reasons it has taken some time, as the Chairman mentioned at the outset. So we are trying to make sure we go through all of those hoops and get this right.

    When we have it right, as we think we have in Corpus Christi and Fort Carson, we can invest a fraction of the resources, sometimes a third, sometimes even as low as a tenth of the government resources that are needed and do that through loan guarantees and other devices rather than direct appropriations and we get the housing for the troops and their families.

    Mr. CRAMER. So with that issue you think that the time delays offer you more benefits in the long run and that it is worth going through?

    Mr. LYNN. We are quite sure of that. And we think we can compress that delay. Part of the delay is just that this is the first time through. We want to be very careful that we do this right and that we protect all the government's priorities as well as that we are choosing.

    Mr. CRAMER. Are there any situations where the private developers will actually own the housing?

    Mr. LYNN. Yes.
 Page 24       PREV PAGE       TOP OF DOC    Segment 1 Of 3  

    Mr. CRAMER. In all cases they will own the housing?

    Mr. LYNN. Not in all cases but in some cases.

    Mr. CRAMER. In some cases they will. And the 50 that were referred to in what I assume is your statement, family housing candidate projects are under review; that is for private development?

    Mr. LYNN. That is right.

    Mr. CRAMER. But you are still proceeding with housing?

    Mr. LYNN. Oh, absolutely.

    Mr. CRAMER. And have a priority list to accomplish that, as well?

    Mr. LYNN. Yes.

    Mr. CRAMER. Quickly back to your expectations of two rounds of BRAC, and my colleague from Mississippi brought up some very legitimate points with regard to the past rounds of BRAC. I know my community there in Alabama at Redstone Arsenal, we inherited a number of BRAC jobs from other commands and it was very, very difficult to sort through the units that were supposed to come, to cause those commands to be efficiently located in one place. The resistance there—I would resist it, too, if it were the other way. So it is bound to happen. I would assume that you have a lot of loose ends left that would motivate you to want two more rounds in a BRAC.
 Page 25       PREV PAGE       TOP OF DOC    Segment 1 Of 3  

    In other words, are your new rounds of BRAC motivated as much because you didn't accomplish everything you felt like you should have accomplished in the past rounds or because you just need to extend the closing to more bases?

    Mr. LYNN. Both. I think at one point both the '93 and the '95 rounds were projected to be larger than they, in fact, were. What we found, frankly, is that there is a limit to what the Congress, the public, the Department can absorb in any one round. You can't just load it up and have a massive round. You need to do this in segments.

    Mr. CRAMER. Any time you are transferring thousands of personnel from one location to another you have housing needs, you have working area needs, you have all kinds of military construction issues that would have to be a big domino into the military construction budgets that we are reviewing.

    Thank you, Mr. Chairman.

    Mr. PACKARD. Thank you, Mr. Cramer. One of the reasons we were anxious to pursue privatization as an alternative, a secondary process, is that it would allow us to leverage federal dollars with the private sector and thus try to catch up with the backlog much quicker.

    In some instances our hope is that they will also develop a long-term maintenance agreement with the owner and they would maintain the facilities, we hope, better than we have done in the past with military money. That is our hope. We recognize that not every project fits into that mold and they are sifting through that now because again they haven't come up with the mold. They are still developing that and I guess——
 Page 26       PREV PAGE       TOP OF DOC    Segment 1 Of 3  

    Mr. CRAMER. But making progress, I hear.

    Mr. PACKARD. That is what I understand.

    Mr. Tiahrt.

OVERALL FUNDING

    Mr. TIAHRT. Good morning, Mr. Lynn. I don't know you yet. I am sure we will get to know each other better in the future.

    I have to tell you I am very disappointed with this budget. I think it falls short. There is not enough money to address quality of life issues or the operational issues or maintainability issues. I don't think it considered the five-year plan (FYDP). It may have in part but certainly not in whole.

    And I want to take off a little bit on the BRAC because I believe that there is not going to be support for a fifth or sixth BRAC because we have not completed the first four. And I believe this Administration politicized the process and will not have any support on the House floor until we get beyond that politicization and complete the first four BRACs.

    I also believe your budget ignores the needs of our troops, particularly in the quality of life. I come from Wichita, Kansas, which is the air capital of the world. We have Boeing and Beech and Cessna and Learjet with all large facilities there. We also have McConnell Air Force Base. And we know of the demand for pilots in the next decade, and that demand exceeds the number of pilots we currently have in the military.
 Page 27       PREV PAGE       TOP OF DOC    Segment 1 Of 3  

    You know, it is very appealing to go fly for an airline and we have to do something to hold these people in the military, to make it at least appealing to them to stay and serve our country. And those are all related to quality of life issues and I am concerned that we don't address those quality of life issues in the Administration's military construction budget.

    And I see it sort of as starving our military and it is really not acceptable. We have not budgeted for Bosnia in this year's budget. And last year we were told that the Administration would budget for Bosnia, but it is not there. Now, is that extra money going to come out of military construction?

    We haven't budgeted for a military strike in Iraq. Where is that money going to come from? Is it going to be coming out of military construction? I think this committee is going to work very hard to see that we do the right thing for our personnel. And last year we fought very hard to do what we believed was the right thing for our personnel, and yet we suffered the veto pen.

    So before the President writes the first letter with his veto pen, I hope he will consider some things. First, let's check the five-year plan, the FYDP. Make sure that we take that into consideration. We believe that the people who are working the hard work of freedom, where the rubber meets the road, know what their needs are better than we do, so we listen to them. And I would ask the Administration to do the same thing.

    Second, let's complete the BRACs that we already have gone through before we start another one. We have bases out there that need to be closed, some that the process needs to be completed on. When we get that done we will be in a better financial state, and you will certainly have more support in the House of Representatives to move on to another BRAC.
 Page 28       PREV PAGE       TOP OF DOC    Segment 1 Of 3  

    And last of all, I want you to consider the quality of life issues related to keeping our personnel. It is a volunteer service and if it is not appealing, they are going to go on. And next year I hope that you will fulfill last year's promise to budget for Iraq and for Haiti and for whatever else comes up including Bosnia. We want to make sure that that is in the budget that comes from the Administration so that we don't feel like we are vulnerable to being robbed of the needed resources to keep our military personnel in a good quality of life, with the operation and maintainability that they need through capital investment.

    Mr. LYNN. Let me respond. I think we are more in agreement on Bosnia than it appears. On Bosnia in the budget that is before you, and let me come back to '98, but the FY 99 budget that is before you, we have proposed that Bosnia and any increased cost in Southwest Asia be paid for out of a $3 billion allowance that is outside the defense budget.

    What we are proposing is along the lines of what you were saying, that we cannot afford to divert resources from military construction, from family housing, from readiness, from modernization and from maintaining those things the way we think we need to maintain them.

    What we should do is pay for Bosnia within this allowance, which is outside the defense topline. If the Congress approves that, we would be able, as I say, to protect those defense priorities, and that is indeed what we have proposed.

    So I think we may be in agreement on that.

    Mr. TIAHRT. That is encouraging.
 Page 29       PREV PAGE       TOP OF DOC    Segment 1 Of 3  

BASE REALIGNMENT AND CLOSURE

    Mr. LYNN. Now, let me come back to your first point on BRAC. I take your point that you want to complete the first four rounds before embarking on a fifth and a sixth round. That is part of the reason that we have shifted the years. The first four rounds will be completed by 2001, the time a fifth round would begin. So you would have that kind of breathing space as you proposed.

    Mr. TIAHRT. There have been bases that we are currently holding open that were scheduled for closure. So you're saying that this has been stopped further down to the right or further on in time?

    Mr. LYNN. No, I am saying that the projection right now for the closure of the last bases in the fourth BRAC round, the 1995 round, all of them would be closed by 2001 and that would be the starting point for the next round, so there would be a gap.

    Similarly, you mentioned politics involved in this and I understand the controversies that surrounded the last base round. By moving it out a couple of years, you try and insulate it Instead now of '99, you now have it in 2001 and you will be able to, I think, start with a clean slate in that way.

    So we think that we have taken on both of your points and addressed them with the 2001 timing.

 Page 30       PREV PAGE       TOP OF DOC    Segment 1 Of 3  
    Mr. TIAHRT. Well, they got tangled up in the presidential campaign last time and now we are setting ourselves up for the same problem, and that is not the way BRAC was designed. It was designed to be set aside from the political process. You set the commission up, you designate the bases and they close.

    Instead, we are seeing it caught up in the political process and that is going to damage BRAC. Right now it has no legs on the House floor because of that.

    Mr. LYNN. I understand the concerns. As I say, that is why we would propose that it be done after rather than before the next presidential election, to try and get it out of those politics. If you look at the detailed proposal, you will see that we have changed the dates somewhat, just a couple of months, but what we are trying to do is ensure that the incoming administration will have the opportunity to appoint the commissioners as well as review and submit the list.

    Under the prior schedule, within a couple of months of taking office the new administration would have had to move forward on this. We are trying to build in a couple of extra months so that there is time for that new administration to review this. It will be that new administration, after the next election, that will put forward the realignment and closure list.

    Mr. TIAHRT. Thank you, Mr. Chairman.

    Mr. PACKARD. Thank you, Mr. Tiahrt.

    Mr. Olver.
 Page 31       PREV PAGE       TOP OF DOC    Segment 1 Of 3  

AIR GUARD AND AIR RESERVE

    Mr. OLVER. Thank you, Mr. Chairman.

    I want to ask you a question, Mr. Lynn. I can't imagine that you would have the answer offhand but somebody around here might have it and I will come back to it and maybe get the answer after another couple of parts of my questioning.

    Can you give me the number of people who are presently in filled positions, not authorized positions but filled positions in the Air Reserve, the Air Guard, the Army Reserve and the Army Guard?

    Mr. LYNN. The actual number of people on board is what you are looking for?

    Mr. OLVER. Yes, the filled positions, the positions that are people on board, as opposed to whatever the——

    Mr. LYNN. The authorized spaces. I am not sure we are going to have that here. We don't have the personnel people here but we can provide that to you for the record.

    Mr. OLVER. Then since you won't have that during my time of questions, whatever I may have, I would like to know what the filled positions in those categories were at the end of fiscal years '96, '97, '98 and your projection for 1999 because those are—there may be more that are important but those are key to this question of base structure in line with force structure, which is driving your arguments in relation to base closure grounds in the future, which, by the way, I think your timing is very responsible, given the need for completion and cleaning up loose ends and so forth and getting it away from the political process, away from the political process that it had been in the last time.
 Page 32       PREV PAGE       TOP OF DOC    Segment 1 Of 3  

    So I would like to see those data over several fiscal years if you would, please.

    Mr. LYNN. We will provide that.

    [The information follows:]

Table 1



    Mr. OLVER. If there is any question, whoever is going to do this might talk with me later because I can obviously get more sophisticated questioning out of it.

OVERALL FUNDING

    My other comment, I want to just reiterate and support what my ranking member has said about adequacy of budget, and others have already alluded to that I have heard here today. I think we are functioning under—the Chairman's comments are very much in line there—I think we are operating under a kind of formula which says to request 15 percent less than you had last year, 15 percent less, and so on.

    Let me tell you that I think that that is rather tragic, given that we are talking about housing and quality of life and the training facilities for men and women up and down the line through reserve and reserve components, as well as the active components.

 Page 33       PREV PAGE       TOP OF DOC    Segment 1 Of 3  
    One year ago, and I have only been on this committee just for the last year, one year ago the budget request was 15 percent of what had been enacted the previous year. This committee added onto that and the final budget, the only one of the 13 budgets going through this place that ended up being below what had been the previous year's budget was 6 percent below, ultimately, requested 15 percent below. We added on and brought it up to only 6 percent below.

    So the request this year, today, one year later, is 15 percent below last year's enacted budget. And I would hope that, Mr. Chairman, that when we get done here, that we aren't going to be a penny below last year's enacted budget because I think that that is what in quality of life issues, in training facilities and in housing and day care facilities and health facilities, it seems to me that that is what we owe the people who are serving us.

    So let me go on, if I may, to a different issue. I probably won't have time to really flesh this out but I may sit around here until we have another round of questioning and finish fleshing it out.

AIR GUARD AND AIR RESERVE

    You had spoken in part in your testimony about the review of the assessment of how projects are coming forward, construction projects are coming forward. I wonder if you would review for me how those are assessed, particularly in the reserve and guard components, the Air Reserve, the Air Guard, the Army Reserve and the Army Guard components.

    Do we have a common set of procedures by which those are assessed, some sort of objective set of procedures or criteria by which they are assessed or is it done in the Air Reserve and Air Guard different ways from the Army Guard and Army Reserve, each in their own way?
 Page 34       PREV PAGE       TOP OF DOC    Segment 1 Of 3  

    Mr. LYNN. The general process is a devolved one in the sense that the initial responsibility for developing those requests does lie with the military departments.

    So the Army goes through its process and the Army Guard and Army Reserve projects are considered against active duty projects, as well as the other needs of the Army. The Army puts together what they consider to be a balanced program. The Air Force and the Navy Departments do the same.

    Frankly, the Air Force, the Air Force and the Navy have done a better job of integrating the guard and reserve components and considering those needs.

    We are aspiring to bring the Army up to that level. That is what we would like to do. That involves actions by us and actions by the Army.

    As I indicated, we took the initial steps this year with the defensewide process when those military department programs were submitted. We have now pulled the guard and reserve in. They are at the table. They are heard. The Secretary listens to their criticisms, their priorities, how they would assess the program before he makes his final decisions on what to submit to Congress.

    As a result of that, this year there were some changes, increases in every case, made to the Guard and Reserve, not just in military construction but operating tempo, personnel, and equipment accounts as well.
 Page 35       PREV PAGE       TOP OF DOC    Segment 1 Of 3  

    I think we need to continue. I don't think we are done. I think we need to continue that process and to try and get what I suggested at the outset was a total force approach.

    We are never going to have all the resources we think we need to meet all of the priorities. We are going to have to make choices. But the choices should be made from a total force approach.

    Mr. OLVER. Is it made in an objective way? Is there some set of criteria that all of these groups—the Army Reserve, the Air Reserve—of course, Naval Reserve, they don't have guard but the Air Guard and the Army Guard, they are all using the same set of criteria for how you assess what is the need for these particular projects?

    Mr. LYNN. I wouldn't say they are all using the same criteria. They are different kinds of units with different kinds of missions. What we try to do, at a general level, we fund the first-to-fight units, the units that are most likely to go in the earliest stages, at a higher priority than units that are, say, in strategic reserve. You will find that kind of general priority across all of the services.

    But when you get more specific, the Air Guard is integrated into the Air Force in ways that are different from the way the Army Guard is integrated into the Army. They get into more specific kinds of metrics and criteria that are unique to those missions and units.

    Mr. OLVER. Mr. Chairman, I will come back to it. I will pick up right where we were.
 Page 36       PREV PAGE       TOP OF DOC    Segment 1 Of 3  

    Mr. PACKARD. That will be fine, if you can catch it on the next cycle.

    Mr. Wamp.

    Mr. WAMP. Mr. Chairman, at the risk of sounding like Martha Stewart, let me recommend that while we are gone next week that we take those five seals and put them above these pictures here, above the speaker's chair for our Department of Defense and respective service academies. I kept wondering last year why we didn't do that, so I am just going to recommend that we do it.

    Second thing, I want to applaud Mr. Hefner for his career and his service, his commitment. I grew up a Democrat and he was my kind of Democrat. He is my kind of Democrat—good, conservative——

    Mr. HEFNER. Would the gentleman yield?

    Mr. WAMP. Yes, sir, I would be happy to yield to you.

    Mr. HEFNER. I was born in Tennessee. In fact, they are going to build a log cabin where I was born. But I thank you for that.

    Mr. PARKER. Would the gentleman yield? Bill is the reason I left the Democratic Party.
 Page 37       PREV PAGE       TOP OF DOC    Segment 1 Of 3  

    Mr. WAMP. I didn't mean to open up a can of worms. I reclaim my time. But I do applaud you, sir. This institution is sorely going to miss you. I think you are a tribute to the people of North Carolina and I wish you all the best.

    And I welcome my good friend and neighbor, Congressman Cramer, to the subcommittee. He and I adjoin, our districts, and have worked together in a bipartisan way many, many times.

PRIORITIES

    I also want to say I think this dilemma that we find with the President's budget request and this subcommittee is a perfect example of the problem that the House faces, that the Congress faces in the shift of constitutional authority on funding over the last generation, and it is a problem. I think more and more the Congress is giving up its constitutional responsibility for spending the taxpayer dollars to the presidency.

    This is a perfect example of why the Congress needs to be the funding agent for the people and why we know the needs and we are close to the ground and we have the hearings and you can't just come through with some kind of magical pen every year without the hearings, without knowing the needs, without staying on top of the needs through these hearings for setting these priorities up.

    To me, it is like trying to save money in Medicare and taking it out of beneficiaries instead of the providers. In many ways here that is what happens when you are talking about our troops.
 Page 38       PREV PAGE       TOP OF DOC    Segment 1 Of 3  

    I just want you to know I appreciate the work that you do, sir, but I would like to know what the President's goals are for the quality of life of our active duty troops. What are his objectives? I.e., do his goals include resigning every troop that is serving with honor and does this budget reflect the commitment to carry that forward? I mean, what is the end game, or is this just an annual process of we are going to try to get what we can and meet the priorities elsewhere?

    I really think if you are going to try to set those goals, you have to have a stronger commitment than we are seeing. How do you establish the priorities when you send this budget over, for quality of life? That is the number one issue for me. I hear it through all these hearings—quality of life, resign our troops, safety, security, that spouse that is going to make that decision.

    You know, we have to increase their life and our ability to maintain the quality force that, in the modern era, we have become known for. When we deployed Desert Storm I think we were at a peak and frankly, I think we have suffered ever since then. I just want to know how you set the priorities, or is there a process at the White House where those priorities are actually analyzed?

    Mr. LYNN. I think the President's priorities are what he has received on advice from the uniformed military leadership and those priorities are four: pay, retirement, medical and housing. Those are the four highest priorities for quality of life that General Shalikashvili first recommended, endorsed by the Joint Chiefs of Staff. I know General Shelton and General Ralston and the current chiefs endorse those four also.
 Page 39       PREV PAGE       TOP OF DOC    Segment 1 Of 3  

    Mr. PACKARD. Let me interrupt for just a moment. Before Mr. Parker leaves I want to mention that he also is retiring and he is going to be sorely missed by this subcommittee.

    Mr. PARKER. Mr. Chairman, I am not retiring. I am going to the House.

    Mr. PACKARD. I don't mean that he will be retiring today. I mean at the end of this term and we certainly appreciate him.

    Now please proceed.

    Mr. LYNN. As I said, the four priorities, and these are the strong recommendations of the current and immediate past Joint Chiefs. Again, pay, retirement, medical care and housing. Those are the four key ones. Let me say where we are.

    On pay, there is a 3.1 percent pay raise funded in the budget all the way through the FYDP. I don't want to say 3 percent is a huge amount of money but it is, given where inflation is, a real increase in buying power this year. It is higher than inflation. It is certainly well needed by the troops and their families.

    On retirement, there have been no changes but the retirement program has been protected and is again fully funded.

 Page 40       PREV PAGE       TOP OF DOC    Segment 1 Of 3  
    On the medical, I indicated in response to Mr. Hefner's question that we found that the program was underfunded and we added over $2 billion across the FYDP and about $500 million in '99 alone to try and ensure that the defense health program is fully funded, and that we don't run into the kinds of issues that we have run into in past years.

    Mr. KINGSTON. Would the gentleman yield a minute on that? I would like to ask, is that for active and retiree? And if it is for both, what is the split?

    Mr. LYNN. It is for active and retirees, for the whole defense health program, which is around $10 billion. The split—I don't know what the split is in dollars. I believe just over 50 percent of the care now goes to retirees. How that tracks in dollars, I am not certain, although retiree care would tend to be higher, just because of the demographics.

    So those are the first three. Housing is indeed the fourth priority and I think that is—I am sure the congressman would agree—that that is where we need to continue to do the most work. We think through the privatization initiatives, we are going to be able to leverage the money we have. But as the committee uniformly has noted, the resources don't match what the committee provided last year and I think we need to address that in the out-years.

    And, as I indicated in my opening statement, we think that the growing savings from the Quadrennial Defense Review will allow us the opportunity to do that.

    Mr. PACKARD. Mr. Wamp, would you yield for just a moment on that?
 Page 41       PREV PAGE       TOP OF DOC    Segment 1 Of 3  

    Mr. WAMP. I would be happy to yield, Mr. Chairman.

    Mr. PACKARD. You have pointed out, I think properly, that in each of first three priorities—pay, retirement, medical—that you either held the funding or increased the funding. It is no priority when you make a 32 percent cut, and that is what you have done in housing. That is not a priority. That has been dropped off your list as a priority. You just can't consider that a priority if you have made a 32 percent cut.

    So even though you are trying to address it, we will look at that very carefully but I would suspect that there is no way in the out-years that you are going to be able to recapture that 32 percent loss that we are sustaining this year.

    Mr. WAMP. Reclaiming my time, if you had four children in college and you told three of them that we are going to give you the same budget you had last year or more and one of them you are going to cut 32 percent, that child would be a proverbial redheaded stepchild, and that is where we are. That is the point the Chairman made.

    Mr. HEFNER. Would the gentleman yield?

    Mr. WAMP. Yes, sir.

    Mr. HEFNER. The problem as I see it goes back to when I was chairman for about 10 years before Mr. Packard. You talk about priorities. The guys across the river get together and work their special programs. This happens especially with the defense bill that you are talking about. ''I instigated this program,'' whether it is the B1, the M1 tank or whatever. ''This is my program,'' and they shepherd it through and they get the money.
 Page 42       PREV PAGE       TOP OF DOC    Segment 1 Of 3  

    But they don't really get the money, when it gets down to military construction and housing projects. They figure those guys over there that have all these bases are going to look after the military guys. This committee, is going to look after that, so we won't put too much priority on it because they are going to beef it up.

    Then we come in here with reduced budgets compared to the Reagan Administration and the Bush Administration, With these reduced budgets it is hard for us to get a fair and equitable allocation.

    Last year or the year before last—I forget which year—we had a rescission and we were responsible for the major part of that rescission in this little committee, and this is, moneywise, a small committee.

    So I still maintain that President Clinton and previous Presidents, Reagan, Bush or Carter don't really have any idea what quality of life is all about. We didn't use to even talk about quality of life until a few years ago. Now you have a subcommittee on quality of life here in the House.

    So I don't think the people down on Pennsylvania Avenue are as concerned about military folks as they should be. I don't think they even zero in on quality of life and housing for our military. It is just something that is not real high on their radar screen.

    And we have fought it every year. We have talked to the chiefs as they come in here and told them that you are going to have to be a little bit more concerned about your troops. And they all say, ''Well, we are working hard over there. We are working hard.''
 Page 43       PREV PAGE       TOP OF DOC    Segment 1 Of 3  

    Then we come back with this budget and I am assuming that the chiefs have all signed off on this.

    Mr. LYNN. Yes.

    Mr. HEFNER. So where do we go? We don't have anyplace to go except to the Budget Committee and say, ''Hey, we have to have more allocation.'' Then the people in all the other committees say, ''Hey, we can't give up any of our allocation.'' So we wind up behind the you know what.

    I thank the gentleman.

    Mr. WAMP. I yield back, Mr. Chairman.

    Mr. PACKARD. Thank you.

    Mr. Kingston.

BASE REALIGNMENT AND CLOSURE

    Mr. KINGSTON. Mr. Lynn, let me say a guy told me one time you can fire somebody at the U.S. Post Office but you have to have the stomach to do it. It takes a long time to get through the process, and I think that is the process you all are in with the BRAC. I realize that you are damned if you do and damned if you don't but I don't know that you can really be expected to fully implement all of BRAC. And I do think that a lot of people use that as a shield to stop the fifth round.
 Page 44       PREV PAGE       TOP OF DOC    Segment 1 Of 3  

    So it is a horrible job but I don't know that you should necessarily stop. I have military bases in my area that are very important to us economically but no one is guaranteed a military base. The point of the military is to fight and win wars, not economic development for an area. I just want to say that.

    You know, deployability may be outside your realm but we do talk about quality of life in today's military. You have a lot more young moms and dads in it. As I understand it, during Desert Storm there were a lot of popular bills that popped up that nobody who was pregnant would be deployed and so forth. You know, there is a good humanitarian argument for that but it is a very lousy military situation.

    Under your realm is there anybody looking under the deployability of the United States armed services right now? You know, the question that if we get involved in Iraq, Bosnia, wherever, how many of the soldiers that we are counting on are actually deployable? Do we know?

    Mr. LYNN. We do know. I don't. You would want to talk to Rudy de Leon, the Under Secretary for Personnel and Readiness. We can get you the answer for the record but I can't provide it to you at this minute.

    [The information follows:]

    As of September 30, 1997, the number of permanent non-deployables is 3,817. Of that number, 3,265 are male and 552 are female. The number of temporary non-deployables is 59,316. Of that number, 42,168 are male and 17,148 are female.
 Page 45       PREV PAGE       TOP OF DOC    Segment 1 Of 3  

    Figures on non-deployable personnel are provided each year in the Secretary of Defense's Annual Report to Congress.

    Mr. KINGSTON. It has been suggested that it is—you know, we are sitting on a volcano here because we are not as deployable as we think we are because of all the young moms and dads and single moms and dads that are out there.

CHILD CARE

    Child care. I had one of the more respected members of my military advisory community tell me that child care is a loser for posts and bases, that we are pushing child care but it is a loser. It takes a lot of money and we should privatize military child care. How do you feel about that?

    Mr. LYNN. It is certainly something we should look at as far as the defense reform initiative. We are trying to look at any functions that aren't core functions in terms of the fighting forces. We ought to look at privatization of child care. Certainly that is a candidate.

    I think we have quite a strong system. You have seen the President has actually been using DOD as an example in some of his child care initiatives for how well we are doing.

    So I think we want to make sure that as we look at privatization in that area, we don't lose the quality that we have obtained because, for exactly the reasons that you are indicating, we have shifted to a very married force, a very family-oriented force. If we are going to be able to deploy people and have people ready, we have to provide them with the kind of child care they need.
 Page 46       PREV PAGE       TOP OF DOC    Segment 1 Of 3  

    I think you can see a change in each of the military department's attitude toward child care over the last decade. It is a consequence of how the make-up of the force has changed.

    Mr. KINGSTON. I think we should certainly continue providing child care but based on the military mission, it really murks up the water, if you will, and I think we should really move toward privatization of these child care facilities, as I understand it, because it is taking money out of other things, anything from quality of life, pay raises and retirement, to maintenance of weapons.

    One issue that comes up from time to time is when you have a military post or base and they have housing there, and the local community has entrepreneurs who have apartments and so forth, they feel that they are competing and they should not be competing.

    And not only is it limited to housing but often to restaurants who have to compete against an officers' club, which is now open to the public and can advertise on the radio, ''Come eat at the officers' club.'' The price is about the same but the officers' club really isn't there to compete against other restaurants.

    Are we addressing this or are we just kind of fighting these on a case by case basis?

    Mr. LYNN. There is no general policy in the Department, to my knowledge, at this point. It is being worked on a base by base basis.
 Page 47       PREV PAGE       TOP OF DOC    Segment 1 Of 3  

    Mr. WAMP. Would the gentleman yield?

    Mr. KINGSTON. Yes.

    Mr. WAMP. Last year I know when we addressed that issue clearly troops would rather live on the base in most cities because of security. So that frankly puts those private developers off the base unless they are involved in one of our privatization programs, at a competitive disadvantage because the security on the base is much more amenable to the average troop.

    Mr. PACKARD. And usually, if you would yield, usually there is a differential between the housing allowance and what it costs them to live off-base.

    Mr. HEFNER. If the gentleman would yield, on the other hand, the people who say, and I remember in King's Bay before you came here they always brought the mayor up here from—what little town is that near——

    Mr. KINGSTON. St. Mary's.

    Mr. HEFNER. They would introduce the mayor and he would give this story about how tough it is and all this stuff. So one day we just said, ''Well, would it be satisfactory with you if we closed the base?'' ''Oh, no, no, no.''

    So the people that are working outside the base, they can get the support people to come in, so they have not an advantage but they have the possibility of renting their stuff and they sure wouldn't want the base to close under any circumstances.
 Page 48       PREV PAGE       TOP OF DOC    Segment 1 Of 3  

    And this program that they are embarking on, what amounts to a housing authority, I think has some real merit to it.

    Mr. KINGSTON. I think it does, too.

    Mr. HEFNER. It is going to let entrepreneurs go in and it is going to guarantee them occupancy and it is going to be that after a certain time it will be theirs. Of course, I can't see them moving it off the base but I think that has some real potential.

    I thank the gentleman for yielding.

    Mr. PACKARD. There is a vote on and I have several questions that I wanted to ask and I know that Mr. Olver had some questions.

    Mr. OLVER. I do. If you want to ask, I will go vote. If you want to let me ask, you can go vote. Or we will come back.

    Mr. PACKARD. You go ahead. I will run and vote and come right back because I would like to spend just a little bit of additional time. If you need to adjourn for just a few minutes, do that.

    Mr. OLVER. I think I can probably filibuster until you get back.
 Page 49       PREV PAGE       TOP OF DOC    Segment 1 Of 3  

PRIORITIES

    Mr. Lynn, to go back to where we were before, and this way only the staff will have to endure this, rather than all the members. The question I was asking, and now I will add a little bit to the original question because it will be fairly easy for you to give it, I think.

    The question I was asking about the number of filled positions was to get a sense of how fast the force structure is changing over the last few years as we have gone through the recent BRAC commissions and what your projections are for the future in order to assess how your goals fit in with what is actually happening on the ground. There also is another purpose but that is the key purpose.

    So I think I was saying give me, please, the Air Reserve and Guard and the Army Reserve and Guard, but I think it is probably just as well if you gave me the air active, army active and the naval active and reserve. So there would be eight—it is just a chart, eight groups. The army active reserve and guard, air similar and naval active and reserve.

    And then I think the chart ought to be a little bit longer. Rather than just going to projections for fiscal '99, it ought to include fiscal '00 and fiscal '01 because that would get us really up to where the BRAC commissions are.

    That would help me, at least, understand what has happened to the force structures and what they are likely to look like and see how this all makes some sense.
 Page 50       PREV PAGE       TOP OF DOC    Segment 1 Of 3  

    Now, to go back to the other question where I was asking about how projects that come to the FYDPs were being reviewed, I wasn't sure, and maybe the question as I put it forward is not sufficiently precise so that you would know what I was looking for but I have been following last year the procedure by which Army Guard projects came up and in part, we all discovered, I think, and maybe everybody else knew, I am sure—I certainly discovered that the procedures in the Army, the relationship between the Army Guard and the Army active and reserve are different, say, from the procedures and the relationships between the Air Force active, reserve and Air Guard. They seem to be somewhat more coordinated in the case of the air, rather than the army.

    And I had then asked questions and was told that there was a procedure being developed, in the case of Army Guard, for assessing in an objective way, hopefully objective way, what projects ought to be put forward. And the three criteria being used there are what you called first-to-fight, which is how it fits into the readiness needs of the whole force structure; secondly, the adequacy of the facility as it sits in place; and thirdly, the state priority. Well now, state priority only relates to the Army Guard and the Air Guard.

    So what I was wondering was whether this was a common set of systems. I am told that the issue of first-to-fight essentially represents 40 percent of the total determination, that the issue of adequacy of the facility represents 35 percent of the determination and the priority, the state priority represents 25 percent of the determination of what the score is that projects get in the consideration of the Army Guard Bureau.

    I was wondering whether there was a similar sort of situation, same or similar, that represents the criteria, the objective criteria being used in looking at what Air Guard projects are under consideration.
 Page 51       PREV PAGE       TOP OF DOC    Segment 1 Of 3  

    And then, of course, I am projecting backward. I am wondering are we using, for instance, for considering Army Reserve or Air Reserve projects and how they are to be assessed, are we using only the first-to-fight and the facility adequacy as issues under consideration or some other set? Maybe you could tell me whether there is some set of criteria in each of these categories that defines how those projects are being put forward.

    Mr. LYNN. I am afraid I can't. I think you will have before you during your month of hearings the representatives from each of the services who are, I think, doing what you have suggested. They are the ones who are putting forward the MILCON projects in their individual component and how the Air Force system compares to the Army system, I'm afraid, is a question that I can't answer at this point.

    Mr. OLVER. They are in deep trouble because that will mean I will have to ask every one of them as they come in, what is the procedure by which they are bringing forward those projects. I think as one looks at what has happened in the case of the guard, it is quite interesting what projects are coming forward.

    Mr. LYNN. I don't understand where you want an answer on that.

    Mr. OLVER. Well, in relation to the Army Guard, my read of the objective scoring system is I see all of the projects which have the highest score are sitting out in '03 and beyond.

    Mr. LYNN. As I said, I am not familiar with the scoring system that you have described. The Army does that internally. I think you would have to discuss that with them. I can try to get smarter on this and come back to you but I just can't answer your question.
 Page 52       PREV PAGE       TOP OF DOC    Segment 1 Of 3  

    Mr. OLVER. I thought you might have some sense of whether this was a procedure that was being implemented across the eight, really. I have now discussed it in relation to four of the eight categories, whether it was being done across all those categories or only in those four or maybe it is only in the one. Maybe it is only being done in the Army Guard, but I will explore the answer to that over time.

    Mr. SODANO. We review the requests that come in for executability, pricing and whether they satisfy the most critical requirements, but we review what the services normally submit. We don't go out into the out-years.

    So placement in the out-years is really dependent upon the services' priority and whatever scheme they use to rank those. That is pretty much up to the individual service.

    So the Army may do it one way, whereas the Air Force may do it a different way.

    Mr. OLVER. Maybe there ought to be a systemwide approach to this because if you put forward a so-called objective scoring system and all the projects with the highest score are sitting out in '03 or beyond, then I would wonder how projects are getting—what is the point of having an objective scoring system if all the highest scoring projects are out there. If you have right criteria, which it seems to me first-to-fight is a perfectly reasonable criteria, and adequacy of facilities is a perfectly reasonable criteria.

 Page 53       PREV PAGE       TOP OF DOC    Segment 1 Of 3  
    When you're talking about guard structures, priorities at the state level ought to have a certain play in this. It seems to me that those are reasonable criteria to consider. But if the end result of that is a scoring system that puts all your highest scoring projects out in the '03 and beyond, something is not playing out.

    Mr. LYNN. If the situation is as you describe it, obviously there is a problem. I am not familiar with that scoring system or the placement you describe in '03.

    Mr. OLVER. All right. So that is the rest of my comments and I think we will have to recess until the Chairman comes back.

    [Recess.]

    Mr. PACKARD. We will reconvene. Let me go into some of the questions that I had.

LINE ITEM VETO

    I was going to have a question on the line item veto. As you know, we overrode rather overwhelmingly in the House and I expect them to do the same in the Senate and I wanted to—I was going to ask really—first of all, I was going to comment that we would hope that we are never put in that position. We did not want to be in that position. We felt that we did our work and in this instance there were some things that we didn't think the President did.
 Page 54       PREV PAGE       TOP OF DOC    Segment 1 Of 3  

    We would hope that on any line item veto that we would be consulted before and at least brought in because we are not the enemy. We are trying to accomplish the same goals.

    But that all may be moot now. I just have an announcement that was handed to me from the Associated Press that a federal judge today declared the President's new line item veto authority unconstitutional, so it may be a moot point.

ADVANCE APPROPRIATIONS

    Let me cover a couple or three other items, though, that I think are important. For the first time in over a decade, we see where in your budget request you are requesting the advanced appropriation opportunity. Would you explain why you are seeking that and how you expect it would benefit in terms of your budgeting?

    Mr. LYNN. The answer to both questions is the same. We thought that in a certain number of projects, longer ones that take longer than 26 months, the advance appropriation would allow us to get the most out of the individual year's appropriations for construction projects.

    Mr. PACKARD. Do you have any criteria that you use to determine which projects you would use that procedure?

    Mr. LYNN. They have to be longer than 26 months and the contracts have to be separable.
 Page 55       PREV PAGE       TOP OF DOC    Segment 1 Of 3  

FAMILY HOUSING CONSTRUCTION REDUCTIONS

    Mr. PACKARD. Okay. On the family housing construction request, we have talked quite a bit about that, the fact that we are concerned about how much of a reduction is being requested. But in terms of privatizing, do you see privatizing eventually replacing our traditional way of funding base and family housing and other infrastructure, or do you feel that it will be a supplement to our existing program?

    Mr. LYNN. I don't think we know yet. At this point it is a supplement. I think the Army in particular is interested in being very aggressive and moving heavily in that direction. We are still in the stage of reviewing the projects that we have and seeing where this is applicable, and I don't think we can see yet what the end point is.

    There certainly are going to be some projects that are not going to fit and we are going to have to retain the traditional means to do that. What the proportion is going to be at the end, I would hesitate to predict.

    I should add that what we are going to do, I think, is try and use the most effective means we have to replace the housing on that 10-year schedule. So if that means going heavily in this direction, we would be inclined to. If it means more of a mix, we would do that.

PRIVATIZATION

 Page 56       PREV PAGE       TOP OF DOC    Segment 1 Of 3  
    Mr. PACKARD. Do you feel that you are aggressively pursing the privatization?

    Mr. LYNN. Oh, absolutely. I think that the numbers show that.

    Mr. PACKARD. And I was particularly pleased to hear your comments early on regarding utilities, particularly underground utilities, expensive replacement of utilities that you are looking at the private sector in that area. I think that is a good effort and obviously we hope that your review will help you to determine what is the best course of action to take.

    I am not saying which is the best course of action but I have felt that one of the neglected areas for some time is the very expensive and yet often antiquated underground utility system that many of our bases have.

    Many of our bases are World War II vintage, at least, and that is when the water systems and the sewer lines and the power systems and all of the other—your drainage systems and so forth—were put in, and often they have outlived their life and they are going to require huge amounts of money to replace.

    Your utility companies and a variety of other private sector opportunities are there and we hope that you will look at it carefully but recognize that—and it's true in housing—that at the present time it is a supplement. It doesn't replace. It may eventually move in that direction but right now I think we would hope that you would not throw all your eggs into the private basket.
 Page 57       PREV PAGE       TOP OF DOC    Segment 1 Of 3  

    Do you have any comments on that?

    Mr. LYNN. I think we are thinking along the same lines as you, Mr. Chairman. We think that there is an enormous amount of potential in the privatization of the utilities.

    As you know, it is not without controversy and we need to work through those issues as we do it, but the potential is certainly there.

FAMILY HOUSING IMPROVEMENT FUND

    Mr. PACKARD. It appears that you are going to be shifting the costs of housing from construction and operation and maintenance accounts to personnel accounts. I think your comments indicated that. Do you anticipate any savings next year?

    Mr. LYNN. The impetus here is not really savings. It is really trying to get the most out of the housing dollar. We have a very large inventory, 345,000 family housing units. We need to turn as many as two-thirds of those over, replace or renovate two-thirds of those—by 2010. And what we are looking for is to be able to leverage the dollars we have to achieve that goal. We are not really looking to generate savings for other priorities in that instance.

TRANSFER AUTHORITY

 Page 58       PREV PAGE       TOP OF DOC    Segment 1 Of 3  
    Mr. PACKARD. The transfer authority, your budget proposes a provision that would allow you transfer up to $200 million between accounts. Why do you seek that?

    Mr. LYNN. Flexibility of management, to be able to meet the needs of the Department. It's a fact of life that changes come up.

    Mr. PACKARD. $200 million is a lot. When you are looking at, in your budget, a $7.8 billion budget, that is a pretty healthy percent. What percent would that be? About 1 or 2 or 3 percent.

    You are not going to get that. I can almost assure you of that because what you are doing is you are bypassing this committee. We have a reprogramming process that allows us to make those kinds of transfers and I think you are going to see that probably continue.

    Nevertheless, I was wondering what—of course, you would like to have all the flexibility you could.

    Mr. LYNN. There is clearly a tension between management flexibility and committee oversight.

    Mr. PACKARD. We have been pretty good at managing to grant transfers. We will continue to do that, but I don't know that you will see language in the bill that will reflect that kind of transfer.

 Page 59       PREV PAGE       TOP OF DOC    Segment 1 Of 3  
    Lastly, it looks like they are still on the first vote so I am glad I came back and we can wrap this up. I applaud what I see happening in terms of your total force concept, particularly with the Army, the way they are trying to integrate the guard and reserve into their active duty programs.

    I went to a briefing. Were you there at that briefing?

    Mr. LYNN. No.

    Mr. PACKARD. I went to a briefing the other morning and listened carefully to their plan and frankly, I liked what I heard. But my concern is that as there is an effort to integrate their functions, I will be very interested and observant as to how they integrate the budgeting process, whether, in fact, the guard and reserve's budgets will become an integral part of the budgeting from the active duty Army. That will be a very interesting thing.

    If that happens, then I think you are definitely moving in the right direction. But if the guard and reserve remain as a stepchild in the budgeting process, as it has in the past, and you go through the rhetoric of integration but if there is not a true integration of budgeting for guard and reserve functions as they are integrated into the Army, active duty Army functions, then it is a sham and I would not be happy about that.

ARMY GUARD AND RESERVE

    But I think that it is moving in the right direction if the budget of the Army Guard and Reserve is also integrated and becomes an active part of the overall budgeting process for Army activities and functions. If they truly become an integral part, a total force part of the Army, then the budget will reflect that all the way through. And if that happens, then I think we have made a good move.
 Page 60       PREV PAGE       TOP OF DOC    Segment 1 Of 3  

    I will watch carefully to see how the budgeting aspect of guard and reserve, Army Guard and Reserve, in light of what the emphasis is and you have certainly been emphasizing this total force and particularly the integration aspect of guard and reserve with the Army itself.

    Do you have a comment?

    Mr. LYNN. I think you have well stated the goal. This has to be more than rhetoric. But I would even go further. I think we need to not only change the processes, like the budget process; we need to change the fundamental relationship. I think the integration has to be at the personal level between the Guard leadership and the active Army leadership and then down the chain, and that is what we are seeking to achieve. And General Reimer, I think that is the briefing that you are referring to, with General Reimer——

    Mr. PACKARD. Right.

    Mr. LYNN. I think General Reimer is stepping out to do that. And I am hoping under his leadership we will be able to make dramatic improvements. If we do, I think they will show up in the budget process, but that will be the manifestation of a more fundamental change.

    Mr. PACKARD. If it doesn't show up in the budget process, then I don't think it will ever show up in reality, in terms of the integration effort. And that means that the guard and reserve people need to be at the table—at the planning table, at the development of the budget table. They need to be at the table where their input is heard. Otherwise they will still look upon themselves as a stepchild.
 Page 61       PREV PAGE       TOP OF DOC    Segment 1 Of 3  

    Well, I believe that that covers virtually all that I had to talk about and bring up.

QUADRENNIAL DEFENSE REVIEW

    I guess one further question would be you had indicated in your Quadrennial Defense Review you intend to beef up MILCON in the out-years. Has that been quantified? Do you know; are there figures? Or is that simply an intent at this time?

    Mr. LYNN. It is both. It is quantified. The Future Years Defense Plan lags some of the other budget preparation material a little bit. You will be getting that in the next few weeks and you will see for yourselves.

    In that you will see that the family housing program goes up and the military construction, taking the BRAC costs out as separate entity, also will go up. And that is what is in the current program.

    Now, I don't want to stop there though, because the key point and where the rubber meets the road is when you take that outyear program and you craft it into a budget that is submitted to the Hill.

    And I think the key test and the one that you should hold us accountable on is how much of that outyear programmed increase is actually held in as we present the budget to you next year and in the following years.
 Page 62       PREV PAGE       TOP OF DOC    Segment 1 Of 3  

LINE ITEM VETO

    Mr. PACKARD. Let me conclude the hearing with picking back up on the line item veto that we have been through. We would hope that what took us to the point where we had to make the effort to override would be resolved. In other words, it should never have gotten to that point. We would hope—you see, we have been set up for that. This committee has been set up for that because you come in with a low budget request. We automatically strengthen that. We are expected to do that in some areas and we do. Then that sets us up for a line item veto, and that is what happened this last time.

    We scrubbed those projects very carefully and they were good projects and I am not going to go through that, but the President didn't do his homework adequately on many of those projects. Thus, the line item veto took place and that is where we got the bipartisan momentum to override.

    We don't like to do that. That is a bad position for us to be in. I don't relish the need to even challenge the President on a line item veto, but we had to this time, we felt.

    But even if the line item veto now is not constitutional, and that could be challenged but we don't know, it still puts us in a position that I don't like to be in, and that is where the President has submitted what I consider a low budget; we are going to strengthen that with add-ons. It is just going to be an automatic part of our duty and our effort. Then we run the risk of having the President veto it. In this case it was line item veto but the next time, if he doesn't have the line item veto, he may feel inclined to actually veto our bill.
 Page 63       PREV PAGE       TOP OF DOC    Segment 1 Of 3  

    I really want to emphasize that the President's people, and really that is you guys, ought to come to us and sit down and say, ''Look, we have different views on this. Where do we go? What can we do?'' rather than putting us in a position to either override an outright veto of the entire bill or, if something happens to the line item veto authority but it still is challenged and delayed, and we are going to get into that this year or next year, that we don't have to override a line item veto, either.

    Communication, sitting down and working and so forth because the very issues that the President stated publicly for his justification of lining out the 38 projects are the very issues that we concentrated on on each project—making sure it was executable, making sure that it addressed quality of life issues. All of the things that he listed that we didn't do, that gave him justification for lining out, are the very areas that we really concentrated on doing our duty and our job on.

    So he simply did not consult with us. He didn't consult with the DOD adequately and we hope you will use your influence to help us avoid those kinds of conflicts. We don't want to be in those kinds of conflicts. We are willing to work but, at the same time, we are not going to allow the President's authority to veto or line item veto without us doing what we have to do.

    So with that, I am going to conclude the hearing, unless you have some further comments.

    Mr. LYNN. No. Let me just say I agree with the Chairman that we need to improve the communication. Last year was the first year of the line item veto and this was the first bill, and I think it suffered from that. I think we can improve.
 Page 64       PREV PAGE       TOP OF DOC    Segment 1 Of 3  

    Mr. PACKARD. I think you already did. I think on the defense budget you made significant changes in the way you approached the line item veto and, frankly, all the rest of the appropriations bills.

    So we were not harsh on the President. It is just simply that we weren't going to allow mistakes to be made with this kind of authority. So we felt that we were gentle with the President and, at the same time, sending a message back that this great power that we had entrusted to him had to be used properly. And I think that because it was a bipartisan effort, I think it is a message the President probably heard on the other bills.

    At any rate, we want to work with the Administration, not in opposition. Frankly, that is my style and I want to emphasize that.

    Mr. PACKARD. With that, I will close the hearing.

    [CLERK'S NOTE.—Questions for the record submitted by Chairman Packard:]

Overall Funding

    Question. The budget proposes a reduction of $1.4 billion, or 15 percent. This is on top of a $2 billion reduction in the past two years. We continue to see a trend of decreased funding, with resources taken away from badly needed facility and infrastructure improvements. What do you attribute this to, and what can be done about the lack of support and funding requested by the Department?
 Page 65       PREV PAGE       TOP OF DOC    Segment 1 Of 3  

    Answer. While it may appear that military construction is taking a disproportionate share of the reduction between FY 1998 and FY 1999, in reality the FY 1999 Military Construction program is returning to planned program levels. The FY 1999 program reflects a level of funding that is affordable, satisfies the highest priority requirements, and is relatively constant with previous levels, less congressional adds which account for $800 million of the reduction. In addition, the requirements for Base Realignment and Closure (BRAC) have declined due to the completion of BRAC II at the end of FY 1997, and reduced requirements for BRAC III, primarily in the construction area, which accounts for another $327 million of the reduction. The balance is primarily attributable to a $276 million decrease in family housing construction of which over $200 million is related to congressional adds in FY 1998.

Advance Appropriations

    Question. The request includes advance appropriations of $569,000,000. Why are we seeing this change in policy?

    Answer. This change in policy will enable us to manage our resources more effectively by not budgeting resources in advance of need.

    Question. What criteria were applied in determining which projects would receive advance appropriations?

    Answer. The nine projects, eight Army and one Navy, were selected based on the length of the construction schedules (more than twenty-six months) and the Service's estimate of the amount of construction the could be executed in each fiscal year.
 Page 66       PREV PAGE       TOP OF DOC    Segment 1 Of 3  

    Question. What benefit is expected from advance appropriations?

    Answer. The Department hopes to make more efficient use of the available funding and of existing authorities by not budgeting funding for construction that cannot be executed in the budget year. For these nine projects, the Department has requested full authorization up front as well as advance appropriations in subsequent fiscal years for the completion of the construction.

Line Item Veto

    Question. What efforts will the Department take to correct the inaccuracies which resulted in the line item veto of $287 million from our bill?

    Answer. The Department directed the Components to place any vetoed projects back in the program year in which it was originally identified. For example, if a project was programmed in FY 2001 and it was accelerated into the FY 1998 program, but was vetoed, the project should be placed back into the Components' FY 2001 program.

    Question. We anticipate the Senate will vote to override for last week of February. Can you ensure the Committee that, if the veto is overridden, the appropriations for these projects will be allocated to the various Components for execution?

    Answer. Yes, if the veto is overridden, I can assure you that the full amount associated with these projects would be provided to the Components for execution.
 Page 67       PREV PAGE       TOP OF DOC    Segment 1 Of 3  

Facility Strategic Plan

    Question. Explain the Department's efforts to produce a Facility Strategic Plan by 1999.

    Answer. The Office of the Deputy Under Secretary of Defense (Industrial Affairs and Installations) has sponsored a cross-department working group with the purpose of defining and developing a Defense Facilities Strategic Plan. The working group has created a framework, which is pending formal approval, that includes vision, mission, goals, strategies to achieve goals, tools to implement strategies, and metrics to gauge performance. After formal approval, we implement the framework by refining existing strategies, tools, and metrics and creating new ones where needed. We will complete work on these strategies by the end of Calendar Year 98, in time for their incorporation into the Defense Planning Guidance for the 2001–2005 programs.

Facility Reductions

    Question. What effort is DOD making to dispose of surplus real property?

    Answer. As part of the Defense Reform Initiative, each military Service surveyed its building inventory to identify the obsolete and excess buildings they could demolish or otherwise dispose of. The survey identified 80 million square feet and about 8,000 specific structures. If necessary, the Services increased funding in the Fiscal Year 1999 budget for demolition so they could eliminate these structures by the end of fiscal year 2003.
 Page 68       PREV PAGE       TOP OF DOC    Segment 1 Of 3  

Base Realignment and Closure

    Question. Why does the Department need authority in fiscal year 1999 for rounds of base realignment and closure that would not occur until 2001 and 2005?

    Answer. This summer the Department will make program decisions for FY 2000 through FY 2005. Over the next 3 years, the Department will make important decisions regarding the procurement of many systems critical to our future military capabilities. Receiving authorization now for new BRAC rounds in 2001 and 2005 allows the Department to plan for a smaller base structure so that resources that would otherwise be wasted on excess infrastructure can be devoted to modernization and readiness. We also need additional time to plan for cross-Service and cross-function issues largely unaddressed in past BRAC rounds that could contribute significant savings in future rounds.

    Question. Is it correct that the Future Years Defense Program has ''place-holders'' for FY 02 and FY 03 funding? If so, how much is set aside in each year, and how would administrative expenses be financed prior to FY 02?

    Answer. Yes, the Future Years Defense Program contains funding for additional rounds of base closures. We have set aside $830.0 million in FY 2002 and $1,447.0 million in FY 2003 to initiate an additional round in 2001. Funding for a second additional round for 2005 will be programmed in 2006. All administrative expenses that are not directly related to closing or realigning a base will be financed from the Department's mission funding in the operation and maintenance account.
 Page 69       PREV PAGE       TOP OF DOC    Segment 1 Of 3  

Family Housing Construction Reduction

    Question. For the past two years we have seen funding levels for family housing construction and improvement reduced from enacted levels by 25 percent and 31 percent. Again, this year these accounts are reduced by $221 million, or 32 percent. Explain the repeated decline in this request in light of the importance of this issue.

    Answer. Family Housing remains one of our top quality of life issues; however we are never going to have all the resources we think we need to meet all of the Department's priorities. The decreases reflect the Services' prioritization of programs within available resources. We hope that the privatization authorities will enable us to leverage our housing funds and that growing savings from the Quadrennial Defense Review will allow the Services to devote more resources to Family Housing.

Family Housing Improvement Fund

    Question. The Department is placing an emphasis on family housing privatization, yet only requesting administrative expenses ($7 million) for the Family Housing Improvement Fund. Instead, the Department will rely on transfer authority. Please explain this rationale, and what assurances can you give us that the normal construction programs are not being slowed down due to this form of funding?

    Answer. In most cases, consistent with the design of the initiative, the Services will transfer appropriations for housing construction projects into the Family Housing Improvement Fund (FHIF) to finance privatization projects that leverage these traditional construction funds into more housing more quickly. For projects with high leverage potential without a transfer source, the remaining $20 million from the FY 1996 and FY 1997 FHIF appropriations will be used to fund the government's contribution. Each year, the Department's requested Family Housing projects are for locations with the highest priority housing needs. However, we do not know at which locations privatization is a viable alternative to construction. By requesting appropriations for projects in the family housing construction accounts we can proceed quickly with the construction projects if a privatization deal cannot be worked. The transfer authority provides us the flexibility to leverage the funds if the opportunity for privatization exists. While projects may be delayed during the assessment of privatization options, we believe the potential benefits from leveraging are worth the wait. In no case would we delay a project beyond the period for which funds are made available.
 Page 70       PREV PAGE       TOP OF DOC    Segment 1 Of 3  

    Question. Family housing construction requests are down. And, the Department intends on privatizing 30,000 family housing units by the year 2000. Is this program becoming a substitute for the traditional housing program versus a supplement as it was originally intended?

    Answer. No, privatization is only one of the tools we are using to improve our inventory of approximately 200,000 inadequate family housing units. We expect to privatize where it makes sense and continue to use traditional military construction where it doesn't. The Military Departments are currently preparing base by base plans to meet the Department's goal of eliminating this inadequate housing inventory by 2010 using the combination of military construction, privatization and demolition.

    Question. It appears that we are going to shift the costs of housing from the construction and operation and maintenance accounts to the personnel accounts. Do you anticipate any savings with this shift?

    Answer. First I would point out that the majority of our housing costs are already in the military personnel accounts in the form of housing allowances. The Department's policy is to rely first on local communities to meet our housing needs and two-thirds of our military families live off base and receive housing allowances. Secondly, while our privatization efforts are aimed at leveraging housing funds and not at generating savings, various studies have shown that paying housing allowances is more cost effective than constructing, operating, and maintaining our own housing.

 Page 71       PREV PAGE       TOP OF DOC    Segment 1 Of 3  
Transfer Authority

    Question. The budget proposes a general provision which would allow the transfer of up to $200 million between any accounts in the bill, and this could be accomplished at the determination of the Secretary and upon the approval of OMB. Congress would be given an ''after the fact'' notification. What is the need and rationale behind this request?

    Answer. Under the proposed transfer authority, the reprogramming guidelines established by 10 U.S.C. would still apply. Therefore, the Department would be required to provide congressional notification prior to accomplishing any transfer between accounts. As is the case with all reprogramming requests, the Department does not proceed with the projects until both the House and Senate Appropriations Committees have responded to our request.

    This authority would be useful to the Department in solving urgent, high priority funding problems within our available resources.

CHEMICAL DEMILITARIZATION

    Question. The Army Military Construction account shows an 11% increase. However, after taking into account $125 million for chemical demilitarization, the Army's construction is actually down 8%. Why is the Chemical Demilitarization program being shifted from Defense-wide to the Army account?

    Answer. As part of the Quadrennial Defense Review, Secretary Cohen directed his staff to identify efficiencies and downsize the Office of the Secretary of Defense staff. An Integrated Product Team (IPT) was established to evaluate the Chemical Demilitarization Program. Several program management alternatives were evaluated by the team that recommended that the funding and oversight of the program needed to be devolved to the Department of the Army, thus streamlining management of the program. This is also consistent with the conclusions of the Defense Reform Initiative Study that the Department Headquarters should be flexible enough to deal with future challenges; the office of the Secretary of Defense should focus on corporate-level tasks; and operational management tasks should be pushed to the lowest appropriate level. The Chemical Demilitarization program will be re-designated as ACAT IA with the Milestone Decision Authority being transferred to the Army Acquisition Executive.
 Page 72       PREV PAGE       TOP OF DOC    Segment 1 Of 3  

    Question. What precautions have been taken to make sure the Army program is not reduced to fund chemical demilitarization?

    Answer. A complete evaluation of the Chemical Demilitarization program was accomplished as part of the Quadrennial Defense Review. Consistent with this review, all known Chemical Demilitarization program requirements were fully funded at the time of the transfer to the Army. In addition, due to future uncertainties and past history of the program, additional funding was programmed in the out-years to cover unanticipated requirements before transferring it to the Army.

ARMY NATIONAL GUARD

    Question. The Army is briefing members of Congress about the integration of active, guard, and reserve forces in the total Army force structure. To what extent is there integration in the planning and budgeting process, especially for infrastructure, as is done in the Air Force?

    Answer. In the installations funding arena, this is a success story. All components have joined in the process on equal footing or parity, with each component determining requirements based on the same methodology; for construction requirements, each component uses a 57-year revitalization cycle plus buyout of the facilities deficit spread over 27 years. We believe the concept of parity is fully embodied in the development process of the Army's installations programs. Each component is represented on each of the main decision making bodies involved in the planning and budget process: Program Evaluation Group, Program Budget Committee, and the Army Resource Board. Nevertheless, the Army has only been able to fund its highest propriety programs—statutory requirements (such as environment and Chem Demil); Barracks; and Strategic Mobility. Beyond this, all components are funded equally, for example, in revitalization military construction (Active—13%, Guard—12%, Reserve—38%).
 Page 73       PREV PAGE       TOP OF DOC    Segment 1 Of 3  

TROOP HOUSING—''1 PLUS 1''

    Question. What progress is the Department making in upgrading barracks for our single members to meet the new ''1+1'' standard.

    Answer. Each of the Services are making substantial progress in achieving the 1+1 barracks standard. The Army will achieve full implementation by 2012, the Navy by 2013, and the Air Force by 2009. The Marine Corps (operating under a waiver to the 1+1 standard) will achieve its two person per room standard by 2025.

    Question. What has the average cost per space been under the ''1 plus 1'' barracks standard?

    Answer. For new 1+1 construction, the average cost per space is approximately $54,000.

Barracks

    Question. Submit for the record a chart that will show, by Service, the timetable for completion of the barracks revitalization effort at the time OSD approved the ''1 plus 1'' barracks standard, as compared with the timetable for completion of this effort as a result of Congressional initiatives to accelerate this effort.

    Answer. The additional appropriations for quality of life programs that Congress has added to the FY 97 and 98 budgets have contributed to the acceleration to the 1+1 barracks standard in the Army and Air Force, as indicated in the table below. These funds have also contributed to improvements in other facilities that enhance service members' quality of life as previously reported to Congress.

Table 2


 Page 74       PREV PAGE       TOP OF DOC    Segment 1 Of 3  

Troop Housing: Deficit

    Questions. By Service, what is the total current troop housing deficit?

    Answer. The following number of single service members are included in the respective Services' deficit count either because there are too few barracks spaces to house these members or the existing barracks spaces need to be replaced or improved:

Table 3


    Question. Will the other Services be developing a ''Master Plan'' similar to the Air Force's in the near future?

    Answer. Yes. The Navy/Marine Corps master plan is near completion and the Army has embarked on a similar effort.

Troop Housing: Inadequacies

    Question. Provide a breakout of how many barracks are considered substandard, inadequate and facilities with central latrines/showers.

    Answer: The following table indicates the number of bed spaces in barracks considered substandard and the number with central latrines/showers:

Table 4



 Page 75       PREV PAGE       TOP OF DOC    Segment 1 Of 3  
Troop Housing: Budget Request

    Question. How many spaces are included in the budget request for troop housing?

    Answer. The FY 1999 Military Construction request will enable us to construct or modernize 33 barracks and 8,339 living spaces, with the majority of the new or modernized living units at the ''1+1'' standard, which provides each unaccompanied service member more space and a private sleeping area.

    Question. What would it cost to buy-out the current troop housing deficit?

    Answer. To buy out the current housing deficit, the Army would need $5.1 billion, the Navy $1.5 billion, the Air Force $1.3 billion and the Marine Corps would require $0.775 billion.

    The Army's figure uses an estimate of $86,000 per space to eliminate deficits. This figure includes other buildings which are incorporated into its Whole Barracks Complex Renewal projects such as brigade headquarters, dining facilities, etc.

MUHIF

    Question. The Program and Financing statement for the Military Unaccompanied Housing Improvement Fund shows that the full $5,000,000 available in this account will be obligated during fiscal year 1998, and that $1,000,000 will outlay during fiscal year 1998 and another $1,000,000 will outlay during fiscal year 1999. What is the basis for this projection?
 Page 76       PREV PAGE       TOP OF DOC    Segment 1 Of 3  

    Answer. The budget projected an obligation and outlay plan for the FYs 1998 and 1999 Defense Unaccompanied Housing Investment appropriation based on an assessment of anticipated program plans supporting the privatization of barracks. The execution of those plans is dependent upon stimulating private developers to build, operate and maintain barracks. The extent to which market incentives materialize or do not materialize for private development will determine the execution of these funds during the FY 1998 and FY 1999 timeframe.

Child Development Centers

    Question. The budget request seeks funding for five child development centers. Will this enable all services to meet the goal of providing for 80% of the total child care need? If not, how much further construction by component is necessary?

    Answer. We are not relying on construction alone to reach our goal. Currently we meet 57 percent of the need through a system composed of child development centers, family child care homes, and school-age care programs. Over the next 5 years (1999–03) we hope to construct 22 child development centers and have an additional 9 centers planned beyond 2003. We are also planning to establish off-base family child care homes, contract for spaces in accredited civilian child care centers and form partnerships with local elementary schools to expand school-age care.

    Question. Are all of the components programming to meet the goal of 80% of the potential need?

 Page 77       PREV PAGE       TOP OF DOC    Segment 1 Of 3  
    Answer. Our Military Service components are programmed to meet 65 percent of the need. The Army met the 65 percent goal this year. The Marine Corps expects to reach the goal by 2002, and the Air Force and Navy are programmed to be at 65 percent by 2003. The 80 percent goal is a long-range goal we believe will optimally meet our Departmental need.

    Question. What progress has been made in the pilot program to review ways for providing child care services by using third party contracting?

    Answer. The Department of the Navy and the Defense Logistics Agency (DLA) are serving as the DoD executive agents to study third party contracting. The Navy is purchasing spaces in accredited child development centers by buying down the cost for military families. The Navy has awarded contracts in Jacksonville, Florida; Norfolk, Virginia; San Diego, California; and Pearl Harbor, Hawaii. The much-needed and more costly infant and toddler spaces have been difficult to find in the civilian sector. DLA is testing the management and operation of a military-constructed child development center by a private contractor in Dayton, Ohio. The contract has been in effect a little over a year.

UNOBLIGATED APPROPRIATIONS

    Question. The fiscal year 1999 budget request proposes to finance a total of $15,546,000 from unobligated prior year appropriations, as follows:

Table 5


    Identify the sources for this savings.

    Answer. The sources for these savings are the unobligated balances from awarded FY 1995 family housing new construction projects, less any outstanding commitments, as follows:
 Page 78       PREV PAGE       TOP OF DOC    Segment 1 Of 3  

Table 6


    Question. What is the average age of facilities and family housing?

    Answer. The average age for all facilities including family housing, weighted by plant replacement value, is estimated to be about 41 years as of fiscal year 1998.

REPLACEMENT VALUE

    Question. What is the current housing replacement value for the Department of Defense?

    Answer. Based on a Department-wide average cost of $135,000 for construction of a new family housing unit, it would cost about $42 billion to replace the 313,000 houses owned by DoD.

FAMILY HOUSING: DEFICIT

    Question. What is the current total family housing deficit for the Department of Defense, both in units and in cost of replacement, repair, or improvements?

    Answer. The following table represents the Department's estimate of family housing deficits for new construction, replacement and improvements:

Table 7



 Page 79       PREV PAGE       TOP OF DOC    Segment 1 Of 3  
Family Housing: Average Age of Units

    Question. What is the average age of on-base housing?

    Answer. The Department's average age of on-base housing units is 36 years.

Family Housing: Annual Maintenance and Repair

    Question. What is the annual maintenance and repair bill on average for each unit?

    Answer. The following chart depicts the actual maintenance and repair cost per unit in FY 1997 and the estimated maintenance and repair cost per unit for FY 1998 and FY 1999.

Table 8



Family Housing Maintenance Costs

    Question. Provide for the record a chart which will show a five year history, by Service, of the annual inventory of family housing units, the annual maintenance amount, and the annual maintenance cost per unit.

    Answer. The average annual inventory, maintenance and unit costs for each Service for the last five years are:

Table 9



 Page 80       PREV PAGE       TOP OF DOC    Segment 1 Of 3  
Family Housing Improvement Fund

    Question. The budget includes $7,000,000 for the Family Housing Improvement Fund. For the record, please provide an Object and sub-object Classification listing of expected obligations of this sum.

    Answer. Expected obligations are as follows:

Table 10


Housing Allowances

    Question. Submit for the record a chart that will show, by appropriations account, the amount expended during fiscal year 1997 and the amounts budgeted for fiscal years 1998 and 1999 for housing allowances, separately identifying accompanied and unaccompanied allowances.

    Answer. Requested information is provided in attached chart.
    "The Official Committee record contains additional material here."

Inflation

    Question. What inflation rate was used in formulating the budget request?

    Answer. The FY 1999 budget request for military construction and family housing was formulated using an inflation rate of 1.6 percent, which is a composite rate at the title level.
 Page 81       PREV PAGE       TOP OF DOC    Segment 1 Of 3  

Non-Appropriated Funds

    Question. Provide for the record the estimated costs (by State, Service and project) of all nonappropriated funded construction over $500,000 in fiscal years 1998 and 1999. Also include the lump sum total of all projects between $200,000 and $500,000?

    Answer. The Department of Defense Fiscal Year 1998 Commissary Surcharge and Nonappropriated Fund Construction Program is attached. The estimate total reflects design, equipment, and construction costs.

    The Fiscal year 1998 nonappropriated fund minor construction program is currently estimated at $23 million.

    The FY 1999 Minor and Major Commissary Surcharge and Nonappropriated Fund Construction Program is still being developed and will be provided to Congress in July 1998.
    "The Official Committee record contains additional material here."

PLANNING AND DESIGN—NAVY

    Question. Provide for the record a detailed project listing by Service of all projects included in the fiscal year 1999 planning and design request. The listing should include project scope, estimated cost, and estimated design cost.

    Answer. Attached is a list of projects that are included in the FY 1999 planning and design request.
 Page 82       PREV PAGE       TOP OF DOC    Segment 1 Of 3  
    "The Official Committee record contains additional material here."

Planning and Design—Air Force

    Question. Provide for the record a detailed project listing by Service of all projects included in the fiscal year 1999 planning and design request. The listing should include project scope, estimated cost and estimated design cost.

    Answer. The fiscal year 1999 planning and design request is based primarily on the attached list of projects. Additionally, the request includes funds for value engineering and host-nation support programs. The project list will change during the review process in preparation for submission of the fiscal year 2000 President's Budget.
    "The Official Committee record contains additional material here."

Foreign Currency Exchange Rates

    Question. Provide for the record the exchange rates assumed in the fiscal year 1999 budget request.

    Answer. The following foreign currency rates were used in pricing the fiscal year 1999 budget:

Table 11



    Question. What is the current balance in the Foreign Currency Fluctuation Account?
 Page 83       PREV PAGE       TOP OF DOC    Segment 1 Of 3  

    Answer. There are no funds currently in the Foreign Currency Fluctuation, Construction, Account. All funds have been transferred to the Components' Centrally Managed Allotments to meet FY 1998 foreign currency fluctuation requirements. However, the amounts currently held in the Service CMAs are as follows:

Table 12


    Question. How much additional cost has been incurred as a result of foreign currency fluctuation over the last three years, in both construction and family housing operations and maintenance?

    Answer. The following table reflects the additional costs associated with foreign currency fluctuation during the last three years ($ in millions) based on accounting reports for the period ending December 31, 1997:

Table 13



    Question. How much has been transferred into the Foreign Currency Fluctuation Account at the end of the last five fiscal years?

    Answer. The following amounts were transferred into the Foreign Currency Fluctuation Account at the end of the fiscal year:

Table 14


Lost Design

    Question. How much in lost design occurred in fiscal year 1997 for each service?
 Page 84       PREV PAGE       TOP OF DOC    Segment 1 Of 3  

    Answer. The following is the FY 1997 lost design (i.e., the cost of scrapped design or the cost of design that must be redone) by service:

Table 15


Breakage and Design

    Question. How much breakage and deferred design occurred in fiscal year 1997 for each service?

    Answer. The following is the FY 1997 breakage and deferred design (i.e., design obligations against projects that will not be acquired) by service:

Table 16


Davis-Bacon: Costs

    Question. What is your estimate of the amount within the budget request for fiscal year 1999 that is attributable to the provisions of Davis-Bacon?

    Answer. The Department conservatively estimates that 5 to 8 percent of project costs are attributable to Davis-Bacon provisions. Amounts attributable to Davis-Bacon provisions for Military Construction are $103 million at 5 percent and $165 million at 8 percent. Amounts for Family Housing are $25 million and $40 million respectively.

Phase Funding

    Question. Does the budget request include funding for any projects at a level that exceeds the amount of construction that can be put in place during fiscal year 1999?
 Page 85       PREV PAGE       TOP OF DOC    Segment 1 Of 3  

    Answer. No. All the funds we requested for FY99 are for construction we can obligate in FY99.

DOD Environmental Project List

    Question. Provide for the record a list of environmental compliance projects requested in the budget, sorted by service, installation, and by level of compliance.

    Answer. Attached is a list of environmental compliance projects by service and installation. All of the projects listed are Class I projects.
    "The Official Committee record contains additional material here."

Environmental Levels

    Question. For the record what is the definition of ''Level 1'', ''Level II'', and ''Level III'' projects?

    Answer. To meet DoD's Environmental Security goals, the DoD Components fund all requirements based on the Environmental Quality Classes defined consistent with timely execution to meet future deadlines. Determination of classes at overseas locations will be based on the Final Governing Standards. DoD policy is to fund all Class I projects, which are projects intended to address activities that are frequently out of compliance (whether they have received an NOV or not) and those activities that will be out of compliance if not funded in the fiscal year requested. DoD policy also requires the Military Departments to budget prudently to fund Class II projects to meet future deadlines. Class II projects are those projects where a deadline exists, but where project completion could possibly be delayed and the Defense activity still not be in violation. The best example of Class II projects is Underground Storage Tanks (USTs).
 Page 86       PREV PAGE       TOP OF DOC    Segment 1 Of 3  

    Class I—Projects and activities needed that are currently out of compliance (have received an enforcement action from a duly authorized Federal agency, state, or local authority; have signed a compliance agreement or received a consent order; and/or have not met requirements based on applicable compliance requirements). This class also includes projects and activities needed that are not currently out of compliance (deadlines or requirements have been established by applicable requirements, but deadlines have not passed or requirements are not in force) but will be if projects or activities are not implemented within the current program year. These activities include the preparation of plans (e.g., NEPA documentation, master plans, emergency response plans, integrated natural and cultural resource management plans, pollution prevention plans, etc.), opportunity assessments and inventories. The preferred approach is to use pollution prevention projects or activities, if cost effective, as the means of bringing a facility into compliance. This class includes overseas projects and activities necessary to alleviate the human health threats, threats to ongoing operations, or necessary to comply with applicable treaties and agreements.

    Class II—Projects and activities needed that are not presently out of compliance (deadlines or requirements have been established by applicable compliance requirements, but deadlines have not passed or requirements are not in force) but will be if projects or activities are not implemented in time to meet an established deadline beyond the current year. This class should also include requirements that demonstrate a three year or less return or investment or that significantly reduce environmental life cycle costs for facilities, installations, and deployed weapons. Overseas this class includes projects and activities identified using risk based prioritization practices that meet the long term objective of full implementation by FY03 of the Final Governing Standards for each foreign country where DoD maintains substantial installations.
 Page 87       PREV PAGE       TOP OF DOC    Segment 1 Of 3  

    Class III—Includes projects and activities that are not explicitly required by law but are needed to address overall environmental goals and objectives.

DOD Life Safety/Health Project List

    Question. Provide for the record a list of life safety/health compliance projects requested in the budget, sorted by service, installation, and by level of compliance.

    Answer. DoD Component hazard abatement projects are prioritized for funding using the DoD Risk Assessment Code (RAC) system instead of the Environmental Quality classes defined in the answer to Question Number 52. Each installation is required to maintain a formal hazard abatement plan listing projects with RAC funding priorities (RAC 1—critical, RAC 2—serious, RAC 3—moderate risk). However, there is no institutionalized DoD requirement for the Components to report on the status of these installation level projects. Nor is there a specific line item in the DoD budget identifying all hazard abatement projects. Although some projects may be funded by MILCON, many are funded in the O&M accounts or in the Defense Health Program, without any visible breakouts in the budget.

    Commencing this year, however, the Deputy Under Secretary of Defense (Environmental Security) asked the Military Services and DLA to report the status of funding for their hazard abatement programs during the first DoD Safety In Progress Reviews (IPR). The Air Force and Navy identified RAC 1, 2 and 3 project requirements totaling over $524M, with active plans to fund about $120M in the FY 1999 budget. The remaining components need additional time to identify their requirements. We anticipate better reporting in the status of hazard abatement projects in successive IPRs. The Department plans to formalize the collection of this data through a change to DoDI 6055.1, ''DoD Occupational Safety and Health Program.
 Page 88       PREV PAGE       TOP OF DOC    Segment 1 Of 3  

Planning & Design—Army

    Question. Provide for the record a detailed project listing by Service of all projects included in the fiscal year 1999 planning and design request. The listing should include project scope, estimated cost and estimated design cost.

    Answer. The detailed project listing follows:
    "The Official Committee record contains additional material here."

Unspecified Minor Construction

    Question. Provide for the record a chart which will show the budget request for unspecified minor construction by component, compared to the enacted fiscal year 1998 level.

    Answer. The following table identifies the FY 1999 request and the FY 1998 enacted level by component for unspecified minor construction.

Table 17



Execution Rates

    Question. What is the execution rate of each component for the past three years for both the military construction and family housing?

 Page 89       PREV PAGE       TOP OF DOC    Segment 1 Of 3  
    Answer. The execution rate for the components military construction in the first year of the appropriation is as follows:

Table 18



    Answer. The execution rate for the components family housing in the first year of the appropriation is as follows:

Table 19



    NOTE.—The Family Housing, Defense-Wide program had only two projects in FY 1997, one each for NSA and DLA. DLA did not execute the project in FY 1997 due to higher than expected bid offers, and a reprogramming request was required. The reprogramming request was approved by the HAC on February 4, 1998, and the SAC on February 6, 1998.
    "The Official Committee record contains additional material here."

Unspecified Minor Construction—Army

    Question. Has this appropriation met the needs of the components over the last two years. What shortfalls, if any, have been encountered?

    Answer. Over the last two years, the Unspecified Minor Military Construction, Army, appropriation has been used to meet the Army's needs. However, the Army Major Commands have recently been identifying an increasing number of candidate projects for funding.

Unspecified Minor Construction—Navy

 Page 90       PREV PAGE       TOP OF DOC    Segment 1 Of 3  
    Question. Has this (Unspecified Minor Construction) appropriation met the needs of the components over the last two years? What shortfalls if ever have been encountered?

    Answer. In Fiscal Year 1996, the Navy reprogrammed an additional $2.85 million into the Fiscal Year 1995 and Fiscal Year 1996 UMC account to provide the necessary funding to award three additional urgent Unspecified Minor Construction projects. The Fiscal Year 1997 appropriation was adequate to meet requirements.

Unspecified Minor Construction—Air Force

    Question. Has this appropriation met the needs of the components over the last two years? What shortfalls, if any, have been encountered?

    Answer. The appropriation has not been sufficient to meet our most urgent needs. Shortfalls have been accommodated through reprogramming actions in three of the last four years: FY94 reprogrammed $1.711M; FY95 reprogrammed $1.45M; FY96 no reprogramming to date; and FY97 reprogrammed $0.8M.

Unspecified Minor Construction—Air National Guard

    Question. Has this appropriation met the needs of the components over the last two years?

    Answer. Over the last two years, the appropriation was sufficient to meet the Air National Guard's (ANG) most urgent needs. Given the number of short-notice mission conversions requiring facility projects, the ANG historically has had to carefully balance these urgent requirements against the limited availability of funds.
 Page 91       PREV PAGE       TOP OF DOC    Segment 1 Of 3  

    Question. What shortfalls, if any, have been encountered?

    Answer. The Air Force continues to transfer numerous missions into the ANG. Since inclusion in the military construction program is often late-to-need, unspecified minor construction is used to satisfy the most urgent facility needs. Despite the additional appropriations received in fiscal year (FY) 1998, the following $4.66 million is requirements will have to be deferred to FY 1999 and compete with other requirements already anticipated to be more than the budget request:

Table 20



Unspecified Minor Construction—Air Force Reserve

    Question. Has This appropriation met the needs of the components over the last two years? What shortfalls, if any, have been encountered?

    Answer. The unspecified minor construction appropriation has met the needs of the Air Force Reserve over the last two years. However, we foresee a shortfall in FY 1999 due to the following emergency requirements

Table 21


    The urgency of these projects is based upon late notification of simulator procurement at Kelly and Wright-Patterson AFBs, and the requirement to remove temporary facilities (i.e., trailers) at Portland ANGB.

    The following urgent requirements were schedules for FY 1999 but have slipped to FY 2000 due to the more urgent requirements listed above:

Table 22


 Page 92       PREV PAGE       TOP OF DOC    Segment 1 Of 3  
    Therefore, since the Air Force Reserve's total FY 1999 unspecified minor construction requirement is $7,394K (i.e., $2,850K + $4,544K), and our budget request is limited to $2,903K, our shortfall in FY 1999 unspecified minor construction totals $4,491K (i.e., $7,394K—$2,903K).

Planning and Design

    Question. Does the budget request include sufficient planning and design funds to execute the entire fiscal year 1999 program?

    Answer. The Department's estimate for planning and design (P&D) requirements for the budget year is based on the size of the two succeeding fiscal year military construction programs. For example, the amount requested for FY 1999 will be used to complete design on FY 2000 projects and to initiate design on the FY 2001 projects. Therefore, the size of the FY 1999 request is a function of the size of the FY 2000 and FY 2001 construction programs. Consequently, the $200.4 million requested in FY 1999 for planning and design is sufficient to complete design on the FY 2000 military construction program, which will allow for execution in FY 2000, and bring the FY 2001 program to the required 35% design level.

    The $217.6 million enacted in FY 1998 for planning and design is sufficient to complete design on the FY 1999 military construction program, which will allow for execution in FY 1999.

    Question. Are any projects in the budget request at less than 35 percent design? If so, list an justify such projects.
 Page 93       PREV PAGE       TOP OF DOC    Segment 1 Of 3  

    Answer. All of the projects in the FY 1999 request are at the required 35 percent design level which will allow for execution in FY 1999.

Real Property Maintenance—Army

    Question. What is the total fiscal year 1999 budget request for real property maintenance (RPM)?

    Answer. The total fiscal year 1999 budget request for real property maintenance is $1,298 Million, which includes Panama Canal Treaty.

    Question. How much is earmarked for barracks renovation?

    Answer. The Barracks Upgrade Program earmarked $148 Million in FY 1999. (RPM funding only)

Real Property Maintenance—Navy

    Question. What is the fiscal year 1999 budget request for real property maintenance? How much is earmarked for barracks renovation?

    Answer:

    The Navy's total fiscal year 1999 budget request for real property maintenance is $918 million ($877 million active and $41 million reserve), of which $159 million ($156 million active and $3 million reserve) is earmarked for barracks.
 Page 94       PREV PAGE       TOP OF DOC    Segment 1 Of 3  

    The Marine Corps total fiscal year 1999 budget request for real property maintenance is $345 million ($388 million active, $7 million reserve), of which $75 million (all active) is earmarked for barracks.

Real Property Maintenance—Air Force

    Question. What is the total fiscal year 1999 budget request real property maintenance? How much is earmarked for barrack renovation?

    Answer. The Air Force real property maintenance budget request for fiscal year 1999 is $1,414 (Total Force). The budget does not include funding to support dormitory renovations. The 1999 budget continues to fund real property maintenance at the Preventive Maintenance Level which only provides resources to accomplish the periodic maintenance required to sustain day-to-day real property facilities and infrastructure. This funding level limits efforts for maintenance and defers critical repair projects, including dormitory renovations. If funding were made available, the Air Force could execute $160M in quality-of-life projects in fiscal year 1999, which includes $91M in dormitory repair and minor construction projects.

Reprogramming

    Question. Submit for the record a table which will show, by fiscal year, the total amount approved by Congress for reprogramming for ''Military Construction, Defense-Wide'' for the last five years.

 Page 95       PREV PAGE       TOP OF DOC    Segment 1 Of 3  
    Answer:

Table 23



Line Item Veto

    Question. Provide for the record a description of how the amounts line item vetoed from Public Law 105–45 are displayed in the 1998 column of the Program and Financing statements for the seven affected accounts.

    Answer. The Office of Management and Budget directed that reductions pursuant to the line item veto displayed on line 40.79, ''Line item veto cancellation'', in the Program and Financing schedule for the applicable accounts. The amounts on these lines are reflected as a minus, indicating a reduction in appropriation.

    Question. Is it correct that the following two projects included in the budget request for fiscal year 1999 are no longer required, if H.R. 2631 becomes law?

Table 24


    Answer. Yes, that is correct. The FY 1999 President's budget was prepared on the assumption that the line item veto would not be overturned. Therefore, if H.R. 2631 does become law, those two projects would not need to be funded in FY 1999.

Revised Economic Assumptions

    Question. Provide for the record a description of how the amounts appropriated by account in Public Law 105–45 and subsequently reduced by section 125 [due to revised economic assumptions] are displayed in the 1998 column of the Program and Financing statements for the eleven affected accounts.
 Page 96       PREV PAGE       TOP OF DOC    Segment 1 Of 3  

    Answer. Reductions pursuant to section 125 of Public Law 105–45 were account specific and are included in the amount on line 40.00, Appropriation, in the Program and Financing schedule. The direct program amounts in the ''Obligations by program activity'' portion of the Program and Financing schedule were reduced accordingly.

Rossmoor Liquidating Trust Settlement Account

    Question. Provide for the record the plan for work to be accomplished with resources available under the Rossmoor Liquidating Trust Settlement Account?

    Answer. The Department of the Navy plans to purchase available housing units in the San Diego area using the Rossmoor Liquidating Trust Settlement funds. NAVFAC Southwest Division is currently negotiating with contractor and plans should be finalized by early April 1998. The Congressional notification, required by section 2208 of Public Law 104–106, will be provided prior to the obligation of those funds.

General Provisions—Section 101

    Question. Why does the budget propose to delete the word ''work'' and substitute the word ''construction''? Also, in our action on the fiscal year 1996 bill we inserted the proviso regarding BRAC environmental restoration at the request of the Department. Why does the budget propose to delete this proviso?

    Answer. Section 101, which was enacted in 1951, is intended to prevent the use of cost-plus-fixed fee contracts for construction; however, because it does not say that it applies to ''construction'' but to ''work'', it has been viewed as applying to any work being accomplished by contract, including work which is not construction.
 Page 97       PREV PAGE       TOP OF DOC    Segment 1 Of 3  

    Because of this, in the fiscal year 1996 Act, it was amended to provide an exception to its provisions in the case of contracts for environmental restoration at bases being closed when payments are being made from a base realignment and closure (BRAC) account. These are not ''construction'' contracts, but they are contracts for ''work'' identified in this section.

    The exception which was added in the fiscal year 1996 Act recognized the fact that cost plus fixed fee contracts are virtually the only ones which are used in the case of environmental restoration. There are other contracts; however, funded from the BRAC accounts, which are not for environmental restoration but involve general operation and maintenance activities which also should be excepted. One of the reasons for this is that a restriction such as that contained in section 101 does not apply to similar operations and maintenance contracts funded with appropriations in the DoD Appropriations Act. Accordingly, in prior years requests have been made that this provision be amended by expanding the BRAC-funded exception to include all contracts ''other than those for construction.''

    In connection with consideration of making the same request for the FY 1999 budget which had been made in prior years, it was recognized that, if the reference to ''work'' was changed to ''construction,'' not only would the section accurately reflect its true intent to apply to construction contracts but also it would eliminate the necessity for the proviso.

    For the foregoing reasons, the budget proposes that the word ''work'' be changed to ''construction'' and the proviso is proposed for elimination because it would be unnecessary if the foregoing language change were made. If the change is not made, however, then the proviso should be retained. Additionally, it should be modified by deleting the words ''for environmental restoration'' and inserting the parenthetical ''(other than contracts for construction)'' in their place.
 Page 98       PREV PAGE       TOP OF DOC    Segment 1 Of 3  

General Provisions

    Question. Why does the budget propose language permitting transfers from the BRAC accounts to the Family Housing Improvement Fund, rather than requesting direct appropriations to the Fund?

    Answer. We do not know at which BRAC locations, if any, privatization is a viable alternative to construction. By requesting appropriations in the BRAC account we can proceed with construction projects if a privatization deal cannot be worked. The transfer authority provides us the flexibility to leverage the funds if the opportunity for privatization exists.

General Provisions—Deletions

    Question. Why does the budget propose to delete the following sections: Section 111—American preference for A & E contracts; section 113—notification of exercises; section 119—annual burdensharing report; section 121—compliance with ''Buy American Act'', and section 122—American-made equipment and products.

    Answer. Sections 111, 113, and 119 are recommended for deletion because they constitute unnecessary and undesirable restrictions on the management of the Department of Defense. Their recommended deletion is consistent with similar recommendations contained in each President's budget request since the time each of them was enacted.

 Page 99       PREV PAGE       TOP OF DOC    Segment 1 Of 3  
    Sections 121 and 122 are proposed for deletion because their retention is unnecessary.

    Following is a more detailed discussion of each of these provisions.

    Section 111—American Preference for A&E Contracts (Japan and NATO Architect and Engineer Contracts).

    Section 111 constitutes a limitation on the Department's use of appropriated funds. It was first enacted in the fiscal year 1985 Appropriations Act and prohibits the Department from obligating funds for architect and engineer services of more than $500,000 for projects to be accomplished in Japan or NATO countries unless United States firms are in joint ventures with host nation firms. It is not cost effective.

    Section 113—Notification of exercise (Military Exercise Construction).

    Section 113 constitutes a limitation on the Department's use of appropriated funds. It was first enacted in the fiscal year 1985 Appropriations Act and requires the Department to notify the Armed Services and Appropriations Committees 30 days in advance of the plans and scope of any military exercise involving United States personnel if construction related to the exercise is anticipated to be more than $100,000. The Department is opposed to limitations on the way it does business and has requested deletion of this provision on that basis each year since 1985.

 Page 100       PREV PAGE       TOP OF DOC    Segment 1 Of 3  
    Section 119—Annual Burdensharing report (NATO, Japan, and Korea Host Nation Support Negotiations).

    Section 119 is a requirement imposed on the Department of Defense that unnecessarily interferes with the way that negotiations for host nation support are conducted. It was first enacted in the fiscal year 1988 Appropriations Act and requires the Secretary of Defense provide an annual report to the Committees on Appropriations of the specific actions proposed to be taken to encourage NATO countries, Japan and Korea to undertake a greater share of the defense burden of the United States and those nations. While the Department is committed to increasing host nation support in all cases, it does not consider that it is necessary or appropriate to include all of the specific details of the Department's negotiations in this area in annual reports to the Congress. Not only could such reports compromise negotiating positions but also they could interfere with the way the Executive branch conducts those negotiations. The Department is, therefore, strongly opposed to this provision.

    Sections 121 and 122—Compliance with ''Buy American Act'' and Statement of the Sense of the Congress regarding American Made Equipment and Products.

    Section 121 prohibits the expenditure of funds provided in the Act ''by an entity unless that entity agrees that in expending the assistance the entity will comply with [the Buy American Act].'' The only entity which expends funds provided by the Act is the Department of Defense which must comply with the Act in any case. Therefore, the section merely requires the Department to comply with a law that it is already bound to follow.

    Section 122, states the Sense of the Congress in subsection (a). Subsection (b) requires the Secretary of the Treasury ''in providing assistance under this Act'' to provide a notice describing the Statement of the Sense of the Congress.
 Page 101       PREV PAGE       TOP OF DOC    Segment 1 Of 3  

    With respect to subsection (a), because it states the Sense of the Congress, once it is stated, it does not need to be restated again. It remains the Sense of the Congress even through the successor Congresses that follow the one in which it is stated. For instance, the so-called ''User Charges'' statute is, in fact, a statement of the Sense of the Congress which was made in 1951. It is still followed today and is codified at section 9701 of title 31, United States Code. This is true even though it has not been restated in a law in the 24 years since the statement was made. Subsection (b) should not be included in the Department's Military Construction Appropriations Act because it applies to assistance provided by the Secretary of the Treasury, not by the Department of Defense. Furthermore, the only assistance provided by the Department of Defense with funds made available in the Military Construction Appropriations Act is assistance under the Homeowner's assistance program and this program does not involve the expenditure of any funds for equipment or products.

General Provisions

NEW SECTION 121

    Question. Why does the budget propose a new section 121, providing transfer authority from the BRAC accounts to the Homeowners Assistance Fund, rather than requesting direct appropriations to the Fund?

    Answer. We are only trying to regain the authority that was granted in the FY 1996 and FY 1997 Military Construction Acts to transfer funds from the Service BRAC accounts to the Homeowners Assistance Program (HAP). The FY 1998 Military Construction Appropriation Act deleted the general provisions this authority. The HAP program operates as a revolving fund, partially sustaining its operations with proceeds generated from the sale of acquired property and augmenting it by direct appropriations. The Department estimates that it will only need $12.8 million in direct appropriations in FY 1999. However, since HAP expenditures are the direct result of base closures or realignments, to cover any unforeseen HAP requirements that may arise during execution, we are seeking this general provision that will authorize the transfer of funds from the BRAC account to the HAP account.
 Page 102       PREV PAGE       TOP OF DOC    Segment 1 Of 3  

NEW SECTION 122

    Question. Why does the budget propose a new section 122, providing transfer authority not to exceed $200,000,000, rather than requesting sufficient direct appropriations to contingency construction?

    Answer. The Department requests approximately $10 million of direct appropriations for contingency construction each year; however, for three of the past four years, Congress has provided less than half of the amount requested and a significant portion of these amounts has been rescinded. Fiscal constraints do not make it prudent to increase the Department's request for contingencies. The proposed transfer authority would allow the Department to provide funding for high priority MILCON requirements with costs exceeding available contingency funds which surface during the budget year and are too urgent to defer for inclusion in the next budget.

General Provision 123—Navy

    Question. Provide for the record a detailed description of the work to be accomplished (under section 123) at Everett, Washington, including cost estimates, OMB scoring, and form 1391 justification. In addition, please provide copies of the cited legislation for the record.

    Answer. The $6 million in proceeds from the sale of the land and family housing units at Paine Field will be used to acquire additional family housing units for Naval Station Everett, as part of the follow-on PPV project at Everett. The project would involve the construction of new homes on private land. The balance of funding necessary to execute this PPV project would be provided via the Fiscal Year 1997 family housing project that provided $15.0 million to construct 100 homes at Everett. The new homes to be constructed will fully satisfy the current and projected housing deficit at Naval Station Everett. Any remaining funds (from the above-referenced Fiscal Year 1997 family housing project) will be used for other PPV efforts in the Pacific Northwest.
 Page 103       PREV PAGE       TOP OF DOC    Segment 1 Of 3  

    Originally, the Navy had planned to use the proceeds from the sale of Paine Field to acquire land in the vicinity of Everett, in order to construct the 100 units of family housing that were authorized and appropriated in Fiscal Year 1997. Using the Paine Field proceeds in conjunction with the Fiscal Year 1997 funds in a single PPV project will provide four and one-half times as many new homes as could have been provided under the Navy's original plan. Presently, the proceeds received from the sale of Paine Field are being held in escrow, and the Navy is leasing back seventy-four units from the purchaser of the property at a cost of approximately $1 million per year.

    The provision ''subject to the availability of appropriations'' in the authorizing legislation for the sale of Paine Field (Section 2842 of Public Law 102–484) is precluding the Navy from using the funds for their intended purpose. To our knowledge, there are only two means to make the funds available:

    Make an amendment to Section 2842 of the Public Law 102–484, deleting the opening phrase ''Subject to the availability of appropriations for this purpose''. We are unaware of any scoring implications associated with this alternative, but we understand that there are procedural rules that may impact upon this alternative.

    Appropriate the funds received by the Secretary of the Navy making the funds available for the purposes authorized in Public Law 102–484. It is our understanding that the appropriation of these funds for these purposes would be subject to scoring.

    Since the intended use of the proceeds is for a PPV project, a DD 1391 is not applicable.
 Page 104       PREV PAGE       TOP OF DOC    Segment 1 Of 3  

Advance Appropriations

    Question. The budget request includes advance appropriations totaling $569,000,000 as follows:

Table 25


    What criteria were applied in determining which projects could be phased?

    Answer. The nine projects, eight Army and on Navy, were selected based on the length of their construction schedules (more than twenty-six months) and the Service's estimate of the amount of construction that could be executed in each fiscal year.

    Question. The President's budget states that requests for such advance appropriations are required by 31 U.S.C. 1105(a)(17). Have these specific advance appropriations been authorized in an enacted authorization bill?

    Is it correct that these specific advance appropriations are not required under 31 U.S.C. 1105(a)(17) until such time as the related authorizations for advance appropriations have been enacted into law?

    Please submit for the record a copy of 31 U.S.C. 1105(a)(17), together with the Department's interpretation of what this citation requires.

    Answer. The Department's requests for advance appropriations are not required by the provisions of 31 U.S.C. 1105(a)(17) which requires such requests only for programs for which there is an existing authorization of appropriations. Rather, for the reasons indicated in the previous question, the Department's request for advance appropriations in the military construction accounts is based on the length of the construction schedules involved and the Service's estimates of the amount of construction that could be executed in each fiscal year.
 Page 105       PREV PAGE       TOP OF DOC    Segment 1 Of 3  

    These specific appropriations have not yet been authorized in an enacted authorization Act. A request for authorization for these advance appropriations will be included in the Department's requests for authorizations for Fiscal Year 1999.

    A copy of the provisions of 31 U.S.C. 1105(a)(17) is attached.
    "The Official Committee record contains additional material here."

    Question. Is it correct that advance appropriations have not been requested in the past for large construction projects such as replacement hospitals and chemical demilitarization facilities? A noteworthy example is the Portsmouth Naval Hospital, which received ''phase IX'' funding to complete construction in fiscal year 1998.

    Answer. Yes, That is correct. With the FY 1999 President's budget we have followed the lead of the congressional committees and attempted to align the funding with the construction schedule.

    Question. Is it correct that the only precedent for advance appropriations in this Bill was for the construction of Fort Drum?

    Answer. Yes, that is correct.

    Question. If it is sound procedure to advance fund known construction requirements, why is it not sound to advance fund known requirements for family housing operations and maintenance?
 Page 106       PREV PAGE       TOP OF DOC    Segment 1 Of 3  

    Answer. The construction projects where we have requested advance appropriations were selected based on the length of their construction schedules (more than twenty-six months) and the Service's estimate of the amount of construction that could be executed in each fiscal year. Family housing operation and maintenance appropriations fund routine on-going day-to-day costs such as housing management, trash and snow removal services, utilities, and maintenance that do not lend themselves to advance appropriations. Some major maintenance and repair projects may extend beyond a single fiscal year, but are not readily severable into distinct pieces that can be funded with advance appropriations.

    Question. What benefit is expected to be realized from advance appropriations, and what is the basis for this expectation?

    Answer. The Department expects to make more efficient use of our available funding by not budgeting for construction that cannot be execute din a timely manner.

Base Realignment and Closure

    Question. Wouldn't time for advance preparation of future BRAC rounds permit the Services to predetermine the installations that would be closed, by deferring maintenance, within threshold transfers of personnel and missions, and so forth?

    Answer. No, only a BRAC analysis can determine which bases are candidates for closure or realignment. The BRAC process, as defined in law, precludes the Department from predetermining bases for closure or realignment. That process requires the Department's conduct of an exhaustive analysis, based upon published selection criteria and a six-year force structure plan, followed by an independent commission's review. The commission's recommendations must be approved or rejected by the President in total and are subject to Congressional disapproval.
 Page 107       PREV PAGE       TOP OF DOC    Segment 1 Of 3  

    Question. For BRAC I and BRAC II, how much time elapsed between authorization and the establishment of those commissions?

    Answer. For BRAC I, the Defense Secretary chartered a Commission in May 1988 followed by authorizing legislation in October 1988 (Public Law 100–526). The authorizing legislation for BRACs II, III and IV was enacted in November 1990 (Public Law 101–510). The Commissioners for BRAC II were confirmed by the Senate in the March/April 1991 (about 5 months after authorization). For BRAC III, the Commissioners were confirmed in March 1993 (28 months after authorization) and for BRAC IV, the Commissioners were confirmed in March 1995 (52 months after authorization).

Environmental Restoration at BRAC Locations

    Question. How does the Department intend to budget for the continuing environmental restoration efforts at BRAC I through IV locations beyond fiscal year 01—will there be a continuing account in this appropriations bill to fund these cost?

    Answer. After FY 2001, the Dpeartment's current plans are to budget for environmental cleanup requirements at closed bases in the Components' Environmental Restoration accounts and Environmental Quality requirements primarily in the Components' Operation and Maintenance accounts.

    Question. If not, why not?

 Page 108       PREV PAGE       TOP OF DOC    Segment 1 Of 3  
    Answer. The provision in the BRAC law making the Base Closure Accounts the exclusive source of funds for environmental restoration at closing or realigning bases will require in FY 2001 upon the termination of the authority of the Secretary of Defense to carry out a closure or realignment. As a consequence, we see no reason to establish and appropriate funds in a separate account for environmental restoration at bases that have closed or realigned when accounts already exist for this purpose.

    Question. Do you have an estimate of the annual level of funding required for this effort for fiscal year 01 and beyond?

    Answer. Yes, we estimate a funding requirement of $686.9 million in FY 2001 and about $500.0 million annually thereafter until completion.

Summary and Reconciliation

    Question. Please submit for the record the two-page chart titled ''Summary and Reconciliation of Authorization, Authorization of Appropriations, and Appropriations Requested from Congress for FY 1999''.

    Answer. The two page summary chart is attached as requested.
     "The Official Committee record contains additional material here."

Y2K/Millenium Bug

    Question. Is there any impact on programs funded in this bill—particularly any budgetary impact—tied to the Department's efforts to address the ''Y2K/Millenium Bug'' computer problems?
 Page 109       PREV PAGE       TOP OF DOC    Segment 1 Of 3  

    Answer. There are no discrete Y2K impacts on projects funded in the FY99 Military Construction Bill. The federal government currently is addressing the Y2K issue as a part of the federal acquisition process for new construction or purchases. The thrust of the solution will be to require manufacturers to certify and warrantee the Y2K compliance of equipment installed in FY99 MilCon projects.

Public/Private Ventures—Administrative Overhead

    Question. How do contracts for public/private ventures—particularly contracts involving family housing—address administrative overhead for federal expenses?

    Answer. Contracts for privatized housing require analysis of life cycle costs comparing privatization alternatives with the government alternative. This analysis includes all government expenses.

    Question. Do such contracts make any provision for reimbursement of federal expenses in any way?

    Answer. Housing privatization requires reimbursement of any services provided by the government as part of the contracts, such as utilities, fire protection and the like. There are no reimbursements for administrative overhead in any projects currently under consideration.

 Page 110       PREV PAGE       TOP OF DOC    Segment 1 Of 3  
    [CLERK'S NOTE.—End of questions for the record submitted by Chairman Packard.]

    [CLERKS NOTE.—Questions for the record submitted by Congressman Porter.]

Impact Aid

    Question. Mr. Secretary, I am extremely disappointed that, once again, the DoD did not include any funding for Impact Aid in its FY99 budget, a program that military families deem critical to ensuring the best possible education for their children. Fortunately, I was able to include $808M in the FY98 Labor—HHS—Education appropriations bill. Currently, Impact Aid is only funded by the DoE. However, I am greatly disturbed by the apparent lack of concern for this program in the DoD, particularly as it is a significant factor in the quality of life for our military personnel. What plans does DoD have for becoming a stronger advocate of this program?

    Answer. Responsibility for funding and managing the Federal impact aid program rests with the Department of Education (DoED). The Department of Defense maintains a close relationship with DoED to help ensure the DoD concerns about the impact aid program are known to DoED. While DoD seeks to ensure that the children of DoD personnel receive the highest quality of education possible, it recognizes the same concern for all children. DoD is concerned regarding the provision of Federal funds to local education agencies (LEA's) to help support the education of Federally-connected children of Department employees and servicemembers. However, DoD believes that the best solution lies in managing impact aid through DoED and appropriating funds sufficient to enable it to provide funding to all eligible LEA's to the fullest extent allowed by law.
 Page 111       PREV PAGE       TOP OF DOC    Segment 1 Of 3  

    Question. As you know, recruitment and retention of military personnel has been a major concern to DoD. It is my understanding that a contributing factor to this problem is that military families feel their children's needs are not valued as a result of Impact Aid shortfalls. The morale of these families is degenerated further by local citizens who feel that military families do not contribute their fair share to the school district. The resentment and hostility created by this situation permeates the community and causes division within the classroom. For this reason, I believe that it would be in DoD's best interest to adequately fund Impact Aid in order to eliminate education as a concern to military families. How does access to educational services affect recruitment, retention, and morale of personnel?

    Answer. The Department maintains a high level of interest in programs that contribute to a high quality of life for servicemembers and their families. Military members with children have the same or higher expectations for the education of their children as do parents in the general population. DoD's interests and the satisfaction of its personnel are best served by funding the Federal impact aid program managed by the Department of Education sufficiently to ensure that all Federally-connected children receive a quality education.

    [CLERK'S NOTE.—End of questions submitted for the record by Congressman Porter.]

    [CLERK'S NOTE.—Questions for the record submitted by Congressman Hobson.]

Guard and Reserve
 Page 112       PREV PAGE       TOP OF DOC    Segment 1 Of 3  

    Question. Your statement says that the FY99 request of $180M for Guard and Reserve is $7M above the amount in FY98. If I remember correctly, the FY98 MilCon Appropriations conference provided approximately $460M for these activities. Why are the Guard and Reserve consistently shortchanged?

    Answer. The Department consistently weighs competing interests within available resources. The Active and Reserve components and priorities consistent with mission requirements. The Department is making progress in funding Milcon for the Guard and reserve as evidenced by this year's increase of $7 million over last budget request. Further, the Guard and Reserve now have representation on the Defense Resources Board; and for the first time, during the FY 1999 program review, the Reserve components have been given unprecedented consideration.

    Question. Is this a sign that we really don't have a total force—that it is all for the active component and little for the Guard and Reserve stepchildren?

    Answer. To ensure that the Department focuses on integration throughout the force, Secretary Cohen issued a memorandum on September 4, 1997, emphasizing increased reliance on Reserve components. In this memorandum, he outlined what needs to be achieved throughout the Department before the Total Force integration can become a reality. The goal is integration of the Reserve and Active components into a seamless Total Force that provides the National Command Authorities the flexibility and interoperability needed for a full range of military operations.

 Page 113       PREV PAGE       TOP OF DOC    Segment 1 Of 3  
NATO

    Question. I understand that any expansion of NATO will require increased military construction spending. In fact, the MilCon part may be the largest part of any U.S. funding increase. While I know that you are continuing to refine costs, could you briefly walk us through how you derived your current cost estimates?

    Answer. DoD has not refined its earlier NATO enlargement cost estimate; however, DoD has assessed the December 1997 NATO cost estimate for the common-funded costs of NATO enlargement. That NATO report is directly based on initial enlargement requirements developed by NATO Military Authorities in the areas of C3, air defense and air C2, infrastructure for reinforcement, and training and exercises.

    NATO's estimate to meet the initial common-funded requirements for enlargement is around $1.5 billion, of which approximately $1.3 billion will be capital costs paid from the NATO Security Investment Program (NSIP). The U.S. portion of this figure, based on the current NATO cost-sharing formula, is roughly $350 million.

    NATO experts visited many of the sites in the invited countries that may be provided for NATO's use, and their estimates reflect an understanding of the condition of these facilities. In developing their cost estimates, these experts applied their knowledge of NATO common-funded eligibility criteria and of empirical costs of common-funded projects similar to those needed for enlargement.

    These estimates constitute the first step in a longer, comprehensive process of integrating the new members into NATO's defense planning, including detailed site surveys, preparation of detailed operational plans, and implementation of specific capital projects.
 Page 114       PREV PAGE       TOP OF DOC    Segment 1 Of 3  

    DoD has carefully evaluated the capital cost portion of the NATO estimate, including conducting an independent assessment using the same assumption as those used by NATO. The Department has concluded that the NATO figures are sound and reliable—provided that the specific facilities provided for NATO's use have essentially the same characteristics as those visited by NATO experts in developing this estimate.

Base Realignment and Closure

    Question. At first, your statement euphemistically mentions DoD's need to ''shed its excess infrastructure.'' Later you come right out and utter the words: more ''base closure.''

    As your statement notes, the Defense Authorization bill prohibits further base closure until DoD reports on the cost savings.

    Even if the cost savings are certified, I want a report on what you are going to do to ensure that any future base closure round is fair to our communities. How are you going to ensure that proposals are by the numbers and by merit?

    Answer. The legislation we have proposed for BRAC rounds in 2001 and 2005 includes the details of how the process will be conducted. The legislation is modeled on the law authorizing BRACs II, III and IV, which called for the use of published selection criteria, a plan for force structure, certified data and an independent Commission to review recommendations. The BRAC process offers the Department, the Congress and local communities affected by realignments and closures substantial advantages over alternative approaches. This process is also a fair, open, and orderly process. No better approach has been found to reduce DoD's excess base structure. Additionally, DoD has a strong track record for helping communities affected by BRAC. The proposed legislation also facilitates significant assistance to communities for economic assistance, personnel transition, property disposal, environmental cleanup, base reuse planning, and other challenging aspects of the BRAC process. DoD has and will continue to take these commitments to the communities very seriously.
 Page 115       PREV PAGE       TOP OF DOC    Segment 1 Of 3  

    [CLERK'S NOTE.—End of questions for the record submitted by Congressman Hobson.]

    [CLERK'S NOTE.—The following are questions for the record regarding projects and activities under the ''Military Construction, Defense-wide'' account.]

Base Closure and Realignment Schedules

    Question. Submit for the record a copy of the most recent quarterly report on Base Closure and Realignment Schedules, issued by the Office of Economic Adjustment.

    Answer. A copy of the requested report is attached.
    "The Official Committee record contains additional material here."

Metric Conversion Charts

    Question. Submit for the record tables that will facilitate conversion between metric and customary measures.

    Answer. A table providing area, length, and volume conversion factors follows. In addition, a copy of Metric Guide for Federal Construction is attached.

Table 26



 Page 116       PREV PAGE       TOP OF DOC    Segment 1 Of 3  
    "The Official Committee record contains additional material here."

BALLISTIC MISSILE DEFENSE ORGANIZATION

KWAJALEIN—MULTIPURPOSE MISSILE TEST FACILITIES ($4,600,000)

    Question. On which island will this project be located?

    Answer. Facilities will be located on Kwajalein Island and Meck Island, about 20 miles northeast of Kwajalein.

    Question. What options were studied for alteration and expansion of existing facilities, rather than new construction?

    Answer. The following options were studied:

    1. On Meck Island, Building 5050, a large multi-purpose support building, will be altered to meet project requirements. This facility was available and more cost effective than new construction or alteration of other facilities.

    2. On Kwajalein Island, building 1500, a large building was studied for expansion. A technical analysis showed the necessary alteration and expansion of Building 1500 would interfere with testing the National Missile Defense (NMD) Ground-Based Radar-Prototype (GBR–P). No other facilities were available for alteration that could meet project requirements. Consequently a new facility was designed for construction to avoid interference with the NMD GBR–P tests.
 Page 117       PREV PAGE       TOP OF DOC    Segment 1 Of 3  

    3. Existing hardstands on Kwajalein cannot be used for high power radar testing such as the THAAD tests without interfering with current installation operations. Therefore construction of a new hardstand was required.

    Question. Under block 12 of the form 1391, will this project include equipment to be provided from other appropriations?

    Answer. The new and altered facilities will support mobile tactically configured components of TMD systems. The first equipment scheduled to use the facilities is the THAAD mobile equipment and tests sets that are included in the FY92–99 RDT&E appropriations. No installed equipment from other appropriations is planned.

DEFENSE MEDICAL FACILITIES

    Question. Of the 27 medical projects included in the budget request, identify which projects (if any) are justified primarily by the need to correct accreditation deficiencies.

    Answer. None of the projects in the FY 99 Medical MILCON Program are justified primarily by the need to correct accreditation deficiencies. Only the Pensacola Naval Hospital Addition/Alteration/LSU project contains work to correct accreditation deficiencies. In this project, $3,108,000 of the total cost of $25,400,000 will be utilized for correcting life safety, fire protection, and electrical deficiencies.

 Page 118       PREV PAGE       TOP OF DOC    Segment 1 Of 3  
Beale Air Force Base, California—Physiological Support Facility

ADDITION/ALTERATION ($3,500,000)

    Question. It appears that this project is justified primarily due to the physiological training requirements of the U–2 aircraft. How many U–2 aircraft are in the inventory, how old are they, and what is the expected remaining service life of these aircraft?

    Answer. There are 35 U–2s in the inventory. The fleet is relatively new with an average airframe flying time of 5300 hours. The expected service life of these aircraft is beyond 2035.

    Question. How long has this function been performed in the current facility, and why is this method of operations no longer acceptable?

    Answer. The original facility was constructed in 1966 as a SR–71 alert facility. There have been several building additions over the years as the mission gradually evolved to its present training and support functions. In addition to supporting the worldwide U–2 mission, the 9th Physiological Support Squadron provides physiological training to aircrews in the Northwest Region. The facility has always been significantly short of training and storage space, which negatively impacts their mission. Lack of adequate classroom space continues to hamper their pilot and technician training programs. Lack of maintenance space has forced the staff to use two trailers adjacent to their building for testing and calibration. Lack of storage space has resulted in the use of metal shipping containers to store mobility assets. Perhaps most importantly, current space constraints have forced the staff to mix training equipment with operational assets. This is a majority safety concern because U–2 pilot full pressure suits (valued at $100K each) and seat kits (containing emergency oxygen system and survival gear) keep the pilots alive at high altitude. If a training suit or seat kit were to be inadvertently used in a mission, pilots' lives would be placed at risk.
 Page 119       PREV PAGE       TOP OF DOC    Segment 1 Of 3  

CAMP PENDLETON MARINE CORPS BASE, CALIFORNIA—MEDICAL/DENTAL

CL REPL (MARGARITA) ($3,100,000)

CL REPL (SAN MATEO) ($3,200,000)

    Question. Bring us up to date with the request for proposals that was issued in mid-October 1997 for conversion of building 1377 in the base headquarters area to establish in-house dental services for retired and dependent personnel, using licensed professionals from local communities.

    Has final selection/award been made?

    How will these two military construction projects in support of the amphibious training mission affect that morale, welfare, and recreation effort?

    Answer. Alterations to Building 1377 is an MWR initiative. Navy MWR reports that the renovations to building 1377 began on 26 January 1998. The facility is expected to open for service in late July 1998. The service contract was awarded on 20 February 1998. The Margarita and San Mateo projects will have no effect on the morale, welfare, and recreation project. The medical MILCON projects will provide treatment to active duty only. The MWR facility will provide dental services for family members and retirees on a fee for service basis.

    Question. These two military construction projects will vacate buildings 33305, 33306, 62305, and 62306, which are described as ''deteriorated beyond the point of economical repair ... dysfunctional ... (with) major mechanical and plumbing problems''. These buildings will then be returned to the base for disposition. Why not demolish these buildings, and include the cost of demolition in these projects?
 Page 120       PREV PAGE       TOP OF DOC    Segment 1 Of 3  

    Answer. Camp Pendleton Marine Corps Base is the property record cardholder for these four buildings. The base has indicated they wish to utilize these buildings for administrative purposes to alleviate their shortage of administrative spaces. Administrative spaces have far less utility requirements than those for the clinics.

Edwards AFB, California—Aerospace Medical Clinic

ADDITION/ALTERATION ($6,000,000)

    Question. The current temporary modular facilities will be demolished as part of this project. Are these facilities in the footprint of this project? What is the cost of this demolition?

    Answer. Yes. The cost of demolition is currently estimated at $139,000 as shown on the DD Form 1391, block 9 under Support Facilities.

    Question. This project is justified, in part, due to ''growing beneficiary populations''. What accounts for this growth?

    Answer. ''Growing beneficiary populations'' is a result of evolving mission requirements due to continual additions and conversions of flight test squadrons. Current and projected missions include the F–22, C–17, B–2, B–1, C–130, F–16, F–15, Aria, Speckled Trout, and X–33. Flight medicine providers are assigned based on number of squadrons and types of missions. In addition, the Bioenvironmental Engineering (BEE) mission is growing due to increases in environmental regulations. This is particularly true given the large number of industrial sites at Edwards AFB, and the additional regulatory requirements in the state of California.
 Page 121       PREV PAGE       TOP OF DOC    Segment 1 Of 3  

San Diego Naval Hospital, California—Water Storage Tank ($1,350,000)

    Question. The form 1390 for this project states that the estimated cost to remedy the deficiencies in all existing medical facilities at this installation is $10,978,000. Provide for the record a list of this real property maintenance backlog, and describe the program to remedy this backlog.

    Answer. Current Real Property Maintenance (RPM) Backlog is as shown below:

    Naval Medical Center San Diego (NMCSD), Bldg. 1: $4,262,200.

    NMCSD, All other Buildings: $3,710,000.

    NMCSD, Branch Clinics: $1,400,000.

    Total Current RPM Backlog: $9,372,200.

    The total amount of RPM backlog fluctuates due to continuous completion and addition of individual projects.

    Real property maintenance requirements are identified through an Annual Inspection Summary (AIS) at each facility. The Navy Bureau of Medicine and Surgery (BUMED) programs approximately 3.0% of plant replacement value in maintenance funds to correct identified deficiencies and perform scheduled maintenance. Two thirds of the programmed amount is provided to the activity facility manager to perform small maintenance, repair, equipment installation, and repair projects that are within the commanding officers funding authority. The BUMED managed special projects program is the vehicle for accomplishing maintenance, repair, equipment installation, and construction projects when funded costs exceed the activity commanding officer's funding authority but are less than Military Construction scope. The activity prepares and submits special project documentation upon determination of a major facility, safety, or environmental deficiency. The project documentation is reviewed and validated by the responsible NAVFAC engineering field division and forwarded to BUMED via the regional Healthcare Support Office. BUMBED then spends the remaining one third on projects that have been prioritized and programmed at its annual special projects programming Board for that given fiscal year.
 Page 122       PREV PAGE       TOP OF DOC    Segment 1 Of 3  

    Question. What is the cost of landscaping that is included in this project, separately identifying the portion that is functional as opposed to aesthetic.

    Answer. Landscaping costs for this project is estimated at $3,280. The landscaping included in the project is primarily for erosion control on the hillside, but will also function aesthetically.

    Question. Is it correct that there is no current need for this project, that the tank would provide fresh water that would be needed only in the event of an earthquake?

    Answer. Naval Medical Center San Diego (NMCSD) is located in an area rated as seismic zone 4. NMCSD is considered an essential facility and is part of the regional emergency preparedness response plan. DoD technical criteria require a separate water storage facility, as a source of supply for three days, at all essential facilities. This water will be available in the event of an earthquake or any other disaster disrupting city supply.

Travis Air Force Base, California—Patient Movement Items OPS and Dist Ctr ($1,700,000)

    Question. What are ''patient movement items'', and what will be the total value of such items that will be stored in this facility?

    Answer. Patient Movement Items (PMI) are crucial medical equipment and supplies, such as ventilators, infusion pumps, litters, drugs and dressings, necessary to support a patient through the Aeromedical Evacuation (AE) System in both peace and wartime. At Travis AFB, there will be over 1,500 individual pieces of medical equipment along with 10,000 durable assets (litters, restraints, blankets, etc), consumable supplies for the equipment, spare parts, test equipment, and an information management system. The aggregate value of the PMI Program assets for Travis AFB will be $7.9 million.
 Page 123       PREV PAGE       TOP OF DOC    Segment 1 Of 3  

    Question. These items will be purchased beginning in fiscal year 1998. When will the construction of this project be completed, and where will the items be stored until it is completed?

    Answer. Items purchased prior to project completion will be temporarily stored at Operating Location 2, Fort Worth, Texas. They will be forwarded to Travis (Western Hub) when the project is completed around March 2000.

    Question. Why will this project be executed as a design-build contract?

    Answer. The design-build process is used for simple straightforward projects that can be designed and executed in the year of their appropriations. The PMI warehouse project meets that requirement. Design-build projects tend to reduce design and construction costs, time, risks and change orders.

Eglin Air Force Base, Florida—Central Energy Plant ($9,200,000)

    Question. The form 1390 for this project states that the estimated cost to remedy the deficiencies in all existing medical facilities at this installation is $34,630,000. Provide for the record a list of this real property maintenance backlog, and described the program to remedy this backlog.

    Answer. The following provides a list of the real property maintenance backlog at this installation:
 Page 124       PREV PAGE       TOP OF DOC    Segment 1 Of 3  

    Life Safety Violations (firewalls, penetrations, doors, etc)

    Handicapped Accessibility Violations (handrails, casework, signage, etc)

    Fire Protection/Sprinklers

    Domestic Water System

    Medical Gases

    Sanitary Sewer

    Heating, Ventilation, and Cooling System

    Electric System

    Asbestos Abatement

    The backlog is dynamic and changes as projects are completed or added to the Annual Inspection Summary. A facility master plan has been established to correct all of these deficiencies over a period of five years.

    Question. The Services have robust programs to get out of the utility business through privatization. What privatization and other alternatives were studied before deciding to proceed with this central energy plant project?
 Page 125       PREV PAGE       TOP OF DOC    Segment 1 Of 3  

    Answer. Privatization is concerned with utility suppliers and primary power generation facilities. The CEP does not generate power or water, and is, therefore, not a candidate for privatization. The CEP project upgrades and relocates heating, cooling, and emergency power equipment and provides a new building in which to house this equipment.

    Question. How many parking spaces will be displaced by this project, and what is the replacement cost that is included in this project?

    Answer. The project will displace 48 parking spots at a cost of approximately $50K.

    Question. How will the space occupied by the existing plant, which is structurally sound but on the basement level, be reutilized?

    Answer. The existing Energy Plant will continue to house pumps, electrical distribution panels, primary mechanical piping, and spare parts.

Pensacola Naval Air Station, Florida—Hospital

ADDITIONAL/ALTERATION/LU ($25,400,000)

    Question. This project is justified, in part, by the fact that ''demand for services is greater than the current facility limitations can accommodate''. To what extent is this demand a BRAC–93 generated workload, as a result of relocation of personnel from NAS Memphis? Why is this project not requested under the BRAC–93 account, either for exclusive BRAC funding, or for conjunctive funding?
 Page 126       PREV PAGE       TOP OF DOC    Segment 1 Of 3  

    Answer. Requirements for the addition, alteration and life safety upgrades are due to the age of the facility, changes in criteria, command organization, and the standard of practice. Facility deficiencies existed prior to BRAC 93. Additional beneficiaries brought about by BRAC migrations are not considered a significant impact to the core medical facility. A BRAC funded project in FY 1996 ($4.2 million) provided for an expansion of a branch clinic providing primary care to active duty personnel.

Moody Air Force Base, Georgia—CMF Alteration/Dental Cl Add ($11,000,000)

    Question. This project will convert existing hospital space to an Ambulatory Health Care Center configuration. How much hospital space will remain, and does this meet the requirement?

    Answer. At the completion of this project, no inpatient capability will remain. All current hospital space will be altered or converted to meet required clinic missions. The facility will be redesignated for outpatient care only.

Fort Stewart, Georgia—Medical/Dental Clinic Replacement ($10,400,000)

    Question. This project is justified, in part, by the requirement to provide medical and dental service to ''active duty personnel assigned to Hunter Army Air Field, their dependents and other authorized beneficiaries''. How are non-active duty beneficiaries factored into sizing decisions for medical facilities?

 Page 127       PREV PAGE       TOP OF DOC    Segment 1 Of 3  
    Answer. Non-active duty beneficiaries are factored into sizing decisions for medical facilities in a variety of combinations. Family members of active duty beneficiaries are counted and provided care identical to active duty beneficiaries in medical clinics but not in the dental clinics. Retirees and their family members are provided care on a space available basis in both medical facilities. These beneficiaries are not included in the figures justifying staffing requirements. National Guard and US Army Reserve personnel are counted the same as Active Duty when working full time. Survivors of deceased members are often very small in number and usually are included.

    The beneficiary categories included as discussed above provide the simple mathematics to obtain a figure. However, the figure is reviewed in the Economic Analysis, and depending on cost of visits in different clinical disciplines (e.g. Pediatrics, Psychiatry, Podiatry, etc.), appropriate staffing levels are considered.

    Sizing is a function of patients, staff, mission, and logistics requirements. During the Economic Analysis, if it appears more economical to outsource several specialities, none of the beneficiaries would be considered except active duty and family members.

    Question. When this project is completed, will the contract clinic in Savannah cease to operate? If so, what is the estimate of recurring annual savings?

    Answer. Reference to the Savannah contract clinic is in error. The Savannah PRIMUS clinic was in fact closed in December 1996 at the direction of the Installation Commander. The PRIMUS clinic contract was not renewed and the services were moved to Hunter Army Airfield for better access and enhanced quality of service. Savings from closing the PRIMUS clinic are estimated to be $1.6 million. However, the Fort Steward MEDDAC budget was reduced accordingly, eliminating any local reinvestment.
 Page 128       PREV PAGE       TOP OF DOC    Segment 1 Of 3  

    Question. Upon completion of this project, dental clinic 3 as well as the Tuttle clinic will be turned over to the base. Are these facilities structurally sound and reusable?

    Answer. The installation Department of Public Works has determined these buildings to be structurally sound and plans to reuse them for non-medical functions.

Great Lakes Naval Station, Illinois—Hospitalman ''A'' School Addition ($7,100,000)

    Question. This project is justified, in part, by the ''consolidation of Hospitalman schools at Great Lakes''. Is this a consolidation of schools already located at Great Lakes or a relocation of schools from elsewhere? If from elsewhere, was this a BRAC-generated requirement?

    Answer. No, it is not a BRAC-generated requirement but a self-initiative. The Inter-service Training Review Organization examined the possibility of consolidating service schools for all the services. Service unique requirements proved this initiative too difficult to carry out, but Navy Medicine realized the benefit through consolidation of the two ''A'' schools (San Diego, CA and Great Lakes, IL) at Great Lakes. The consolidation was completed in FY 96 with scattered on-base locations. The MILCON project will satisfy space constraints, as a result of the consolidation, and functional requirements that had not previously existed (e.g. training labs).

Barksdale Air Force Base, Louisiana—Clinic Addition/Alteration ($3,450,000)

 Page 129       PREV PAGE       TOP OF DOC    Segment 1 Of 3  
    Question. This project will house pediatrics, immunizations, and flight medicine. Roughly what percent of the cost of the project is for each of the three functions?

    Answer. The estimated cost breakdown is: Pediatric clinic $1,380,000 (40%); Aerospace Medicine Squadron $1,380,000 (40%); and Immunizations function $690,000 (20%) for a total of $3,450,000.

Keesler Air Force Base, Mississippi—Bioenvironmental Eng Fac Repl ($700,000)

    Question. Why is this replacement facility more than twice as large as the space currently occupied by the bioenvironmental engineering function . . . 390 square meters versus 158 square meters?

    Answer. The Bioenvironmental Engineering (BEE) function is shoe-horned into an old dormitory building that is approximately 50% undersized based on need and DoD space planning criteria. This dormitory facility was intended to serve as temporary space to house the BEE until a permanent facility of appropriate size could be constructed. Lack of space has forced the BEE to inefficiently split their operations and use the hospital laboratory for critical water lab functions.

Holloman Air Force Base, New Mexico—War Readiness Material Warehouse ($1,300,000)

    Question. This project is justified, in part, by the fact that the existing warehouse lacks climate control, which causes ''the government to replace materials early at an enormous cost''. What is the value of the material to be stored, and how quickly will the new facility pay for itself by avoiding such costs?
 Page 130       PREV PAGE       TOP OF DOC    Segment 1 Of 3  

    Answer. This warehouse is required for mission readiness. It will store over $3.8 million in WRM assets, $200,000 of which are expiration-dated items requiring temperature control that are located in 50 storage containers and ISO-shelters. In as little as six months, these items require replacement due to the extreme hot-cold

temperature or exposure to sun and precipitation. The same inadequate storage conditions also severely affect the remaining assets, including promoting untimely failure of sensitive medical equipment. Manufacturers will not warranty material or equipment or efficacy of drugs stored in conditions other than those recommended as safe for preserving their reliability. Payback is estimated to be less than 3 years for expiration-dated items itself.

Fort Bragg, North Carolina—44th Medical Brigade WRM Warehouse ($6,500,000)

    Question. This appears to be a straightforward warehouse project. Why can't it be executed using a standard or definitive design, in order to avoid most or all of the $490,000 total design cost?

    Answer. This project has unique medical requirements that preclude use of a standard or definitive warehouse design. These requirements include optical fabrication and repair operations, medical maintenance repair operations, blood storage and distribution, refrigerated medical supply storage, vault storage for controlled substances, and humidity-controlled areas for medical supplies with unique equipment storage requirements.

Grand Forks Air Force Base, North Dakota—Medical Dental Clinic Add/Alt ($5,600,000)
 Page 131       PREV PAGE       TOP OF DOC    Segment 1 Of 3  

    Question. Is it correct that this project will vacate the existing dental clinic, and that that space will be reutilized by the bioenvironmental engineering and the military public health functions?

    Answer. Yes.

    Question. Will the existing utilities, which are encased in the concrete slab on grade, be abandoned in place in order to avoid malfunctions and maintenance problems?

    Answer. Yes.

Carlisle Barracks, Pennsylvania—Health Clinic Addition ($4,678,000)

    Question. There is a companion O&M project, budgeted at $3,500,000, to bring the existing clinic up standards for ambulatory services. How will the O&M work be divided from the military construction work? Will these projects be executed as separate bids and separate contracts? Will they be bundled into a single contract?

    Answer. The project was separated between O&M and MILCON based on definitions of work classification. The Military Construction portion will be used to build an addition to the existing facility. Both O&M and MILCON contracts will be bundled together for bidding and contractual purposes but separate accounting will be maintained for each type of work. To keep the clinic operational during construction, the current project transition plan has identified a combined usage of temporary facilities and the new addition as swing space during the O&M renovation.
 Page 132       PREV PAGE       TOP OF DOC    Segment 1 Of 3  

Fort Hood, Texas—Blood Donor Center ($3,100,000)

Fort Hood, Texas—Primary Care Clinic ($11,000,000)

    Question. Why were these two efforts presented as separate projects, rather than being combined into a single project?

    Answer. The Primary Care Clinic and the Blood Donor Center accommodate unique and separate missions with different staffs and operating conditions and are, therefore, not combined.

    Question. Will they be executed as separate bids and separate contracts, or will they be bundled into a single contract?

    Answer. These projects have different design and construction awards schedules. Therefore, they will not be bundled into a single contract.

    Question. The blood donor center is currently housed in three inadequate, aging buildings, two of which are World War II temporary barracks infested with termites. Will these buildings be demolished upon completion of this project?

    Answer. Yes.

Cheatham Annex, Virginia—Fleet Hospital Support Operation Admin Ofc ($1,900,000)
 Page 133       PREV PAGE       TOP OF DOC    Segment 1 Of 3  

    Question. Is it correct that the requirement for this project was created by the BRAC closures in Oakland and Alameda? Why is it not completely or conjunctively funded under the BRAC account?

    Answer. The requirement for replacement of these facilities was pre-existing and originally programmed and approved by the Congress in FY 92 ($22 million) at Stockton, CA. The project provided for a temperature and humidity controlled warehouse with administrative space and maintenance functions. DoD/IG recommended that the project be deferred due to force restructuring at that time and the need for future fleet hospital storage requirements. Office of the Assistant Secretary of Defense (Health Affairs) agreed with this recommendation and the Congress was duly notified. With the BRAC closure of Alameda and Oakland, Navy decided to consolidate the Fleet Hospital Life Extension Program at Cheatham Annex, VA. Two BRAC projects were approved in FY98, Cargo Staging Area at a cost of $1,443,000 and a building renovation in the amount of $2,837,000 to support this effort. Remaining 3 projects (2 in FY99 and one in FY00 for construction of hard stand) have also been pursued for BRAC funding unsuccessfully.

    Question. This project is justified, in part, by the fact that ''ownership of the third site, Rough and Ready Island Stockton, could transfer from Naval Communications Station Stockton to the Port of Stockton under special land conveyance legislation''. Submit a copy of this legislative proposal for the record, together with a justification for the conveyance and the estimated value of the property.

    Answer.
 Page 134       PREV PAGE       TOP OF DOC    Segment 1 Of 3  

Naval Communication Station, Stockton, California, Special Legislation

    SEC 2871, the National Defense Authorization Act for Fiscal Year 1996, Public Law 104–106 (10 February 1996)

    Section. 2871, Land Conveyance

    (a) Conveyance Authorized—Subject to subsection (b), the Secretary of the Navy may convey to the Port of Stockton, California (In this section referred to as the ''Port''), all right, title, and interest of the United States in and to a parcel of real property, including any improvements thereon, consisting of approximately 1,450 acres at the Naval Communication Station, Stockton, California.

    (b) Requirement for Federal Screening of Property—The Secretary may not carry out the conveyance of property authorized by subsection (a) unless the Secretary determines that no department or agency of the Federal Government will accept the transfer of the property.

    (c) Interim Lease—Until such time as the real property described in subsection (a) is conveyed by deed, the Secretary may lease the property, along with improvement thereon, to the Port under terms and conditions satisfactory to the Secretary.

    (d) Consideration—The conveyance may be made as a public benefit conveyance for port development as defined in section 203 of the Federal Property and Administrative Services Act of 1949 (40 U.S.C. 484) if the Port satisfies the criteria in such section and the regulations prescribed to implement such section. If the Port fails to qualify for a public benefit conveyance and still desires to acquire the property, the Port shall pay to the United States an amount equal to the fair market value of the property to be conveyed, as determined by the Secretary.
 Page 135       PREV PAGE       TOP OF DOC    Segment 1 Of 3  

    (e) Federal Lease of Conveyed Property—As a condition for transfer of this property under subparagraph (a), the Secretary may require that the Port lease to the Department of Defense or any other Federal agency all or any part of the property being used by the Federal Government at the time of conveyance. Any such lease shall be made under the same terms and conditions as in force at the time of the conveyance. Such terms and conditions will continue to include payment to the Port for maintenance of facilities leased to the Federal Government. Such maintenance of the Federal premises shall be to the reasonable satisfaction of the United States, or as required by all applicable Federal, State, and local laws and ordinances.

    (f) Description of Property—The exact acreage and legal description of the property to be conveyed under subsection (a) shall be determined by a survey satisfactory to the Secretary. The cost of the survey shall be borne by the Port.

    (g) Additional Terms—The Secretary may require such additional terms and conditions in connection with the conveyance under subsection (a) or the lease under subsection (c) as the Secretary considers appropriate to protect the interests of the United States.

    It is important to note that the decision for the relocation and consolidation at Cheatham Annex was based on economic considerations as explored in the Alternative Analysis/Site Evaluation Study of 15 Jul 96.

Cheatham Annex, Virginia—Fleet Hospital Sup Ops Operational Whse ($9,400,000)

 Page 136       PREV PAGE       TOP OF DOC    Segment 1 Of 3  
    Question. Is it correct that requirement for this project was created by the BRAC closures in Oakland and Alameda? Why is it not completely or conjunctively funded under the BRAC account?

    Answer. The requirement for replacement of these facilities was pre-existing and originally programmed and approved by the Congress in FY 92 ($22 million) at Stockton, CA. The project provided for a temperature and humidity controlled warehouse with administrative space and maintenance functions. DoD/IG recommended that the project be deferred due to force restructuring at that time and the need for future fleet hospital storage requirements. Office of the Assistant Secretary of Defense (Health Affairs) agreed with this recommendation and the Congress was duly notified. With the BRAC closure of Alameda and Oakland, Navy decided to consolidate the Fleet Hospital Life Extension Program at Cheatham Annex, VA. Two BRAC projects were approved in FY 98, Cargo Staging Area at a cost of $1,443,000 and a building renovation in the amount of $2,837,000 to support this effort. Remaining 3 projects (2 in FY99 and one in FY00 for construction of hard stand) have also been pursued for BRAC funding unsuccessfully.

    Question. Where are the ten prepositioned fleet hospitals located, what is the value of a single hospital, and what is the cost to rebuild a single hospital?

    Answer. The ten prepositioned fleet hospitals are located as follows: Norway—2; Sasebo, Japan—1; Okinawa, Japan—2; Guam—1; South Korea—1; Maritime Squardron, Indian Ocean—1; Service Life Extension Program (SLEP)—2 (at Cheatham Annex, VA).
 Page 137       PREV PAGE       TOP OF DOC    Segment 1 Of 3  

    The current replacement value of a Fleet Hospital is $37 million. The cost to rebuild a Fleet Hospital is approximately $6.45 million. Of this cost, approximately $450,000 is for contractor labor. Actual cost varies with the extent of repair and maintenance required.

    Question. Will the administrative office and the operational warehouse be executed as a single contract?

    Answer. Yes.

Portsmouth Naval Hospital, Virgina—Hospital Replacement Phase X ($17,954,000)

    Question. The form 1390 for this project states that the estimated cost to remedy the deficiencies in all existing medical facilities at this installation is $11,873,000. Provide for the record a list of this real property maintenance backlog, and described the program to remedy this backlog. Does this estimate include deficiencies in building 215 that will be corrected by this project?

    Answer. The requested list can be provided under separate cover. The backlog is dynamic and changes as projects are completed or added to the Annual Inspection Summary. Real property maintenance requirements are identified through an Annual Inspection Summary (AIS) at each facility. The Navy Bureau of Medicine and Surgery (BUMED) programs approximately 3.0% of plant replacement value in maintenance funds to correct identified deficiencies and perform scheduled maintenance. Two thirds of the programmed amount is provided to the activity facility manager to perform small maintenance, repair, equipment installation, and repair projects that are within the commanding officers funding authority. The BUMED managed special projects program is the vehicle for accomplishing maintenance, repair, equipment installation, and construction projects when funded costs exceed the activity commanding officer's funding authority but are less than Military Construction scope. The activity prepares and submits special project documentation upon determination of a major facility, safety, or environmental deficiency. The project documentation is reviewed and validated by the responsible NAVFAC engineering field division and forwarded to BUMED via the regional Healthcare Support Office. BUMED then spends the remaining one third on projects that have been prioritized and programmed at its annual special projects programming Board for that given fiscal year.
 Page 138       PREV PAGE       TOP OF DOC    Segment 1 Of 3  

    This estimate includes Building 215 deficiencies. The Building 215 backlog will be eliminated upon completion of this project.

    Question. What is the status of the GAO review of the proposed renovation of building 215?

    Answer. The GAO review was completed in June 1997 and results are included in their ''Report to Congressional Requesters; MILITARY CONSTRUCTION, Renovation Plans at the Portsmouth Naval Medical Center (GAO/NSIAD–97–144).'' The Report concluded that fully renovating Building 215 is a practical option. Based on the net present-value analysis, a full renovation was determined to be more cost-effective over a 30-year period.

    Question. List, by phase, the total cost for the Portsmouth hospital replacement.

    Answer.—

Table 27



    Question. This tenth phase shows an estimated unit cost of $88.50 per square foot to renovate building 215. What would it cost per square foot to simply demolish building 215? Is there an estimate for the plant replacement value of building 215?

    Answer. Complete demolition of Building 215 is estimated to cost $9,847,540 or $19.79 per square foot—but the space must then be replaced to meet the installation's medical program requirements. Alternate scenarios were studied but found to be infeasible based on economics.
 Page 139       PREV PAGE       TOP OF DOC    Segment 1 Of 3  

    The current plant value for building 215 is $95,957,168.

    Question. If this alteration and modernization phase is completed, what are the estimated annual savings that would result from all sources, including consolidation of essential medical and medical support function from five different locations in Hampton Roads, and consolidation of TRICARE and the Healthcare Support Office in Norfolk?

    Answer. Approximately 20% of Building 215 (105,000 of the 497,500 square feet) is designated for consolidation of the referenced functions. The annual savings are estimated to be $1.6 million to offset lease costs. Renovating space in Building 215 for these tenants is estimated at $5,900,000 with a payback in less than 4 years.

Bangor Naval Submarine Base, Washington—Disease Vector Ecology and Control Center ($5,700,000)

    Question. Is it correct that the requirement for this project was created by the BRAC–95 closure of NAS Alameda? Why is it not completely or conjunctively funded under the BRAC account?

    Answer. No. The requirement for this project was identified and a MILCON project was developed prior to BRAC–95. Relocation of these activities had begun to temporary locations and thus the project was not eligible for BRAC funding.

    Question. What is the annual cost of the two leased spaces at Poulsbo and Keyport, Washington, in which the Center is currently operating?
 Page 140       PREV PAGE       TOP OF DOC    Segment 1 Of 3  

    Answer. Poulsbo: $155,000 per year; Keyport: $66,760 per year. Current facilities are inadequate and dysfunctional due their location in the residential area and safety concern from the chemicals. Other suitable facilities are not available to meet the mission requirements.

Bremerton Naval Hospital, Washington—Hospital Addition/ Alteration ($28,000,000)

    Question. Roughly 11.5% of the cost of the primary facility to be constructed under this project is for a parking structure. How many spaces will be provided, at what cost per space? Why is it necessary to include a parking structure, at a cost of $82.16 per square foot, rather than surface parking?

    Will this project complete all medical facilities construction requirements in Bremerton?

    Answer. The project provides 250 spaces in the parking structure at a cost of $2,258,000. The Bremerton hospital site is of limited area with sloping topography and environmentally sensitive. All land available for surface parking is already being utilized. A vertical structure is the only solution. The cost of the parking structure is $24.11 per square foot. The $82.16 per square foot cost is for alterations of the existing hospital. This project will meet all current medical facilities construction requirements at Naval Hospital Bremerton.

McChord Air Force Base, Washington—Clinic/WRM Warehouse Replacement ($20,000,000)

 Page 141       PREV PAGE       TOP OF DOC    Segment 1 Of 3  
    Question. Will this project construct a single facility? Why are such dissimilar medical missions combined into a single project?

    Answer. This project will construct two facilities connected by an environmentally controlled hallway.

    The project is a clinic and logistics support replacement project. The logistics support facility will contain the critical support functions for the clinic such as housekeeping, facility management, central sterilization, medical supply, and biomedical equipment repair. Medical WRM assets are also to be stored in the warehouse portion of the logistics support facility, along with operating stocks of medical supplies.

    Question. Provide for the record a list and description of all facilities that will be vacated upon completion of this project, including the intended disposition of such facilities.

    Answer.

    Building 100, Base Headquarters Building (1940).—Small portion of the building occupied by medical clinic will be returned to the base for their use

    Building 160, Main Clinic (1939).—This building will be returned to the base. It is being nominated for inclusion on the Washington State Historic Registry.

 Page 142       PREV PAGE       TOP OF DOC    Segment 1 Of 3  
    Building 161, Garage (1940).—3 bay storage garage, to be turned over to the base for use with building 160.

    Building 164, Outpatient Records (1987).—Pre-Engineered building, to be demolished

    Building 165, Radiology/Ambulance Service (1987).—Pre-Engineered building, to be demolished

    Building 168, Pharmacy/Lab (1988).—Pre-Engineered building, to be demolished

    Building 169, Family Medicine (1989).—Pre-Engineered building, to be demolished

    Building 170, Mental Health (1996).—Pre-Engineered building, to be demolished

    Building 173, Flight Medicine (1988).—Pre-Engineered building, to be demolished

    Building 435, Oxygen Storage (1943).—Wood structure, to be demolished

    Building 501, Warehouse (1942).—Storage warehouse, wood construction, to be demolished
 Page 143       PREV PAGE       TOP OF DOC    Segment 1 Of 3  

    Building 507, Warehouse (1942).—Storage warehouse, wood construction, to be demolished

    Building 518, BEE Bldg (1951).—Shared with Civil Engineering (CE), space vacated by Bioenvironmental Engineering will be absorbed by CE.

Sigonella Naval Air Station, Italy—Flight Line Dispensary ($5,300,000)

    Question. How does a ''dispensary'' differ from a ''clinic?''

    Answer. The host nation defines a clinic as a hospital. For this reason, the term dispensary was selected as a more accurate description of the proposed outpatient facility to avoid compliance with hospital standards.

    Question. What is the definition of ''category E equipment?''

    Answer. Category E equipment is a reference to the equipment that is government furnished and contractor installed from Military Construction Appropriation funds such as X-ray equipment, sterilizers, etc.

    Question. Provide for the record a list of such equipment that is included in the cost of this project, totaling $350,000.

    Answer. This figure represents cost estimates for category E and F equipment. Category F equipment is government furnished and government installed from Military Construction Appropriation funds. The proposed equipment is:

Table 28


 Page 144       PREV PAGE       TOP OF DOC    Segment 1 Of 3  

RAF Lakenheath, United Kingdom—Hospital Annex Replacement ($10,800,000)

    Question. To what extent do British and U.S. forces share medical facilities and services at RAF Lakenheath?

    Will the existing hospital annex be demolished upon completion of this project?

    If so, why is the cost of demolition not included in this project?

    If not, why not, and what is the intended reuse?

    Answer. British forces do not ''share'' U.S. medical facilities. British military personnel are treated as NATO military personnel. The only time healthcare should be utilized at a U.S facility by a NATO military member would be in an emergency situation.

    The existing hospital annex will be demolished upon completion of this project. The cost of demolition ($203,000) is included in the project under Supporting Facilities, block 9 of the DD Form 1391. An additional $250,000 is listed under Primary Facility for related asbestos removal in the annex.

Management of Medical Projects

    Question. As part of the Defense Reform Initiative, the Defense Medical Program Activity has been disestablished and replaced with the Tricare Management Activity. Who is now responsible for the distribution of resources for medical construction, the Services or OSD? Who is now responsible for sizing and engineering decisions regarding medical construction? That is, do the Services have a free hand or are they subject to OSD review and approval?
 Page 145       PREV PAGE       TOP OF DOC    Segment 1 Of 3  

    Answer. Tricare Management Activity (TMA) is responsible for the distribution of medical construction funds and the program management oversight in coordination with the Services.

    The services develop requirements utilizing Tri-service standards and criteria. TMA remains responsible for sizing and engineering decisions based on the Services' recommendations.

DEFENSE LOGISTICS AGENCY

Defense Fuel Support Point Elmendorf AFB, Alaska Replace Hydrant Fuel System ($19,500,000)

    Question. The DD Form 1390 for this project indicates that a second phase of this project will be included in the fiscal year 2000 program at a cost of $29,500,000. Why were advance appropriations for this second phase not requested in the President's budget for fiscal year 1999—as they were for a number of phased projects for the Army and the Navy?

    Answer. The FY 1999 project is a complete, stand-alone fueling system of 12 hydrant outlets that replaces three failing, antiquated fueling systems. Therefore, it was not necessary for the Department to request advance appropriations in order to satisfy OMB's full funding requirement.

    The second phase of the project was not programmed in FY 1999 because planning for this phase was not far enough along to provide reasonable assurance of a construction contract award in FY 1999. Additionally, building these two systems in different fiscal years minimizes the mission impact on the base's flight operations caused by construction activities.
 Page 146       PREV PAGE       TOP OF DOC    Segment 1 Of 3  

    Question. How is the work divided between the two phases?

    Answer. Each project provides a complete and usable hydrant fuel system.

    Question. What would happen if the first phase were funded and the second phase were not?

    Answer. If the second project were not funded, the base would have one fully operational hydrant fuel system of 12 outlets and a failed or failing 40-year-old system of 20 outlets.

    Question. Does the authorization request include both phases, so that they can be awarded as a single contract, subject to the availability of funds in the second year?

    Answer. No. Separate authorization and appropriation will be requested for each project.

    Question. Will the removal of 21 underground tanks be accomplished in the first phase, and is this the $1,500,000 cost for demolition shown on the DD Form 1391?

    Answer. Yes, this project cost is for the demolition of 21 underground fuel storage thanks, 12 hydrant outlet pits, 3 pumphouses, and an extensive network of underground fuel piping.
 Page 147       PREV PAGE       TOP OF DOC    Segment 1 Of 3  

Defense Fuel Support Point Jacksonville-Mayport Annex, Florida Replace Fuel Tanks ($11,020,000)

    Question. The C–1 shows this project at ''Defense Fuel Support Point JAX–ANX'', but the DD Forms 1390 and 1391 show it at ''Naval Station Mayport''. Which is the correct location, or are they the same?

    Answer. Defense Fuel Support Point (DFSP) Jacksonville-Mayport Annex is the commonly referred name for the fuel point located at Naval Station Mayport, FL. This fuel point receives jet and marine diesel fuel by barge from the bulk storage facilities at DFSP Jacksonville, approximately 15 miles northwest of Mayport. The DD Forms 1390 and 1391 for this project show the location as Naval Station Mayport to avoid confusion with a DFSP Jacksonville project also programmed in the DLA FY 1999 Military Construction program.

    Question. Submit for the record a copy of the signed consent agreement with the State of Florida regarding this project.

    Answer. A copy of the signed Consent Agreement between the State of Florida Department of Environmental Regulations and the United States Department of Navy, dated June 3, 1993, is enclosed.
     "The Official Committee record contains additional material here."

Defense Fuel Support Point (DFSP) Jacksonville, Florida Replace Fuel Tanks ($11,000,000)

    Question. This project will replace substandard storage of 44,500 kiloliters capacity with 33,300 kiloliters capacity, a reduction of 25%. Will this fully satisfy the requirement at this location?
 Page 148       PREV PAGE       TOP OF DOC    Segment 1 Of 3  

    Answer. Yes. The proposed construction of three 11,000-kiloliter tanks will satisfy the storage requirement for marine diesel fuel for this DFSP's area of responsibility.

Defense Fuel Support Point Camp Shelby, Mississippi Replace Bulk Fuel Facility ($5,300,000)

    Question. Is it correct that this project will be used almost exclusively by Guard and Reserve forces?

    Answer. No. Based on FY 97 figures, 195,388 service people trained at Camp Shelby. Of this number, the breakdown by component was: 49% Army National Guard, 12% U.S. Army Reserves, 3% Air National Guard, and 36% active forces from all three services. Guard and Reserve forces from Mississippi, Tennessee, Alabama, North Carolina, Florida, Louisiana, Puerto Rico, and the Virgin Islands use this training facility.

    Question. Summarize for the record the economic analysis that determined new construction was more cost-effective than refurbishment and expansion of existing facilities.

    Answer. Initial nonrecurring investment costs are the primary cost drivers in both alternatives. Investment costs for refurbishment of the existing facilities is estimated at $8.9 million versus the proposed new construction cost of $5.3 million. Other relevant considerations in this analysis are as follows:

 Page 149       PREV PAGE       TOP OF DOC    Segment 1 Of 3  
    a. The existing bulk fuel site is too small to accommodate the required fuel dispensing facilities without realigning two roads on Camp Shelby. The realignment of these roads is a significant cost driver in the refurbishment alternative. The new construction alternative does not require any modification of the existing road network.

    b. Refurbishment of the existing facilities to repair only the environmental compliance deficiencies does not address the additional storage and dispensing requirements needed at this installation. These dispensing facilities not only improve operational efficiency by refueling a large number of vehicles quickly, but also provide effective spill containment features not available in the present operations to safeguard fuel transfers. The current site cannot be expanded to add these facilities at a later time without a significant higher overall cost to the government than the proposed project.

    c. The refurbishment alternative will require extensive operating workarounds at the existing site, or more likely, the construction of a temporary off-site fuel point for storage and dispensing during the construction period. Although the cost of this temporary facility has not been quantified, it is a relevant factor in the decision-making process since it impacts on the overall cost, safety, and environmental consideration of this alternative.

    Based on the relevant factors considered in the analysis, we concluded the follow points and recommended the proposed MILCON project:

    a. New construction is the more cost-effective solution to meet the objectives of this project in that it will cost $3.6 million less than refurbishing the existing facility.
 Page 150       PREV PAGE       TOP OF DOC    Segment 1 Of 3  

    b. Repair of the existing facility to only correct environmental compliance deficiencies fails to meet long-term storage and operating objectives of this installation.

    c. Providing a temporary fuel point to accommodate construction of the more costly refurbishment alternative makes this alternative impractical.

Defense Fuel Support Point Pope AFB, North Carolina Hydrant Fuel System ($4,100,000)

    Question: This project is required in order to meet force-generation schedules of Fort Bragg troops. Can you quantify the improvement that is expected over the current truck refueling operations?

    Answer: Yes. The maximum aircraft ground time at this location for en route loading of fuel and explosives for C–17s is 2 hours and 15 minutes and for C–5s, 3 hours and 15 minutes. Fuel and explosives may not be loaded at the same time. Currently, the base uses refueler trucks for fueling these types of aircraft, which may require up to seven truckloads of fuel per aircraft and four-to-five hours, not including the additional time to load ordnance. The refueler trucks must travel across an active runway for each individual refueling trip. Consequently, the base cannot meet U.S. Air Force requirements. In contrast, the proposed hydrant fuel system will enable Pope AFB to fuel two aircraft simultaneously in one hour, satisfying these requirements.

    Question: What savings will be realized in equipment and personnel upon completion of this project?
 Page 151       PREV PAGE       TOP OF DOC    Segment 1 Of 3  

    Answer: Truck refueling requires one person at the truck filing station and one on each refueler truck. If available, two trucks are assigned to each aircraft refueling operation. For the four aircraft parking positions at this site, a total of nine people and eight refueler trucks are needed. The proposed hydrant system uses just one individual at each of the four hydrants, allowing five people and eight refueler trucks to service fighters, transient aircraft and perform other day to day operations.

Defense Fuel Support Point Fort Sill, Oklahoma Replace Fuel Storage Facility ($3,500,000)

    Question: The Fort Sill ground fuel facility was permanently shut down in 1994 due to leaks and failures. Why has it taken four years to program a project to correct this situation?

    Answer: Given the lead times necessary to plan, design, and program a military construction project, the timing of this funds request is appropriate. From July 1994 to February 1995, Fort Sill evaluated alternatives to closing the facility, and coordinated remediation plans with the state. By the time a decision was made in February 1995 to permanently shut down the fueling facility and build a new one, Fort Sill had missed the deadline to submit this project for consideration in the FY 1998 DLA MILCON program. We set this milestone to ensure proposed projects are at the requisite 35-percent design stage for consideration by Congress in a budget submission.

    Question: What economies will be realized upon completion of this project, when ''work-arounds'' can cease?
 Page 152       PREV PAGE       TOP OF DOC    Segment 1 Of 3  

    Answer: This project will provide a consolidated fuel storage and dispensing facility, sized at the required storage volume and dispensing rates to meet Fort Sill's mission as a Power Projection Platform. It will eliminate the need for temporary, undersized fuel tanks located at three separate sites on the installation, which cause additional operating and manpower costs. In addition, the facility will significantly reduce the lost training time tactical units incur getting to these dispersed facilities and waiting in line to fuel their vehicles from inadequate facilities. Of course, the greatest intangible benefits to be achieved by this facility are the improved environmental safeguards to avoid potentially costly fuel-spill cleanups and the ability to meet mobilization/deployment timetables.

Defense Supply Center, Richmond, Virginia Convert Warehouse to Admin Space ($10,500,000)

    Question. This project will convert five sections of warehouse #31 to permanent administration space. How is this space currently used?

    Answer. These sections of the warehouse are currently vacant. This warehouse is located in the administrative area of the supply center. Material previously stored in this space was consolidated in other available warehouse space at the adjacent distribution depot.

    Question. What is the remaining life expectancy of warehouse # 31 upon completion of this project?

    Answer. We expect this facility to be in service for a minimum of 25 years based on its current condition and the installation of new building systems during conversion. The warehouse is a sound masonry structure, similar to other administrative structures on the supply center. A new roof was installed in 1994. In addition, the proposed project will provide for all new electrical, heating, ventilation, air conditioning, and fire protection systems. When completed, this project will provide modern, energy-efficient administrative offices.
 Page 153       PREV PAGE       TOP OF DOC    Segment 1 Of 3  

    Question. Is it correct that this administrative space will house 764 personnel?

    Answer: Yes.

    Question. This project will be equipped with modular furniture provided from the DWCF appropriation at a cost of $5,473,000, or nearly $7,200 per person. Describe this furniture in some detail.

    Answer. Regretfully, we erred in stating the cost for systems furniture shown on the DD Form 1391. The current estimated cost is $3,500,000. We plan to install standard systems furniture components typical of UNICOR's product line or their equivalent.

Defense Fuel Support Point Lajes Field, Azores, Portugal Fuel Pumphouse and Tanks ($7,700,000)

    Question. The primary mission for Lajes Field is to support transient aircraft en route to Europe and beyond. The Government of Portugal does not consider the base a NATO air field, and has excluded the Azores from eligibility for NATO infrastructure improvements. Under the terms of the bilateral agreement with the Government of Portugal, the U.S. is responsible for funding this proposed project in support of U.S. force projection. What would prevent the U.S. from financing this project, and also submitting a precautionary prefinancing statement to NATO for reimbursement of this cost, which supports European reinforcement?

    Answer. Nothing would prevent the U.S. from financing this project and preparing a precautionary prefinancing statement to NATO for reimbursement of the cost. However, before the U.S. Mission could submit this prefinancing statement to NATO for Security Investment Program reimbursement, they would have to obtain the concurrence of the Government of Portugal. In the past, Portugal has not supported NATO projects at Lajes Field, Azores, as stated in the DD Form 1391.
 Page 154       PREV PAGE       TOP OF DOC    Segment 1 Of 3  

    Question. Will this tankage serve U.S. military aircraft exclusively?

    Answer. These proposed tanks support a hydrant fueling system used by U.S. aircraft. As defined in the bilateral agreement, other nations must obtain the approval of the Government of Portugal to use this airfield.

Moran AB Spain

    Question. What is the current status of the hydrant fuel system project at Moron Air Base, for which $12,958,000 was appropriated in fiscal year 1997, and for which this subcommittee required a precautionary prefinancing statement?

    Answer: The project is currently 100 percent designed and ready to advertise for a construction contract pending NATO's acceptance of the Notice of Intent to Prefinance (NIP). The Government of Spain just recently approved the submission of this statement to the NATO Infrastructure Committee. The U.S. Mission to NATO will present this notice to the committee shortly. We expect a contract to be awarded by July 1998 for this work.

Various Locations—Worldwide Conforming Storage Facilities ($1,300,000)

    Question. Last year's request and appropriation for conforming storage facilities (CSF) totaled $11,275,000. What accounts for the great decrease in this year's request?
 Page 155       PREV PAGE       TOP OF DOC    Segment 1 Of 3  

    Answer. Our Defense Reutilization and Marketing Service, responsible for conforming storage facilities, is undergoing significant restructuring to reduce operating costs. On December 3, 1997, DLA announced the closure of 49 Defense Reutilization and Marketing Offices (DRMOs) to realize approximately $35–40 million in annual savings, beginning in Fiscal Year 2000.

    The decrease in our CSF-funding request this year is the result of the uncertainty about which of these DRMOs would be closed. Until this decision was made, planning and design of new CSF's was temporarily suspended. The Camp Lejeune CSF, which was already designed, is the only project ready for programming in Fiscal Year 1999.

    Question. The attachment to the form 1391 lists a single location, Camp Lejeune, North Carolina. Has this location received a RCRA permit for conforming storage? If not, what is the timetable for its RCRA permit?

    Answer. Yes, the RCRA permit is in hand.

    Question. Are other locations under consideration, or is the $1,300,000 specific to Camp Lejeune?

    Answer. With the $11.3 million appropriated in FY 1998 and the $1.3 million requested in FY 1999, DLA will construct three CSFs at Elmendorf Air Force Base (AFB), AK; Oklahoma City, OK; and Camp Lejeune, NC. The DRMOs at these sites will remain open based on the December 3, 1997, decision.
 Page 156       PREV PAGE       TOP OF DOC    Segment 1 Of 3  

DOD DEPENDENT EDUCATION

U.S. Military Academy, New York—School Addition ($2,840,000)

    Question. Is it correct that this project is limited to construction of a multipurpose physical education/therapy facility, and conversation of the existing physical education facility into classrooms?

    Answer. Yes.

    Question. Would this commonly be called a 13,629 square foot elementary school gym?

    Answer. Yes, though it will contain some features specifically designed to aid in the occupational and physical therapy programs

Camp LeJeune, North Carolina—Brewster Middle School ($16,900,000)

    Question. Is this project a complete replacement for the existing middle school?

    Answer. Yes.

    Question. Will the existing school be demolished upon completion of this project?
 Page 157       PREV PAGE       TOP OF DOC    Segment 1 Of 3  

    Answer. The existing facility is on the real property records of the Marine Corps, and may continue to be used for some other function after it is vacated by DDESS.

Elementary School, Guam ($8,600,000) High School, Guam ($4,500,000)

    Question. Provide for the record a history of events in the past several years that led to the request for these two school projects on Guam.

    Answer. Beginning in 1988, the Department of Defense contracted with the Government of Guam to provide for the education of dependents residing on military installations on the island. Through this arrangement DoD compensated the Guam Department of Education based on the number of allowable dependents attending the Guam public school system, amounting to approximately $13 million per year dedicated to personnel, education, facilities maintenance, and procurement.

    Recent problems with this arrangement, including an on-going lawsuit between the Governor of Guam and Guam's Department of Education and the failure of the Department of Education to provide proper accountability for expenditure of DoD funds, led to DoD withholding payments beginning in 1996.

    As a result of these on-going problems and in response to requests from the military departments, a fact-finding team from DoDEA visited Guam during May of 1997. This team concluded that no educational infrastructure existed within the Guam Department of Education to suggest that acceptable improvement was possible in the near future. In July of 1997, the Department formally established DDESS schools on Guam.
 Page 158       PREV PAGE       TOP OF DOC    Segment 1 Of 3  

    In September 1997, a central high school and two elementary schools began operation in hastily renovated facilities on Andersen AFB and the Naval Activity. The two military construction projects in the FY 1999 request are to construct additions to and complete renovations in the facilities serving as the central high school and the elementary school serving students from the southern end of the island.

    Question. When did the existing elementary and high schools commence operations?

    Answer. DDESS elementary schools began operation on both Andersen AFB and the Naval Activity, and the common high school on Nimitz Hill on Sept 29, 1997.

    Question. Will these two projects complete the construction requirement for establishing Department of Defense Dependent Schools on Guam?

    Answer. No. A military construction project is programmed for FY 2000 to construct a new 1800 student elementary school on Andersen AFB, estimated at $44.1 million. This will complete the construction requirement for establishing DDESS schools on Guam.

Fort Buchanan, Puerto Rico—Elementary School ($8,805,000)

    Question. To what extent is the requirement for this project driven by the Army's decision to relocate the headquarters of the U.S. Army South from Fort Clayton, Panama to Fort Buchanan, Puerto Rico, which was announced on July 31, 1997?
 Page 159       PREV PAGE       TOP OF DOC    Segment 1 Of 3  

    Answer. This project was required prior to the Army's decision to relocate US Army South to Ft. Buchanan, and will be required regardless of that move. This project will replace an existing 35 year old facility and substandard temporary facilities now in use.

    Question. Is this project a complete replacement for the existing elementary school?

    Answer. Yes.

    Question. Will the existing school be demolished upon completion of this project?

    Answer. The existing facility is on the real property records of Ft. Buchanan, and may continue to be used for some other function after it is vacated by DDESS.

NATIONAL SECURITY AGENCY

Fort Meade, Maryland—Fort Meade Perimeter Fence (West) ($668,000)

    Question. Why is the perimeter fence divided into two small projects, one in 1999 and one in 2000?

    Answer. During an OSD data call for PBD 098, DoD Force Protection and combating Terrorism, December 1996, NSA submitted an anti-terrorism proposal based on a phased approach to security protection, spread over a number of years, and with limited funding. NSA developed, and DoD approved, what was considered a viable program, spread over the out-years, at a ''reasonable'' cost. The fence projects are for separate parts of the Ft Meade complex with priority given to providing protection to activities not currently fenced. The PBD was submitted, congressional approval obtained, and the out-year program approved.
 Page 160       PREV PAGE       TOP OF DOC    Segment 1 Of 3  

    Question. How close will the west campus fence come to the cryptologic museum, and to the Baltimore-Washington Parkway?

    Answer. The fence will be some distance from the Parkway and Museum. The fence will be aligned on the ''outside'' (north side) of Canine Road and parking lots N–8 and N–9, and on Connector Road at the existing split in the roadway. The fence alignment will be 950 feet from the Museum and 1,500 feet away from the Parkway.

U.S. SPECIAL OPERATIONS COMMAND

Naval Amphibious Base Coronado, California—SOF Amphibious Operations Facility ($3,600,000)

    Question. What alternatives were studied under the economic analysis that has been prepared, and what were the findings?

    Answer. The economic analysis considered various options including taking no action, renovating and/or expanding existing facilities, and constructing new facilities. The recommended course of action was to renovate an existing building for storage of operational gear to construct a new structure for boat storage.

    Question. Will this project pay for itself in avoided maintenance and repair to boats, and so forth?

    Answer. The project is expected to recover its capital investment cost over the 25-year life of the facility. This is based on anticipated reduced maintenance and repair and extended life of the assets such as the Rubber Hulled Inflatable Boats valued at $505,000 per craft.
 Page 161       PREV PAGE       TOP OF DOC    Segment 1 Of 3  

Eglin Aux Field 3 (Duke Field), Florida—General Purpose ACFT Maintenance Shop ($2,210,000)

    Question. Describe in general terms the differences in maintenance requirements between the previously assigned AC–130A aircraft and the newly assigned MC–130 aircraft, to explain why the existing shops were marginally adequate for the previous aircraft but are inadequate for the new aircraft.

    Answer. The existing aircraft maintenance shop fails to meet current Air Force Occupational, Safety and Health (AFOSH 91–22 and 127–12) standards for industrial facilities. Specifically, minimum clearances cannot be maintained between equipment and machinery creating unsafe working conditions. Some shop stock is stored outside due to lack of storage space. The electronics shop, environmental shop and battery testing area are located in three difference facilities, causing span-of-work control problems. Industrial gas cylinders are not collocated with the shop where they are used. Key work centers are remote from other primary maintenance shops resulting in transportation problems and loss of productivity. The increased equipment requirements for the MC–130 Combat Talon's Avionics. Electronic warfare counter-measures, and other advanced features aggravate an already crowded situation.

Eglin Aux Field 9 (Hurlburt Field), Florida—Clear Water Aircraft Rinse ($2,400,000)

    Question. The form 1391 states that this project will have an investment payback period of approximately 2 years. Submit for the record a summary of this analysis that is based on a comparison of corrosion related aircraft maintenance requirements of rinsed and non-rinsed aircraft.
 Page 162       PREV PAGE       TOP OF DOC    Segment 1 Of 3  

    Answer. The payback period was determined from extrapolated data based on previously completed studies, i.e., the Draft Engineering Report, ''Study to evaluate the Effectiveness of Fresh Water Rinsing of Aircraft as an Aid in a program of Corrosion Prevention and Control at Myrtle Beach, S.C., ''prepared by Systems Exploration, Inc., Dayton Ohio, 31 August 1988, and Final Engineering Report, ''Effectiveness of Regular Fresh Water Rinsing of Air Craft at RAF Bentwaters and RAF Woodbridge, United Kingdom,'' Systems Exploration, Inc., Dayton, Ohio, May 1990. These reports validate the effectiveness of fresh water rinsing after each operation over salt water. Since Hurlburt field is located less than one mile from the Gulf of Mexico, the proposed clear water rinse facility will be especially effective in preventing corrosion.

Macdill AFB, Florida—Renovate Command and Control Facility ($8,400,000)

    Question. Will this project complete all headquarters construction requirements for the United States Special Operations Command?

    Answer. This project completes construction requirements for the administrative headquarters and will result in terminating temporary host base facility utilization and leasing. The Center for Acquisition and Logistics still has minor military construction requirements for a Development and Test Center and a Public Access Building.

Fort Campbell, Kentucky—Aircraft Maintenance Hangar ($15,000,000)

    Question. Why will this project provide one aircraft maintenance bay for contractor support, rather than including such space in the cost of the support contract?
 Page 163       PREV PAGE       TOP OF DOC    Segment 1 Of 3  

    Answer. The Unit's highly specialized SOF instrument helicopters require the logistics support contractor to provide maintenance inspections, depot-level maintenance, and research, development and testing on site, as well as to deploy with and support the aircraft conducting special operations. The dynamic advanced technical nature of the electronics work and the likelihood of contractor turnover in the future make it impractical for the support contractor to furnish facility workspace without adding considerable cost to the support contract and significantly diminishing contractor responsiveness.

    Question. What percent of the cost of this project is for such contractor space?

    Answer. Contractor space is estimated at 11 percent of the project cost.

NS Roosevelt Roads, Puerto Rico—SOF Operations Facility ($9,600,000)

    Question. What percent of the cost of this project is for space for temporary additional duty personnel, and what is the anticipated utilization rate of this space?

    Answer. Approximately 10 percent of the project cost supports the temporary additional duty personnel. One of the unit's primary missions is to train these personnel who regularly deploy for short duration to this location for specialized SOF training. Projected space utilization is 80–85 percent.

 Page 164       PREV PAGE       TOP OF DOC    Segment 1 Of 3  
    Question. Should some or all of the cost of this project be tallied as a cost of relocating SOUTHCOM from Panama?

    Answer. This project is intended to support Naval Special Warfare Unit FOUR's current mission at Naval Station Roosevelt Roads. None of the project cost is attributable to relocating SOUTHCOM from Panama.

Contingency Construction Various ($9,350,000)

    Question. The form 1390 for this program indicates the request for future years will be $10,000,000 per year through fiscal year 2002. How is this annual program level determined?

    Answer. The Department has historically requested approximately $10,000,000 per year.

Minor Construction Various ($16,094,000)

    Question. The form 1390 for this program indicates the request for future years will be a total of $90,000,000 for fiscal year 2002 through fiscal year 2002. How is this program level determined, and why is it nearly double the fiscal year 1999 level?

    Answer. Minor construction is budgeted based on historical experience and should represent an amount considered prudent to satisfy annual unforeseen minor construction requirements of a non-emergency nature.
 Page 165       PREV PAGE       TOP OF DOC    Segment 1 Of 3  

    Based on historical experience, the Defense agencies projected minor construction requirements of $29 million in FY 1999. The Department's request of $16 million will be supplemented with $13 million of unobligated funds which are available for this purpose. The FY 2000–2002 estimates assume that requirements will be approximately the same as those anticipated for FY 1999, adjusted for inflation. These estimates will be refined based on FY 1998/99 experience.

Planning and Design Various ($39,866,000)

    Question. The form 1390 for this program indicates the request for future years will be a total of $150,000,000 for fiscal year 2000 through fiscal year 2002. How is this program level determined, and why is it twenty-five percent greater than the fiscal year 1999 level?

    Answer: The Components estimate their Planning and Design (P&D) requirements for the budget year based on the size of the two succeeding fiscal year military construction programs. For example, the amount requested for FY 1999 will be used to complete design on FY 2000 projects and to initiate design on FY 2001 projects. Therefore, the size of the FY 1999 request is a function of the size of the FY 2000 and FY 2001 construction programs. The Components' requests for P&D in FY 1999 equate to approximately 4.5 percent of the FY 2000 and 4.5 percent of the FY 2001 military construction programs. However, in FY 1999, some of the agencies had unobligated balances that will be used to satisfy FY 1999 P&D requirements. As a result, their FY 2000–2002 P&D budgets appear to be larger because they assume that no unobligated balances will be available to offset requirements for those years. These estimates will continue to be refined.
 Page 166       PREV PAGE       TOP OF DOC    Segment 1 Of 3  

Family Housing Construction Improvements

    Question. The construction improvements account includes one small project to provide carports in Richmond, Virginia ($245,000), and three very small projects to provide: front doors in Menwith Hill, UK ($50,000); fencing in Lathrop, CA ($30,000); and shelters in Richmond, VA ($20,000). Why couldn't these be accomplished as maintenance projects rather than an improvement projects? What criteria are used for identifying individual projects within the total budget request for funding as improvements rather than as maintenance?

    Answer. The projects are not considered maintenance projects because they require installation, replacement and erection of new nondwelling structures. These projects are best classified as improvements projects as delineated in title 10 U.S.C. 2825.

OSD Guidance Regarding RPMA Definitions

    Question. Submit for the record the current guidance issued by OSD regarding the definition of work that is permissible under the Real Property Maintenance Accounts, as opposed to work that must be funded under the Military Construction appropriations bill.

    Answer. The Components were reminded that Section 2811 of Title 10 provides authority for the Department to carry out repair projects costing more than $5.0 million using operation and maintenance (O&M) funds provided that they are approved in advance by the Secretary concerned. In order to ensure that this authority is being applied in a consistent manner throughout the Department, we provided them with the following standard criteria for determining what constitutes a repair project and directed that these criteria be applied to all future projects.
 Page 167       PREV PAGE       TOP OF DOC    Segment 1 Of 3  

CRITERIA FOR A REPAIR PROJECT

    1. Repairs means ''to restore a real property facility, system or component to such a condition that it may effectively be used for its designated functional purpose.''

    2. When repairing a facility, the components of the facility may be repaired by replacement, and the replacement can be up to current standards or codes. For example, Heating, Ventilation, and Air Conditioning (HVAC) equipment can be repaired by replacement, can be state-of-the-art, and provide for more capacity than the original unit due to increased demand/standards. Interior rearrangements (except for load-bearing walls) and restoration of an existing facility to allow for effective use of existing space or to meet current building code requirements (for example, accessibility, health, safety, or environmental) may be included as repair.

    3. Additions, new facilities and functional conversions must be done as construction. Construction projects may be done concurrent with repair projects as long as the projects are complete and useable.
    "The Official Committee record contains additional material here."

    Question. For the rules issued by your agency between April 1, 1996, and December 31, 1997, please provide the amount of budgeted resources (professional and support staff, travel, meetings, conferences, etc.) dedicated to developing and issuing these rules, from proposed through final (list rules for each fiscal year in development with budget for that rule and a budget total for all rules for that fiscal year).
 Page 168       PREV PAGE       TOP OF DOC    Segment 1 Of 3  

    Answer. No Military Construction, Family Housing, or Base Realignment and Closure (BRAC) funds were used by DoD to develop and/or issue any of the rules issued by the Department for the period April 1, 1996, through December 31, 1997.

    Question. What, if any, procedural changes in the rule development process were made in response to the passage of the Congressional Review Act (CRA)?

    Answer. Public Law 104–121 was signed on 03/29/96. Since that time, the Defense Acquisition Regulations Council (DARC) has issued various non-major rules pertaining to Military Construction, Family Housing, or Base Realignment & Closure Accounts. Before any rule takes effect, the DARC complies with 5 U.S.C. 801, as added by Section 251 of Public Law 104–121. A report is submitted to each Houses of Congress, and the required information is submitted to the Comptroller General and made available to each House of Congress.

    Question. For which rules has the General Accounting Office prepared a report(s)? Has the GAO highlighted any concerns about your agency's compliance with the Congressional Review Act? If so, what procedures have you put in place to ensure hour agency's compliance with the CRA in the future?

    Answer. The General Accounting Office provides a report to the standing committees of jurisdiction of both Houses of Congress on major rules proposed by Federal agencies. None of the rules proposed by DoD effecting MILCON, Family Housing, or BRAC were classified as major rules; therefore, GAO did not prepare a report on any of them.

 Page 169       PREV PAGE       TOP OF DOC    Segment 1 Of 3  
    We are unaware of any concerns by GAO about compliance with the CRA.

Section 6 Schools

    Question. Is it correct that the Department of Defense has resolved the question of whether to continue to operate Section 6 schools, rather than returning these schools to the states?

    Answer. A final decision regarding the transfer of the Domestic Dependent Elementary and Secondary Schools, formerly known as Section 6 Schools, to location education agencies has not been made at this time.

    As you are aware, in 1986, Congress mandated that DoD develop a plan for the orderly transition of all Section 6 Schools to the LEAs by 1990. As part of developing that plan, the DoD conducted (through contract) detailed case studies of each DDESS district. In 1988, a final report of the studies was issued. It concluded that none of the existing DoD stateside schools could be transferred to the LEAs without substantial, and in many instances, continuing costs to the federal government. Impediments to transfers included the inability of the LEA to provide an education of the same high quality as that being provided by DoD, and the inability of the LEAs to financially support increased student enrollments resulting from the enrollment of the military base children. Additionally, the military communities perceived certain drawbacks to the transfer, including the loss of schools specifically geared to the needs of military children and the loss of their own school boards, as established in the DoD requirements. Many of the LEAs would not accept the existing facilities without significant improvements being made, similar to the situations now being corrected in the states of Alaska and Washington. The report noted that depending upon the avenue used to transfer a school to the LEA, requirements for facility improvements may, in fact, remain with the government. After review of the report, both DoD and key elements of the Congress concluded that transfer was inappropriate at that time.
 Page 170       PREV PAGE       TOP OF DOC    Segment 1 Of 3  

    In 1995, the Conference Report on the National Defense Authorization Act for Fiscal Year 1995, Public Law 103–337, requested that the Secretary of Defense collect information concerning the possibility of transferring the schools to LEAs. Unlike the 1988 report, in 1995 Congress requested surveys of parents and students, LEAs, and the leadership of affected military installations regarding transfer of the schools and asked the LEAs what financial, construction, and other support would be required to facilitate a transfer. The report also requested a survey of local public school districts in which over 30 percent of the students are military dependents to determine the level and sources of funding for these schools.

    Although the report is currently being coordinated with the Office of Management and Budget and other involved federal agencies, preliminary reports with regard to the possibility of a transfer are quite similar to those of the 1988 report. Specifically, it appears that although several factors are cited, the primary impediment to possible transfers is the inability of the LEAs to absorb additional students without financial assistance.

    [CLERK'S NOTE.—During March of 1998, the Department of Defense submitted the following Revised Executive Summary of the Base Realignment and Closure Accounts. The revision was necessary due to Navy revisions to ''Savings'' figures. These are no changes to ''Implementation Costs'', to ''Estimated Land Revenues'', or to any figures for components other than the Navy.

    The following Revised Executive Summary supersedes the information appearing in Part 4, pages 1 through 46.]
    "The Official Committee record contains additional material here."
 Page 171       PREV PAGE       TOP OF DOC    Segment 1 Of 3  

Thursday, March 12, 1998.

DEPARTMENT OF DEFENSE, INDUSTRIAL AFFAIRS AND INSTALLATIONS

WITNESS

JOHN B. GOODMAN, DEPUTY UNDER SECRETARY OF DEFENSE

Statement of the Chairman

    Mr. PACKARD. We understand that Mr. Hefner is on his way, and I am sure other members of the committee will be arriving. I think we will go ahead and begin, and they will certainly have some comments and some questions as we reach that point.

    I will call the hearing to order. We certainly are grateful to have you back with us again, Mr. Goodman.

    Mr. GOODMAN. Thank you, Mr. Chairman.

    Mr. PACKARD. This is certainly not your first time before this subcommittee, and hopefully it will not be your last.

    We are looking forward to your testimony. I have read it through. I was telling my clerk that it is kind of messed up in terms of the way I have marked it up and asked questions, because I was reading it while I was on the treadmill last night. [Laughter.]
 Page 172       PREV PAGE       TOP OF DOC    Segment 1 Of 3  

    That is not easy, to make notes while you are on the treadmill.

    But I do appreciate your submitting it to us, and it is well organized and well presented and we are looking forward to having you present your testimony. I would appreciate you summarizing perhaps and covering the highlights rather than reading the testimony to us. Most of us have read it.

    In our deliberations today, I hope that we will be able to get into some discussions about some specifics relative to the privatization program. We have been very strong advocates of the privatization program. And it did not initiate with me as chairman, but it has been an ongoing program to see how we can better fill the need on quality of life issues.

    And we think it is moving in the right direction, although we do have some concerns and we may want to address those concerns, and certainly they may come up in the question and answer period.

    Some of the concerns are we feel that we are quickly moving into large, large, large projects before we have really had a chance to test out the smaller projects that have been underway. Two or three hundred homes—we feel comfortable that that is a good test bed to determine if it is going to work. Two or three thousand homes we have some concerns about, at least this early. It may be that it will work out.

    We are going in almost an irreversible direction. It appears that in our efforts to move into privatization that we are moving rapidly—so rapidly that we may be scuddling the traditional way that we have built homes in the services in the past.
 Page 173       PREV PAGE       TOP OF DOC    Segment 1 Of 3  

    And in every group that we have had before us in our hearings this year, in answer to the question, ''Are we replacing the traditional way of building housing and quality of life facilities with the privatization?'' the answer has been no. But it appears that that may be where we are headed. I would be very anxious to hear your observations in that area.

    We also are delaying the appropriations and the actual funding or spending of monies, waiting for privatization to really get going. That is of some concern. Perhaps thirdly, and maybe more importantly of all of the other concerns, is that we are—I am not sure we see a very well-defined evaluation and analysis laid out to see if, in fact, this is the right way to go.

    I would much prefer to have to make changes in the program early on than to wait until we have had some failures or some real serious problems that arise. And it is very possible, in the new direction that we are going. We simply cannot see all of the eventualities. We do not know all of the nuances, and I am not sure we have a performance system well identified that will tell us if we are moving in the right direction, and tell us that soon enough to where we could make some changes.

    The construction industry, at best, is a precarious industry, and there are all kinds of possibilities for failures. Talk to any contractor, general contractor, and they will certainly tell you that there can be unforeseen problems. Financing may not work out always the way that it is planned.

    Contracts may not always work out the way that they were designed, and we would like to know if there has been some thinking and some directions in terms of what if this happens that does not make it all pencil out the way we have anticipated, where do we go from there, and is it the right direction.
 Page 174       PREV PAGE       TOP OF DOC    Segment 1 Of 3  

    Performance standards/performance evaluation I think is something that we are concerned about. I am. But overall, I do not want that to sound like we are negative on privatization because we are not. We think that it is a very positive, new approach to trying to close the gap.

    One of the concerns I have is is that the primary purposes of looking at privatization and moving in that direction was that it would close the gap much quicker, and we may not be closing it as quickly as we had hoped under current process. Secondly, that we might save money, and are not sure that that is happening. And, thirdly, that we would be able to transfer to our service men and women some ability to develop an equity.

    Those are three very fundamental purposes that we would like to see and evaluate whether, in fact, those purposes are going to be realized in this new approach. I have probably said a lot more than I intended to, but I simply wanted to let you know that even though this subcommittee, and certainly me as the chairman, are very supportive of the privatization efforts, I think there are some yellow lights out there that we would like to at least determine if it is going to work out the way we all hope that it will.

    And so with those comments—do either of you, Mr. Cramer or Mr. Olver, have any opening comments? If not, we will go ahead and have him formally——

    Mr. CRAMER. No opening statement.

    Mr. PACKARD [continuing]. Give his statement, and then we will just open it up for questions.
 Page 175       PREV PAGE       TOP OF DOC    Segment 1 Of 3  

    We are looking forward to your testimony. Again, we have it before us, and we have reviewed it, so if you would like to summarize. Cover it any way you would like, frankly.

STATEMENT OF THE HONORABLE JOHN B. GOODMAN

    Mr. GOODMAN. Thank you, Mr. Chairman.

    First of all, I would like to note that in our first meeting together you strongly expressed your support for privatization as a principle of government, that it was an important way of reducing the size of government and improving the quality of service that we can provide.

    My testimony today is about housing. But as we discussed last year, this year the Department is moving forward in other areas, in utilities, which was a recommendation of yours.

    I am pleased to be here today to talk about the housing privatization program. With your permission, I would like to submit my written statement for the record.

    Mr. PACKARD. Absolutely.

    Mr. GOODMAN. And in light of your comments, rather than simply summarizing my testimony, I would really rather address some of the questions and issues that you have raised, because as I have been reviewed this program in preparation for my testimony, I have also been looking into the kind of concerns that you have expressed, and I would like to talk to you just a little bit about them.
 Page 176       PREV PAGE       TOP OF DOC    Segment 1 Of 3  

    I do not need to tell anyone on this committee how important housing is to the quality of life, nor do I need to tell anyone on this committee how important good planning is to management. For these combined reasons, Deputy Secretary Hamre this last year established an important goal. Namely, he directed that the military departments eliminate our inventory of inadequate housing by the year 2010. That is a big job, because we have 300,000 houses, two-thirds of which are in desperate need of repair or replacement.

    We also, frankly, still have a deficit out there and could benefit from more houses. Using traditional procurement, it would cost us at least $20 billion. And given even previous years' funding levels, because I recognize the comment and your concerns about current funding level, but even in previous years' funding levels, that would take us about 30 years to solve. As you have noted, that is simply unacceptable.

    That is why this housing privatization initiative is so important to Secretary Cohen. The tools that you have provided, which we are now using in this five-year test period, provide the means to leverage our resources. With these tools we can get leverage of at least three to one. In some of our projects, we have found that that leverage rate is 30 to one. That means that we can get significant more housing for the dollars that you are providing than we could through our traditional tools.

    Now, I completely share and agree with earlier witnesses—I know Bill Lynn was here—who noted that the Department's goal is not to replace Milcon with privatization. It is an addition. It is an additional tool, and, frankly, it is too early to tell, as you have noted. But indications are that it certainly does provide the possibility to put our young men and women in uniform with more and better housing, cheaper and faster than would otherwise be possible. That is why the initiative is so important.
 Page 177       PREV PAGE       TOP OF DOC    Segment 1 Of 3  

    Now, we have made progress in our first two years, but it is a little less than we hoped, and I would like to explain why. First, the program was a new start. It required the Department to learn how to deal with the realistic community, with the financial community, to learn how to analyze the viability of projects. Until the Housing Revitalization Support Office gained experience, initial analyses were long.

    Now, part of the reason why this was important—and I testified to this last year—is that while we wanted to use these tools as much as possible, we only wanted to ramp up consistent with our ability to manage the program, with our ability to ensure the government's long-term financial interests, and with our ability to ensure the kind of management issues, Mr. Chairman, that you raised in your opening statement.

    The second cause of delay is that one year into the program we did not have approved scoring guidelines from the Office of Management and Budget. I talked a little bit about this last year. It was not until June last year that OMB established appropriate rules to score obligations to the government incurred by the use of these new authorities. That delay essentially put our work on hold.

    When the guidelines were established, they contained some rules that were different than had been incorporated into our initial analysis—how, for example, utilities should be treated in the financial calculations. That meant that we had to go back and redo the kind of detailed, pro forma analysis, the detailed life cycle cost analysis, that we thought was important to ensure that we were getting the most for our money and that we were able to protect the government's interest.
 Page 178       PREV PAGE       TOP OF DOC    Segment 1 Of 3  

    Third, developing the loan and loan guarantee instruments that you provided to us proved time consuming, because we needed to work with the financial community to translate the concept of loan guarantees into actual documents that would receive favorable—and by favorable I mean investment grade—financing.

    The last thing we wanted, of course, was to have projects that would only qualify for junk financing. Certainly, that was not our intention in this kind of project. These had to be projects. There had to be financial instruments that Standard and Poor's and others would consider of investment grade. We wanted their review. In some sense, we wanted their underwriting of the project to supplement our own to ensure that we are managing it well.

    Finally, and this leads into a point, Mr. Chairman, that you raised and that I want to address, there was some uncertainty at different levels in the services as to whether this initiative would work—uncertainty, frankly, that was compounded by the long deliberations over scoring rules.

    This uncertainty led the services to delay making decisions about whether or not to propose a project to the Housing Revitalization Support Office for privatization. The way we had set up the program, and it was very important to the services, is that the services would propose individual projects.

    In essence, my rule and my office's rule was to ensure that these projects were financially viable, to ensure that they were sound, that the management plan was sound, and then the implementation—the procurement process—was run by the services. What I am noting is that the proposals for the projects proved very slow in coming.
 Page 179       PREV PAGE       TOP OF DOC    Segment 1 Of 3  

    This goes to the third point I wanted to make, because the slow decisionmaking on whether or not to proceed with the projects on the part of the services delayed execution of projects and delayed execution and transfer of the Milcon authority.

    In preparation for this hearing, Mr. Chairman, members of this committee, I learned the extent of those delays. Some extend back to 1996. I know that you and other members of this committee are frustrated by this delay. I want to tell you, so am I. I was, frankly, surprised to learn how large this backlog was, and I appreciate the committee's highlighting this issue.

    As I noted, I have seen the projects when the proposals came forward, and that is the point at which I went to work. But I realize that there is a significant backlog here, and we need to do something about it. And I want to tell you what I am going to do.

    To correct this situation, next week I have called for a summit meeting, to include all key personnel from the services and from the Comptroller. I want to identify what money is being held up and why it is being held up. For each project, I will demand that the services, working with HRSO—that is the Housing Revitalization Support Office—establish a schedule for using the new authorities or for proceeding with Milcon.

    I want to and will share these schedules with this committee. Paralysis by analysis, or worse, indecision, simply cannot continue.

    Now, moving forward, Mr. Chairman, here is what we will do to prevent this kind of problem from recurring. There are a few actions that we are taking. First, it is important to note that HRSO is getting ahead of the curve in their site review process. Already, HRSO has conducted site analyses at more than 30 locations, and that number is growing steadily.
 Page 180       PREV PAGE       TOP OF DOC    Segment 1 Of 3  

    That means, unlike the past, we will not need to wait to determine whether a privatization project will work or not. Increasingly, we will have that information ahead of time.

    Now, in the past, the HRSO team, as I noted, conducted a site analysis and then awaited a service's decision to move forward. As I noted, in many cases that decision simply did not come. To prevent future delays, we are going to modify our procedures and establish a deadline for an up or down decision on the part of the services on how to proceed with that project.

    Finally, now that we are beginning to get ahead of the curve in terms of the kind of analyses that are required to figure out whether privatization projects will work, I think the services can and need to begin making privatization decisions in the budget planning process. In next year's budget bill, I will, therefore, recommend that each of the services specifically budget for these revitalization initiatives.

    In short, we will clear out the execution backlog, we will set procedures to prevent its recurrence, we will establish schedules that we will share with you, and we will begin the process of budgeting directly.

    Now, fourth, one of the reasons that this information about delayed execution is so frustrating to me is that I think in many other respects the program is making real progress, and I think that this committee and Congress should be very proud of that progress because it provides significant improvements in the quality of life for our young men and women in uniform and their families. It provides the kind of housing that we all know that they deserve and we all believe is absolutely critical to retaining the kind of skilled, motivated young folks that we have.
 Page 181       PREV PAGE       TOP OF DOC    Segment 1 Of 3  

    We have resolved the scoring issue. We have developed a trained Housing Revitalization Office. We have obtained and have under contract now leading financial and real estate firms to advise us and to do a lot of the—help us do a lot of the analysis and work with the real estate community.

    And I would note, Mr. Chairman, you had asked the question about our evaluation and analytical process. I would be delighted to share this approach and this material with the committee, because, frankly, I think it is very, very rigorous. We do detailed pro forma financial analysis. We do detailed life cycle analysis, all before—life cycle cost analysis, all before we decide to move ahead.

    Before the project moves forward, there are a number of things that I want to know. First, of course, is, does it meet the needs of the installation and the men and women in uniform? But from a financial perspective, is the project financially feasible? Will it save us money over the long run? Is there a management plan to ensure that we can take care of the needs of this project over the long run? These are all things that we look at.

    RFPs are focusing not just on getting a project out on the street, but ensuring that there is a good financial and management plan to ensure that even once it is built that the project is well managed and that we have procedures in place to ensure that the Commander at the base is able to have a significant influence over the direction and the upkeep of these projects.

    One of the great benefits of this program is that while I think we have done a good job of building houses, we, frankly—the Department, frankly, and services have not done a terribly good job taking care of the houses that we have. With our analysis in these projects and with what we are planning, we are requiring the projects to maintain reserves to ensure that the projects are kept up, to ensure that they are able to be maintained in ways that, frankly, we just have not done a very good job doing.
 Page 182       PREV PAGE       TOP OF DOC    Segment 1 Of 3  

    And I can come here every year and tell you it is a real concern to us and we want to do a better job. But the Department does not do as good a job as we should, you know, keeping reserves, just like a private sector—a real estate project would do to ensure that the project maintains value.

    Well, that is what we have been working on on the programmatic side. In terms of projects, we have completed two projects. We have two projects that we hope we can soon award at Fort Carson, Colorado, and Lackland Air Force Base, Texas. And we are moving forward on seven other projects.

    All together, these projects encompass 18,000 units. That is real leverage. Those are 18,000 new, replaced, or revitalized units that will provide the kind of housing, as I noted, that we all want to give our young men and women and their families.

    Now, in keeping with my earlier commitment, Mr. Chairman, I will provide this committee with copies of the specific schedules mutually agreed upon by my office, the Comptroller, and the services for these projects. And let me assure you, I will monitor closely their performance to ensure that those schedules are achieved, because I know you have a concern about this and so do I.

    The fifth point I want to raise is that I think it is important to keep in mind that these projects are not just collections of houses. They are communities. And that is important to us both in how we design the project, in how we prepare it, and in what we are asking for.
 Page 183       PREV PAGE       TOP OF DOC    Segment 1 Of 3  

    First, in the design of the projects, not only is the commander closely involved in this, but so, too, are the families now living on the base. Mr. Chairman, in our first meeting, you raised this issue to me, that you believed that we needed to have families involved in the decisionmaking process. I agreed completely.

    And in my approval at every project, I have made it a requirement that the RFP process, that the development of the concept of the project involve not just the top brass but involve service members and their families.

    And second, in the construction of the facilities, the projects involve more than just houses. We are requiring community centers, playgrounds, recreational facilities, and parks. Those are the kind of facilities where families can get together, where communities are built, all of which contribute to the quality of life and enable our families to enjoy the same both physical, cultural, and social amenities I think that are such a valuable part of the culture of our uniformed forces.

    But, in closing, Mr. Chairman, I want to thank this committee for its support. I share your frustration and have outlined a series of steps that will clear out the backlog in execution, and I believe will prevent its recurrence.

    We need to do that because this initiative is critical. It is critical to the Secretary, it is critical to all of the military departments, and most of all it is critical to provide quality housing to the young men and women in uniform and their families.

 Page 184       PREV PAGE       TOP OF DOC    Segment 1 Of 3  
    Thank you very much.

    [Prepared statement of the Honorable John B. Goodman follows:]
    "The Official Committee record contains additional material here."

    Mr. PACKARD. Thank you very much, Mr. Secretary.

    We appreciate Mr. Hefner being with us this morning. If you have any opening statement or any comments that you would like to make——

    Mr. HEFNER. Well, Mr. Chairman, I had an opening statement, but what I would prefer to do is ask a couple of questions, lengthy questions, I would like to just do that at the appropriate time, and just welcome Mr. Goodman to the committee.

    Mr. PACKARD. This may be the appropriate time. [Laughter.]

    I will recognize you first, and then we will go to Mr. Cramer, and then to Mr. Olver.

DELAYS IN HOUSING PRIVATIZATION

    Mr. HEFNER. Would you agree that housing projects that are already funded should go forward as intended by Congress, and they should not be held hostage while you work out the bugs in privatization?

 Page 185       PREV PAGE       TOP OF DOC    Segment 1 Of 3  
    We have heard from witnesses that you are not going to save money—but you are going to speed up the process. And, you know, all the houses are not inadequate, and there are many quality units that are being held up we think because of this.

    Let me just follow on along those lines. There is one housing project that was funded at Fort Bragg last year, and it has been held up while you develop your privatization methods. I was kind of surprised to hear this because the original project was very badly needed and privatization is going to delay relief for these families for several years.

    In March 1997, you testified before this committee that 42 sites had been identified as privatization candidates. We have got candidates, not necessarily that they will go, but they are candidates. In February of 1997, the DOD issued a revitalization of military housing report on the first year of the housing privatization initiative.

    Appendix C of that report is the site nomination list. Forty-three sites are listed as nominated, and I am assuming that 43—that is the list that you were referring to in your testimony. Well, among the nominated sites, you list one at Fort Bragg. Now, the report has no further details about the nomination.

    Next, I would like to call your attention to an FY1990 budget justification material submitted to the committee by the DOD and printed by us as part of the hearing record. Now, could you tell me how many of these projects that have been funded and are scheduled to go that are being held up so that you could work out the bugs on the privatization?

    And that is not a parochial thing for me. I feel sure that other members on this committee also would like to have that information. I do not think this is what we understood when we were funding these projects, that they were ready to go, and I do not think they should be held hostage until you work the bugs out of privatization.
 Page 186       PREV PAGE       TOP OF DOC    Segment 1 Of 3  

    Would you just set my mind at ease on that?

    Mr. GOODMAN. My Hefner, you have raised a number of questions. Let me try to address each one in turn. I think they are important questions.

    First, I just want to disagree with the characterization, or rather want to explain the characterization when people say we are not going to save money from these projects. We are going to save money from these projects. One of the conditions for approval, from my perspective, is that there is life cycle savings, one, and, two, that——

    Mr. HEFNER. Would you just hold there just a minute? We may have a vote here. That is a debate that is going to continue, whether you are going to save money or not. You know, we have gone through that argument on base closure.

    Out in the future, there may be savings, and what have you, but we understand the reason that we are doing privatization. It speeds up construction but that is not the question that I basically want answered. The question I want answered is: Are projects being held up until you can work out the bugs, and where are they, and how many of them are there?

    Mr. GOODMAN. Let me answer your question in two parts. The first part is that I think the view of the chiefs, the view of the military departments, is that when and where we can leverage our resources and get three to 30 times as much houses for our young men and women, that that is something that we ought to do.

 Page 187       PREV PAGE       TOP OF DOC    Segment 1 Of 3  
    Frankly, given the quality of life challenge we have, we have to do everything we can to speed revitalization of houses.

    Mr. HEFNER. Well, I understand——

    Mr. GOODMAN. Mr. Hefner, I am going to address the second part of your question. I am not avoiding it.

    Mr. HEFNER. I know you are not avoiding it.

    Mr. GOODMAN. I am not avoiding it.

    Mr. HEFNER. The point I am making is, it just does not make any sense to hold these projects that are ready to go, that people are waiting on. It is going to inconvenience them, until you get the bugs worked out. That is the whole thing.

    Mr. GOODMAN. The second point, and I may have passed this point in my oral statement before you arrived, is that as I prepared for this hearing, I became aware of the delays in the execution of projects. Honestly, sir, I was not aware of the extent of that because the services were the ones who were proposing projects to me.

    Let me clarify. Something was a proposed candidate, and that meant that the service was interested in looking at it. At that point, the HRSO team went out and actually did the kind of detailed analysis that would demonstrate that privatization was feasible. At that point, it went back to the military department for decision.
 Page 188       PREV PAGE       TOP OF DOC    Segment 1 Of 3  

    What I have learned as I have prepared for this hearing is the fact that there are '96 projects that are on hold, that there are '97 projects that are on hold, and there is a sizeable amount of money there. And that does concern me, sir. So what I had stated is that next week—I do not have all of the information on that, because that has not been part of my role in the process.

    Next week I have called a summit meeting with the Comptroller and the services, because I think we owe you—frankly, I think we owe it to ourselves and I think we owe you—a clear schedule of how we are going to proceed, and that we cannot continue simply to delay. We either have to be able to demonstrate to you that there is a clear project here, a clear privatization project with significant leverage that we can obtain, or we need to cut bait and move forward.

    I think it may be worth a short delay of the Milcon project to determine whether three to 30 times the number of housing units makes sense. It is not worth a long delay. In my oral statement—it is not in my written statement, Mr. Hefner, and I apologize for that. I prepared my oral statement last week and my written statement. As I was preparing for this, I learned new data that caused me great concern.

    I think a short delay may be acceptable; a long delay is not. And I have outlined in my oral statement a range of—a number of new procedures to be able to ensure that those delays are short rather than long.

    I know how important the issues are in Fort Bragg. I have traveled down there. I have gone running around the base. And right before he left command, General Keane called me and asked me for my support for the Fort Bragg privatization project, because in his view it was the only way that he was going to be able to ensure that we could take care of the needs of the young soldiers and their families.
 Page 189       PREV PAGE       TOP OF DOC    Segment 1 Of 3  

    So I am not happy, Mr. Hefner, about the delay there, and I will come back to you and this committee with a schedule about how we are going to proceed.

    [The information follows:]

    See attached letters to Chairman Packard dated March 23, 1998, and April 6, 1998.
    "The Official Committee record contains additional material here."

    Mr. HEFNER. Well, General Keane was in my office last week. We work with him very closely. All we want you to do is shoot straight with us, where we can know what we are doing. We have enough problems on this committee with funding to do the things that we need to do with limited funds.

    I would just certainly appreciate that when you give your testimony and we read the record that we find some discrepancies. Whether they are overt or just something you overlook, or what have you, but we need to have a good relationship here, because this is something—this privatization business I have real reservations about it, as I did 801 housing. And we want to get the most for the buck for our troops, but we do not want to give away too much and make too much of a sacrifice just to experiment.

    I would urge you to be very specific for this committee to know where we are going and what the real situation is.

 Page 190       PREV PAGE       TOP OF DOC    Segment 1 Of 3  
    With that, Mr. Chairman, I yield back my time.

    Mr. PACKARD. Thank you.

    Mr. Olver? No questions at this time?

    Let me go into a couple of things, then, as I read your testimony, and then I will get into some of the prepared questions.

LONG-TERM PRIVATIZATION AGREEMENTS

    I am looking for information more than questioning what you have said. On the projects that are in procurement—the Fort Carson, Colorado, project—this will be our largest project——

    Mr. GOODMAN. Yes, sir.

    Mr. PACKARD [continuing]. To date, 2,600 units. And in both of these projects that are in procurement now, Lackland and Fort Carson, it is anticipated that the housing allowance will cover the entire cost of the lease or rent?

    Mr. GOODMAN. Yes.

    Mr. PACKARD. And how have you projected that with some accuracy for a 50-year contract?
 Page 191       PREV PAGE       TOP OF DOC    Segment 1 Of 3  

    Mr. GOODMAN. We have used the normal escalator clauses, and the kind of cost-of-living adjustments in that, and we have provided that information to the bidders.

    Mr. PACKARD. Even builders that build or retain ownership of Section 8 housing in HUD, they run into unforeseen gyrations in the market, or it does not pencil out after, say, five years, because there is a depression or an increase in valuation or a variety of reasons why their projections did not fulfill themselves. And for 50 years that is really a difficult process.

    What would happen if, say, the owner, the contractor, or the owner if they sold it, came back to you 10 years from now, and this only a fifth into the 50-year contract and our lease, and said, ''The housing allowance is simply not covering the cost of maintaining and ownership and servicing the debt, and so forth. We cannot continue.'' What happens?

    Mr. GOODMAN. Mr. Chairman, we are assured—I mean, our—one of the things that we look at when we are doing our analysis is what is the debt coverage ratio? Are we able to ensure, with very conservative assumptions, that the debt is going to be covered?

    If the developer or the owner finds himself or herself unable to proceed or in default, they have a problem with their lender.

    Mr. PACKARD. Well, they do, but the problem is, we are the lender in some instances. In Lackland, of course, as you have outlined, the government will have some form of limited loan guarantee, but so we become the guarantee of the loan. And on the second mortgage, we become the mortgagor.
 Page 192       PREV PAGE       TOP OF DOC    Segment 1 Of 3  

    Mr. GOODMAN. Well, actually, we are not, neither at Lackland or at Fort Carson, are we guaranteeing the loan against economic risk. But the critical point is that we are projecting—we are certainly projecting forward what the anticipated escalation could be in terms of their cost.

    I mean, there are a couple of kinds of costs that can rise. We are asking them to budget for their reserves, we are going to be monitoring that process as they move forward, and we are overseeing this process as part of a management committee.

    Mr. PACKARD. I think the thing that I am hoping that we are able to do—and I am not sure how well we can do it on a long-term lease—as we have moved into this kind of an approach in a big way, but with multiple, multiple contracts, dozens and dozens of contracts, 30 or 40 contracts, and then find that for some reason the owner walks away, simply says, ''It is not working out for me, and I am going to walk away.''

    Then, we not only have one project; we could have multiple projects come back on us with some real dilemmas. And I am trying to evaluate and look ahead and see what those kinds of dilemmas are and what kind of problems would confront this committee, having to come up with unexpected monies, embarrassments at what we have planned and set up that simply did not work out. I am looking to avoid those kinds of problems.

    Mr. GOODMAN. Mr. Chairman, absolutely. First, I should note it is my understanding if there ever were a default it is not—it comes out of the—I believe it comes out of the Treasury, not out of the Defense budget, so I believe—I think you are off the hook.
 Page 193       PREV PAGE       TOP OF DOC    Segment 1 Of 3  

    Mr. PACKARD. No, not entirely.

    Mr. GOODMAN. But, Mr. Chairman——

    Mr. PACKARD. That is not a good answer. [Laughter.]

    Mr. GOODMAN. And, Mr. Chairman, that was not my complete answer.

    Frankly, I think the greater risk is during the construction period rather than during the management period, because where these projects for the developer will really pay off is by managing the project over the long run. That is where they are going to be getting their fees that justify the initial investment that——

    Mr. PACKARD. Is that where the loan guarantees are, in the construction phase?

    Mr. GOODMAN. The loan guarantees that we are providing are as limited as possible, and they are focusing on what I would characterize as political risk—namely, the risk that the base might close. If it were the case that a base would close, in certain communities then it might well be that there are not other people, other civilians can move into those houses.

    And that could trigger a default, and if it triggered a default that is what we are providing coverage for, because developers simply will not move into a community. The developers, Standard and Poor's, their underwriters, the banks, they can evaluate the economic risk. What they cannot evaluate, however, is the political risk, and that is what the guarantee is providing for.
 Page 194       PREV PAGE       TOP OF DOC    Segment 1 Of 3  

HOUSING ALLOWANCES

    Mr. PACKARD. Are you committing in your contracts all of the housing allowance? Are you committing all of the airmen or the soldiers' housing allowance in perpetuity, or at least for the 50-year contract? If we increased here the housing allowance 20 percent over the next five years, would that automatically go to the contractor, or is there some adjustment there? Is there some consideration?

    We are committing to the housing allowance as it now exists, but suppose that housing allowance varies either up or down, how is that written into the contract?

    Mr. GOODMAN. Mr. Chairman, that is a detail on which I am going to need——

    Mr. PACKARD. And I do not mean to micromanage.

    Mr. GOODMAN. No, not at all, sir. I think it is a perfectly reasonable question. Under our agreement, contract terms would then be renegotiated.

    Mr. PACKARD. So as the housing allowance changes, there is an opportunity to——

    Mr. GOODMAN. Yes.

 Page 195       PREV PAGE       TOP OF DOC    Segment 1 Of 3  
    Mr. PACKARD [continuing]. Renegotiate?

    Mr. GOODMAN. Yes. And, Mr. Chairman, one of the things that I have wanted to make sure that we have as well in our contract terms is that if the projects—you talked about the downside risk—I mean, obviously, the safe and conservative thing to look at.

    In addition, I think we need to look at the upside risk on the developer's side, because we certainly do not want to be creating huge windfall profits for the developer. And we want to make sure that if they are making—for no fault of their own are making a lot more money than was anticipated at signing, that we get more for it.

    Mr. PACKARD. I would hate to see us create a situation where our deliberations on whether we ought to increase or adjust the housing allowance is made on the basis of what kind of multiple contract you have out there that would create a windfall for a developer.

    Mr. GOODMAN. Well, and that is——

    Mr. PACKARD. And that would not be a good circumstance for us to put upon ourselves. We ought to be able to make those decisions based upon what is good for the servicemen and women, not on what is good for the contractors that may be affected by a housing allowance adjustment.

    Mr. GOODMAN. You are absolutely right, because 70 percent of our people are not living in government housing now—they live on the economy—and we need to not lose sight of those folks while we are just focusing on our housing project. I agree with you, and we are including that in our contractual terms.
 Page 196       PREV PAGE       TOP OF DOC    Segment 1 Of 3  

    Mr. HEFNER. Would you yield for just one quick comment?.

    Mr. PACKARD. Of course, Mr. Hefner.

50-YEAR CONTRACTS

    Mr. HEFNER. When you were putting together this 50-year contract, did you use a program that showed a floor on the monthly payments, or whatever? If you had a floor and it could not go below that could the contractor make his determination that it will not be less than this amount?

    Mr. GOODMAN. Certainly.

    Mr. HEFNER. Was that figured into the model?

    Mr. GOODMAN. Yes. Yes.

    Mr. HEFNER. Okay. I understand.

    Mr. GOODMAN. I mean, certainly, if Congress were to abolish, God forbid, housing allowances, that would be a serious problem for the projects. But I certainly do not think we anticipated that happening at all.

    Mr. HEFNER. Well, I have got to tell you that I have said this before. I would like to be a contractor and do nothing but build, if I was guaranteed full occupancy and I was guaranteed that I was going to make a profit. If at a certain time I did decide to go belly up, the lender is going to be responsible, and the lender is going to come which is the government, and we are going to pick up the tab. That is a pretty good deal. It is sort of the only risk free business that I can think of right now.
 Page 197       PREV PAGE       TOP OF DOC    Segment 1 Of 3  

    Mr. GOODMAN. Well, Mr. Hefner, let me just clarify that we are not guaranteeing occupancy, and that was part of—that was one of the rules that the Office of Management and Budget laid forward. Our families have the right of first refusal on the units, but we are not guaranteeing the developer that they will live there.

    Mr. HEFNER. Yes, but let us be realistic about this. If you are going to build 300 units on the base, as short as we are of houses, that is pretty much a guarantee that you are going to have full occupancy. I do not mean to be argumentative, but I have just got some concerns about it.

    Mr. Chairman, I do not want to belabor this.

    Mr. PACKARD. I want to get to Mr. Edwards as quickly as I can. Let me just finish up on this, though.

HOUSING ALLOWANCES

    Not only do we not wish to create a disparity and complicate the decisionmaking process of adjustments to the housing allowance, but are we creating a disparity—a greater disparity between families that have to live on the economy and families that can qualify for these new housing arrangements and have their allowances cover the entire cost? That is already a disparity that is of concern to this committee—those that have to live on the economy because we simply do not have the adequate housing available for their—that their housing allowance covers.
 Page 198       PREV PAGE       TOP OF DOC    Segment 1 Of 3  

    Are we creating a greater problem, or are we diminishing the problem? That is the question.

    Mr. GOODMAN. First, I share your view that that disparity is—that the disparity is a problem. We have not in this project—we have not seen privatization as the means to be able to resolve that. I think it is a very complicated problem, but——

    Mr. PACKARD. But does it exacerbate that?

    Mr. GOODMAN. No, sir, to the contrary. In fact, I think what it does is it simply ensures that people who are living on base housing or in government projects are living in adequate and quality housing. You know, we have had this sort of strange compromise in the past. You can either take your money and go out on the economy, or you can live on base, no out-of-pocket expense, but your house might be crumbling and not in very good condition.

    We certainly do not want that to happen. If anything, in a number of these projects—and, frankly, in some of the larger projects, we are able to significantly build down the deficit. That is part of the plan at Fort Bragg, part of the plan at Fort Carson, with 800 new houses, part at the plan of Food Hood, part of the plan at Pendleton.

    We could help more people who would like to live on base, either because of cost reasons or because of the kind of community reasons that motivate a number of members, especially in those forces where the service member deploys. So, no, I think this helps rather than hurts.
 Page 199       PREV PAGE       TOP OF DOC    Segment 1 Of 3  

    Mr. PACKARD. Thank you.

    Mr. Edwards.

HOUSING PRIVATIZATION—REASONS FOR IMPLEMENTATION

    Mr. EDWARDS. Thank you, Mr. Chairman. Personally, I want to thank you for calling the hearing. I happen to be one who strongly believes in trying this program, because I have seen the frustration of how long it would take through the traditional process to provide for adequate housing. People that are babies now will be, you know, 30 years old by the time housing is upgraded under the traditional process.

    But as a believer in the program, I think the tough questions you are asking, and Mr. Hefner, and the points he has made are very important. And I hope our committee can work with you, Mr. Secretary, to work out whatever problems might exist.

    I assume that, just like any major procurement program, in a way this is a new procurement program. Whether it is aircraft carriers or Air Force planes, you know, there are always start-up problems. My urging to you would be to encourage everybody within the Department of Defense to take bureaucratic hats off if necessary and be decisive, move, get some of the projects up and going, because a lot of these hypothetical questions we cannot—you cannot answer honestly until we have some projects on the ground.

    I also hope that where projects—traditional Milcon projects have been unnecessarily delayed because of this that you can work through that. And if you do not work through those problems, this program may come tumbling down before it is ever given a chance to succeed.
 Page 200       PREV PAGE       TOP OF DOC    Segment 1 Of 3  

    But because I was late—and I apologize for that; I had another hearing—could I ask you again to go back to the basic reason Congress put in place this program. Under traditional Milcon, what is the cycle for upgrading the inadequate housing at our military installations? And what will this system, if we can get it up and going more quickly and effectively, what will this do to that cycle?

    Mr. GOODMAN. With an inventory of 200,000 inadequate houses that need repair or replacement, with any kind of reasonably funding projection, it would cost at least $20 billion, current dollars, and take about 30 years to resolve that problem, more in some services, clearly more in some services, tougher for the Marine Corps, tougher for the Army than for the Navy or the Air Force, but still very tough.

    I think the reason Congress took this important and innovative step is that it recognized that as much as we would like to be able to solve—I am sure we all would have liked to have been able to solve the problem using our traditional procurement approach. We just were not going to get there.

    Mr. EDWARDS. You know, I think everyone on this committee would prefer to have more funding for this subcommittee, but it is not there. One issue——

    Mr. GOODMAN. Mr. Edwards, if I might. In some ways, I think this helps achieve a number of the other objectives of this committee, because I do believe that we are saving money. We are getting more good housing up front. That means that we will be able to devote more resources to barracks and the other kind of operational facilities that are very important that affect quality of life at the workplace, but where I am sure you all would agree we need to do a better job, too.
 Page 201       PREV PAGE       TOP OF DOC    Segment 1 Of 3  

    Mr. EDWARDS. Mr. Secretary, could I ask—you know, obviously, Fort Carson has taken longer than any of us, including yourself, would have wanted to take. There is a project at Fort Hood pending. I would have loved to have seen that happen more quickly. Having dealt with the problems you have dealt with now, for new projects starting today, you get the authorization to develop the project at installation XYZ, how long will it take from, you know, that initial approval to actually having the military family move in?

    Mr. GOODMAN. What we are working towards is having all of our site visits complete, so we know ahead of time, before money is authorized and appropriated, whether or not a privatization project will work at that particular base.

    If we have that done, what I stated in my oral statement is that rather than leaving the military department to prevaricate, and try to figure out whether or not they want to move forward in this way or that, we need to set a fixed deadline. I am not sure right now what that deadline is for a decision. I would like it to be short. This is one of the issues that I want to discuss next week with the services in their meeting.

    At the point where we then go ahead, an RFP needs to be written. That has taken probably about six months. I think that can be shortened. It certainly can be shortened with experience. Procurement process then takes about eight to nine months. So once the decision is made to move forward and to write the RFP, I think the entire award process is in the order of about 14—this is, please understand, off the top of my head—about 14, 15 months to award.

 Page 202       PREV PAGE       TOP OF DOC    Segment 1 Of 3  
    Mr. Edwards, what I had committed to the Chairman before you arrived is that—and I think Mr. Hefner emphasized this point and requested, if you will, that there be truth in advertising, and I think that is a very fair request. And what I want to be able to provide to you are, first, the specific schedules and dates of those projects that we have where we are moving forward.

    Frankly, in light of some of the delays that I have seen, I want to go back and see those scrubbed with the services, precisely, Mr. Hefner, because once we give them to you you have a right to expect that that is what we are going to do.

    And as I said, second, moving forward, I think we are moving into a position so this whole process will move much more rapidly.

    But, Mr. Edwards, if I might, just to—I know you have a special interest in Fort Hood. I just want to emphasize what the benefits are, because this committee appropriated funds for two Milcon projects that were worth about $40 million, and for that money would be able to buy 284 units, which is important.

    But using those funds, with the tools that you have provided us, will enable Fort Hood to immediately replace 700 existing units—immediately replace them—construct an additional 350 units, and a lot of those units are four-bedroom units that do not exist and that our young E–1s to E–4s simply cannot afford. They have large—I mean, they have large families. We need to provide them a quality of life.

    And in addition to those, there are a total of 1,000 new units. The project will renovate over 4,000 existing units on base. So for the same amount of funds that we could buy 285 units—our traditional tools—we will get 1,000 new units and 4,000 revitalized units.
 Page 203       PREV PAGE       TOP OF DOC    Segment 1 Of 3  

    And, moreover, we will have a budgeted process set up whereby the developer is going to be maintaining reserves so all of those units are going to be kept up over the life of the project. I could not guarantee that we would be able to do that for the 284 units that you have appropriated.

    Mr. EDWARDS. And those are some of the reasons that caused Commander Schwartz, General Schwartz, to become very enthusiastic about the program. I appreciate your working, you know, on not only that but this whole program. I would just urge you to urge the respective services to take off their bureaucratic hats, put on their entrepreneurial hats, and be decisive, move quickly. Do not, you know, try to make every project absolutely perfect before they agree to move on. Too much delay is going to sink this program, and it will not happen.

    But for the very numbers you mentioned, I am still strongly supportive of this, and hope with very fair and tough questions that the Chairman, Mr. Hefner, and others have asked, we can prove this program and not kill it.

    Thank you, Mr. Secretary.

    Thank you, Mr. Chairman.

CAMP PENDLETON, CALIFORNIA

    Mr. PACKARD. Thank you, Mr. Edwards.

 Page 204       PREV PAGE       TOP OF DOC    Segment 1 Of 3  
    Let me get back to your written testimony. On the Camp Pendleton thing, I simply for my own personal edification would like you to provide me with details on that project, where the 284 units are, the 512 existing units are, and just I would like some real detail on that because I am very personally acquainted with Camp Pendleton and it would help me just to have some details on that. There is no further question on that, however.

OMB SCORING GUIDELINES

    Mr. GOODMAN. Mr. Chairman, I would be delighted.

    Mr. PACKARD. Thank you. I would like you to outline the guidelines that you have addressed that OMB has approved in terms of scoring guidelines. Would you outline what those scoring guidelines are and what changes were made, if any, to accommodate this whole process of privatization?

    Mr. GOODMAN. Yes, sir. Most of our initial discussion with OMB focused on determining—figuring out how to determine whether there was enough private sector risk in the projects to benefit from scoring under the Credit Reform Act, and we came to an agreement on that.

    Now, under the Credit Reform Act, any direct investment in the project is scored as an upfront obligation. Any contractual agreement or guarantee is also scored as an upfront investment, or the net present value of the funds that are guaranteed. A rental guarantee would be scored as an upfront investment.

 Page 205       PREV PAGE       TOP OF DOC    Segment 1 Of 3  
    Loans and loan guarantees are scored at the subsidy rate, which seeks to estimate what the cost to the government would be from default. And that is why the use of loans and loan guarantee instruments gets us such greater leverage than we can by the other tools.

    One of the changes that I noted that has taken place is that OMB's—which I do not disagree with—is that all costs to the project ought to be included, one, in the analysis; and, two, paid—there ought not to be any hidden indirect subsidies by the Department of Defense. Or, to put it differently, a base ought not to be providing additional services that are not somehow paid for in the project.

    That particularly affected utilities, because in our initial two projects, in our initial projects we had not planned—we had planned on providing utilities directly to the project from the base. OMB rules say that those costs have to be picked up as part of the costs to the project. That meant that we have to go back and redo the numbers and make sure that that would work.

    I think the guidelines that OMB has put forward are fair. I think they ensure that the scoring are—that these projects are going to be scored in a way that actually accounts for what we are putting in or the risks that we are taking on.

    Mr. PACKARD. Can it permit you the leverage far more than what the guidelines before were?

    Mr. GOODMAN. Well, the problem before is that the guidelines kept moving, and we never had any agreement. You know operating in an environment of ad hocery makes it very hard to plan and hard to do the kind of detailed financial analysis that you have outlined is so important, and we agree.
 Page 206       PREV PAGE       TOP OF DOC    Segment 1 Of 3  

    Mr. PACKARD. Would you also provide for the committee a balanced plan—what your plan is in terms of balancing out your programs? Milcon, privatization, variable housing allowance—we would like to know if you have an actual plan that is including all of these instruments to close the gap, or is all of your energies and your efforts right now in the privatization area?

OVERALL FUNDING

    Mr. GOODMAN. Deputy Secretary Hamre directed, as part of his direction, that the services eliminate their inventory of inadequate housing by 2010, directed that they develop detailed installation-level plans on how they were going to achieve that objective. The services are now working on those plans. They are not complete.

    Mr. PACKARD. They are not complete.

    Mr. GOODMAN. When they are complete, Mr. Chairman, I would be pleased to share those with this committee, but it will not—I think——

    Mr. PACKARD. No, I am not asking you to go down a different track, then. If it is already underway, that is——

    Mr. GOODMAN. We have done this in the area of barracks.

    Mr. PACKARD. Do you have an idea of when they might be completed, or when they will be available?
 Page 207       PREV PAGE       TOP OF DOC    Segment 1 Of 3  

    Mr. GOODMAN. The direction was that they would be finished later this spring. As part of this meeting next week, I am going to get a status update of where we are at. But we want to know how they are planning to achieve that objective, or how at least at this point they think that they will be able to achieve that objective, and I want to share that with you, too.

    Mr. PACKARD. That will be fine. I was a little concerned to find that initially we had planned to have the backlog resolved with current funding levels in 10 years, and it has now stretched to 15. Why is that?

    Mr. GOODMAN. Do you mean why the 2010 deadline?

    Mr. PACKARD. It was originally, I think——

    Mr. GOODMAN. Oh, I see.

    Mr. PACKARD [continuing]. Earlier. I was wondering why we have now extended that.

    Mr. GOODMAN. Honestly, I do not think there is a strong analytical basis for the change, to be honest. I think when we had looked at the tools and looked at the kind of—we had said it would take 20 years—excuse me, $20 billion and 30 years to fix the problem. If you could get three to one leverage, you could do it in 10 years.

 Page 208       PREV PAGE       TOP OF DOC    Segment 1 Of 3  
    When Deputy Secretary Hamre looked at this, I think he liked the 2010 date. But, Mr. Chairman, I am sure your view—and mine is, too—that we ought to do this as fast as we can within the budget available to us.

    Mr. PACKARD. As you well know, we are realizing rather drastic cuts in these accounts, and that is a grave concern, over a third within the last two years, and with the emphasis on privatization. Why are we only requesting $7 million in administrative expense for the housing—for family housing improvement planning?

    Mr. GOODMAN. That is $7 million of the funds that are required for administration, undertaking the kind of detailed analyses to ensure that we are protecting our interests. That is the overhead costs of the program. The actual budgeting for the program until now has come, first, from the funds that you all provided, and, second, from the transfer of Milcon projects.

    But as I have noted, I think we are now at a point where we come forward with our budget request next year. We ought to be identifying specific projects for you that we believe are amenable to privatization, so that you have that information up front.

    Mr. PACKARD. Will that reduce the reliance on transfer authority?

    Mr. GOODMAN. I think we will—I cannot tell you at this point, since we have not worked this out with the Comptroller or with the services how best to identify those projects. One possibility might be to simply identify them. Another might be to create a separate program—a program line item within each service account for privatization. Another possibility would be to put it in a central account.
 Page 209       PREV PAGE       TOP OF DOC    Segment 1 Of 3  

    I have not worked that issue, Mr. Chairman, within the building. I do not know where we will come out, but I think we want to be able to identify for you up front, as I have noted, what projects we think should involve privatization and which should not.

    Mr. PACKARD. Who has any further questions?

HOUSING SHORTAGES

    Mr. HEFNER. I have just one. Previously at the authorization subcommittee on installations and facilities this week Mr. Paul Johnson and the Assistant Under Secretary of Army for Installations and Facilities, testified that at each of the bases where privatization occurs that this will eliminate the housing shortages. Do you agree with that statement?

    Mr. GOODMAN. Well, Mr. Hefner, I have only carefully looked at four Army housing projects that are part of the 18,000 that I mentioned—Fort Carson, Fort Hood, Fort Lewis, and Fort Stewart. In each of these projects, we are revitalizing existing housing and significantly buying down the deficit.

    I will need to go back and check if it completely eliminates the deficit in each of those cases. Clearly, one of my objectives in each project is that we are working to eliminate—to buy down that deficit to get more of our soldiers and their families in good housing. Let me take a close look at those projects for the record.

 Page 210       PREV PAGE       TOP OF DOC    Segment 1 Of 3  
    [The Information follows:]

    The Army's strategy is to eliminate the deficit for four and five bedroom rental requirements at each of its installations whenever possible. Private industry does not normally build four and five bedroom multi-family housing rental communities; however, it does build two and three bedroom rental properties. The Army consults with the local housing industry and local government housing authorities prior to developing its privatization project. These consultations generally result in an agreement that the privatization projects will concentrate on four and five bedroom requirements deficit reductions while leaving the two and three bedroom requirement to private developers. The Army has its Army Audit Agency validate its requirements as part of the privatization project approval process. The deficit may not be completely eliminated, the proposed projects represent the Army's and local communities' best approach to solving the installations' housing deficits with the available resources.

    Mr. HEFNER. Thank you, Mr. Chairman. That is all.

    Mr. PACKARD. Thank you.

    Mr. Edwards.

PRIVATIZATION AND IMPACT AID

    Mr. EDWARDS. Question. Mr. Secretary, this gets into other areas like Department of Education, but has anyone that you know of looked at the issue of the impact of this privatization on the impact aid funding? Will those students that are presently defined as type A students, therefore, getting much higher level of compensation through the impact aid program, will they continue to be defined as impact aid part A/type A students, or will they go to B?
 Page 211       PREV PAGE       TOP OF DOC    Segment 1 Of 3  

    Mr. GOODMAN. Let me answer your question—I am a little—I am not familiar enough with the program to—I understand type A/type B, but I do know how these kind of projects treat and will affect impact aid. Let me address that.

    Mr. EDWARDS. Okay.

    Mr. GOODMAN. When these projects are on base, if people are moving on base onto a project, provided that these are not DODIA schools, that will have an impact—that will affect impact aid, because, in essence, we are bringing more families onto government property and on the school system. If they are off-base projects, such as the one in Everett, Washington, then in instances it is neutral in that regard.

    Mr. EDWARDS. The property will still—the actual land, for example, is that still under the title of the United States Government?

    Mr. GOODMAN. Yes. We have the authority to actually transfer the land. In almost all of the cases that I can think of, the way in which we are dealing with that is to provide a ground lease, but we are maintaining jurisdiction.

    Mr. EDWARDS. One thing we might want to do is leave the Department of Education code to just be sure inadvertently we do not hurt bases. What you might end up doing in a number of cases if you are building a much larger quantity of housing units on post versus what has occurred in the past, then you are going to—this will be a tremendous help for the school districts——
 Page 212       PREV PAGE       TOP OF DOC    Segment 1 Of 3  

    Mr. GOODMAN. Local school districts.

    Mr. EDWARDS [continuing]. Around military installations. We might——

    Mr. GOODMAN. Well, the representatives from the school communities have come in to talk to us about that, because they were initially concerned about this.

    Mr. EDWARDS. Right.

    Mr. GOODMAN. I think we have allayed their concerns, and they believe the program is, if anything, beneficial.

    Mr. EDWARDS. Thank you, Mr. Chairman.

LONG TERM PLAN

    Mr. PACKARD. I think you detected a concern or perhaps a lack—and maybe there is that lack—it is not real, but certainly it is perceived by some of us on the committee—of a long-term housing strategy, housing plan. That is probably the thing that concerns me most right now.

    I sense that we are moving into these whole new authorities and the privatization effort on a project-by-project basis, analyzing them as we go into them as best we can. But I have not sensed a standardized methodology or a standardized long-term plan that would evaluate as we go along to see if we are going the right direction, and if we need to make some changes.
 Page 213       PREV PAGE       TOP OF DOC    Segment 1 Of 3  

    That I would appreciate your giving some thought. I would appreciate maybe giving some response back to the committee on if, in fact, you are developing or have developed a standardized methodology for comparing, for instance, Milcon projects versus privatized projects. I think we would have several of our concerns allayed if we saw some of that planning done.

    And then the long-term plan, I would hope that you have a long-term plan that would take all of your strategies for closing the gap into consideration, not only privatization but all of them—how many projects, how many homes you intend to build through privatization versus how many under the standard Milcon process.

    I guess I am one that likes to see a well-planned strategy, especially if we are moving into a whole new area. I just would take some greater confidence in seeking how you have mapped it out, how you plan to go back and evaluate the projects after they are finished, not necessarily before you go into them but after they are finished, to determine what we did wrong, what do we change, what should we change, where we can profit and learn from experience, even bad experience, because I do not expect everything to go smoothly in this process.

    I would be surprised if you expect that. There will be some glitches. There will be some unforeseen and unplanned problems. There will be some bankruptcy. There will be some poor maintenance. There will be some non-performance by the contractor that you are going to have to deal with.

    I sense that it is not going to go all smooth, and I would hope that we would do as best we can to plan for those uneventualities in it beforehand, but certainly review each project to determine where we can improve, where we can avoid problems in the future.
 Page 214       PREV PAGE       TOP OF DOC    Segment 1 Of 3  

    I would hate to have as my legacy to this subcommittee passing on to them 10 years from now a huge, humongous financial problem of how to deal with correcting problems that we did not foresee as we move into this process. And we are moving into it big time, and we are talking about big-time real estate ventures.

    There are not very many real estate ventures that are around this country that will be as big time into this kind of activities as we are moving. And even well thought out, well-planned, and well-legalized contracts can go belly up, can reverse and do the wrong thing.

    I would feel secure if you would provide the committee with what you are doing, what you have done, what you plan on doing, in terms of laying out a strategy and methodology, a review and analysis program that would start at the beginning to the aftermath. I am not sure we have done much since, if we have not done enough in that that we are dealing—we are so overwhelmed by the project-by-project approach that we may not be looking at the big picture, the vision, that will keep us out of trouble.

    Mr. GOODMAN. Mr. Chairman, I think those are important questions. They are important guidelines. I think we need to do two things. First, I think we need to provide you—and I guess I would—I like your characterization of projects and overall housing, project-by-project and overall housing plan.

    I agree with you that the Department has not done the kind of job it needs to do in terms of laying out an overall housing plan. That is one of the directions that Deputy Secretary Hamre has made. The services are working on it. Because I know that a combination—that Milcon and privatization will need to work in conjunction.
 Page 215       PREV PAGE       TOP OF DOC    Segment 1 Of 3  

    The project, for example, at Robbins Air Force Base is one where the initial project that came to me was one that did a very good job of taking care of the senior grades and the young officers. And the junior enlisted were left out in the cold, and I said that is simply unacceptable.

    What we worked out was a project where Milcon—we need to present this probably a little ahead of the Department, because we need to present this to you—but where Milcon was going to ensure that the houses were going to be adequate and appropriate for the junior enlisted, and that privatization would then be able to address some of these other needs, so that we would address the entire needs of the installation and not leave the junior enlisted in the cold.

    I think we are doing—as much as I think we need to do a better job in our long- term planning, we have begun that in barracks. We need to do it in houses. I would go even further, Mr. Chairman. I think we need to do that for our overall facilities and installations, and I have initiated a process to do that. It is broader, and, therefore, it is harder, but it makes it no less important.

    Mr. PACKARD. I totally agree with you. Do you have any underground facilities at——

    Mr. GOODMAN. Yes, sir. But on the specific projects, I think we have done a better job than we have thus far demonstrated to you. If you will, I think we have a far better analytical process, a far better—I think we have an established analytical process. I think——
 Page 216       PREV PAGE       TOP OF DOC    Segment 1 Of 3  

    Mr. PACKARD. Then, you need to educate us.

    Mr. GOODMAN. I will take that as a tasking, Mr. Chairman.

    I think we have already instituted a lessons learned process, both in terms of how we are writing the RFPs, what the issues are. I think you are right there will be glitches along the way. And just as you do not want to leave as a legacy any enormous catastrophes for your successors, I do not want to leave for my successor, or, worse yet, be the person here at the table explaining why this occurred.

    At the same point, I sure would not want to be the person here from the Department in 2010 explaining to the chairman of this subcommittee that, unfortunately, there just is not enough money in our housing budget to take care of the housing needs of our young men and women. And we really think quality of life is important. Is not it a shame?

    I think you have given us the tools that will enable us both to feel better in 2010, and I think we need to work with you to help ensure that together, as Congressman Edwards mentioned, we can make this program very successful.

    Mr. PACKARD. Thank you very much.

    Do you have any further questions?

    Mr. EDWARDS. No.
 Page 217       PREV PAGE       TOP OF DOC    Segment 1 Of 3  

    Mr. PACKARD. Mr. Hefner?

    Mr. HEFNER. No.

    Mr. PACKARD. I think you have said it well. I would like to leave the legacy where we solved the problem of housing in 10 to 15 years from now and not a legacy of heartache.

    I do not want to close a hearing on a note of being overly cautious. We encourage you to move forward. We are, frankly, pleased that you are moving in the direction that you are moving. But we simply feel that caution helps us do a better job of what our goal is, and we certainly want to be informed as to what changes you are making in your plans and in your programs, how they are working, and we are not going to try and micromanage, but at the same time we are certainly going to be interested in how you proceed and how you progress, and where your successes and your failures are.

    Mr. GOODMAN. Mr. Chairman, I take that as a tasking. I have told you the steps that we are going to undertake. We will provide that information to you, and I am going to come back to you to give you more information about the ways in which we have addressed many of the concerns that you have laid out.

    [The information follows:]

    See attached letters to Chairman Packard dated March 23, 1998, and April 6, 1998.
 Page 218       PREV PAGE       TOP OF DOC    Segment 1 Of 3  
    "The Official Committee record contains additional material here."

    Mr. PACKARD. The bells have given us the signal to close this hearing.

    [CLERK'S NOTE.—Questions for the record submitted by Chairman Packard.]

OVERALL FUNDING

    Question. DOD is on record that its initial goal was to use the new authorities to solve the services' housing problems with current funding levels in 10 years. The goal is now 15 years. What has happened to change the ''current funding'' scenario? Why is there a lack of commitment on the Department's part toward the traditional construction program?

    Answer. When the Department initially sought these new privatization authorities, we projected that use of these authorities would allow us to leverage our funds at a ratio of at least 3:1. Maintaining current funding levels would therefore allow us to solve a 30 year problem in 10 years. Our experience after the first two years of testing these authorities confirms that we can achieve a 3:1 leverage and often much higher. However these authorities are not the answer at all locations and must be combined with traditional Military Construction to solve our total housing problem. The original timeline would have presumed using all family housing construction funds for privatization.

    It is important to remember that housing privatization is still a pilot program. Our traditional construction program remains an important tool in providing quality housing for our military families and the great majority of our housing projects are still implemented using traditional military construction.
 Page 219       PREV PAGE       TOP OF DOC    Segment 1 Of 3  

    The Deputy Secretary of Defense established a goal for the Services to fix their inventories of inadequate family housing by the year 2010, hence the 15 year timeline. We expect to archive this goal through a combination of privatization, traditional construction and, when appropriate, demolition. As noted during my testimony, there have been some delays in the Services' normal family housing construction programs while they decided which method was most appropriate. We remain committed to carefully investigating the opportunity to significantly leverage our funds through our privatization authorities, however, we will not let our existing family housing Milcon be paralyzed by our privatization analysis. The winners will be our service members and their families.

    Question. With such drastic reductions to these accounts, one third of prior year spending, and the emphasis put on privatization, why are you only requesting $7 million in administrative expenses for the Family Housing Improvement Fund and relying on transfer authority for the program?

    Answer. As we test our authorities and determine the best ways of using them and which locations are appropriate for their application, we continue to program for traditional family housing MilCon projects. This method, in conjunction with our transfer authority, allows us to do privatization where appropriate while preserving our ability to continue using traditional military construction where necessary. As I noted in my testimony, we expect to propose an improved method of identifying and budgeting for privatization projects when we submit our FY2000/2001 budget next year.

    Question. At what point do you envision DOD will be ready to properly budget for this program versus relying on transfer authority?
 Page 220       PREV PAGE       TOP OF DOC    Segment 1 Of 3  

    Answer. As I noted in my testimony, we are developing an improved method of identifying and budgeting for privatization projects as part of the Department's FY 2000/2001 budget preparation next year.

SAVINGS FROM PRIVATIZATION

    Question. Does DOD expect that housing projects financed through the privatization initiative will be less costly to the government, and if so, why?

    Answer. Yes. In order to be approved for privatization, projects must leverage our initial investment at least 3:1 and also demonstrate life cycle costs which do not exceed comparable military construction alternatives. Our first projects indicate that privatization does save the government money over the life of the project. The savings in upfront privatization costs compared to building houses using traditional Milcon normally outweighs any expenses incurred by providing allowances as compared to the cost of operating and maintaining the housing ourselves. In addition to life cycle savings, privatization also enables the Department to solve its housing problem much faster than it could through military construction.

    Question. We have received testimony from each of the services that they do not see privatization as being less costly. That it helps to leverage additional housing, but is not saving money. To a large degree we will see a shift in funding from housing constructions and operations and maintenance accounts to military personnel accounts to pay for increased housing allowances. Is this budgeted for and factored into the Future Year Defense Plan?

 Page 221       PREV PAGE       TOP OF DOC    Segment 1 Of 3  
    Answer. Yes. As noted earlier, we also realize life cycle savings from privatization projects.

COST COMPARISON METHODOLOGY

    Question. Has the Department developed a standardized methodology for comparing the government's long-term costs for a housing project financed with traditional military construction funds and with the privatization authorities. Describe the methodology. If there is no standardized methodology in place, why not?

    Answer. No. The Department does not currently have a standardized methodology. Part of the purpose of our being a pilot program is so that we can test and learn the best ways of operating our program. Currently, the Department requires all costs to the government, as opposed to DOD only, to be compared in both military construction and privatization options of comparable scope and term. The factors to be compared are common and include housing allowances, residual personnel costs, and budget scoring for the privatization option; and construction, revitalization and operations/maintenance costs for the military construction option. School impact fees are also considered under both options. The HRSO is currently adapting Service procedures to a standardized model.
    "The Official Committee record contains additional material here."

    Question. Where will the privatization take place and what will the cost be?

    Answer. The Services have nominated nearly 70 sites from housing privatization. The following 11 sites totaling 24,149 housing units, have approved concept plans and have either been procured or have solicitations being developed. Their status is listed as follows:

Table 29


 Page 222       PREV PAGE       TOP OF DOC    Segment 1 Of 3  

    The costs of these projects will be determined by scoring. Sufficient appropriations must be available to cover the amount obligated for each contract. The Department, with OMB concurrence, will determine the amount of funds to be obligated to cover future costs that are associated with the use of the tools provided in the Military Housing Privatization Initiative. These amounts are not finalized until contract award.

LONG TERM PLAN

    Question. Has DOD prepared a written plan describing the key steps and milestones of the privatization initiative that are envisioned to solve the services' housing problems? If so, describe the plan and submit the written plan for the record; and if not, why not?

    Answer. Based on our experience and lessons learned, DOD hopes to expand privatization to reach 24,149 by FY99, and at least 30,000 by FY 2000. Beginning in 2001, DOD expects to be able to privatize at a steady level each year, as required to meet the Deputy Secretary's 2010 goal. The Military Departments are currently preparing detailed written housing plans, to be used in the FY 2000/2001 program review. The Department has set a goal of revitalizing, divesting through privatization or demolishing inadequate family housing, consisting of approximately 200,000 units, by 2010. The program review will determine how much of that goal will be achieved by privatization.

WELL PLANNED OVERALL HOUSING STRATEGY

 Page 223       PREV PAGE       TOP OF DOC    Segment 1 Of 3  
    Question. It is becoming obvious that DOD is lacking a well planned strategy that integrates all available tools to address the housing problem in an optimum manner. And, within the Department the tools are managed separately—one dealing with allowances/compensation and the other with housing policy. How does DOD ensure that all tools (existing civilian housing, enhanced housing referral services, adequate housing allowances, traditional construction, and, now privatization) are optimally used in an integrated manner to solve the housing crisis?

    Answer. DOD integrates these different housing tools through the planning, programming and budgeting system, as it does all its many, varied functions. Currently, as part of the upcoming program review, the Services are preparing detailed plans to integrate use of DOD's various construction tools in dealing with our inadequate housing by 2010. Additionally, outside the PPBS process, joint working groups address issues such as how allowances, local market housing, and improved referral interact in meeting the overall housing needs of our military families. These working groups are comprised of members from all segments of DOD.

BUDGET TOOL

    Question. It concerns me that the Department may be using housing privatization as a budget tool as the expense of the service members and their families, rather than as a program tool to provide suitable, quality, affordable housing for them.

What assurances do you have for the Committee that the major goal of providing better quality housing for our service members is still the number one priority?

 Page 224       PREV PAGE       TOP OF DOC    Segment 1 Of 3  
    Answer. The Secretary of Defense is committed to improving the quality of life for service members and their families. For this reason, the Deputy Secretary directed that the Military Departments eliminate inadequate housing by the year 2010. The goal of housing privatization remains leveraging private capital to fix our inventory of inadequate housing sooner than possible through traditional military construction. Our military families deserve quality housing and DOD's priority is to provide it as soon as possible. An example of this commitment is contained in the notification letter for Fort Carson (attached at Answer 57) which shows that, due to leveraging, more than $6 million will be left over from the original family housing appropriation. This amount will remain in the Family Housing Improvement Fund to be used for other family housing privatization projects. This is in addition to providing more than 2600 housing units at Fort Carson.

OVERSIGHT

    Question. What happens, as is the case of the Bridge Point Landing Development in Texas, when families become frustrated if the developer does not provide the promised level of quality housing and service? Who will hold the developers accountable for fulfilling the provisions of the contracts?

    Answer. The base commander through the base housing office will coordinate with the Southern Division of Naval Facilities Engineering Command to determine whether the promised services are being performed. Southern Division has the oversight/administrative responsibility to ensure that the terms and conditions of the Limited Partnership Agreement are adhered to and, if not, to take appropriate action. The Navy receives quarterly operating reports which show how much of the Operating Expense Budget has been expended during the reporting period to provide services which the developer/operator/manager agreed to provide. The scope of services is described in the Management Plan which was incorporated into the Limited Partnership Agreement. The reports are submitted to the Southern Division.
 Page 225       PREV PAGE       TOP OF DOC    Segment 1 Of 3  

SUCCESS OF FAMILY HOUSING PRIVATIZATION

    Question. What standards will be used to measure the success of the housing privatization program?

    Answer. Success of the housing privatization program is measured on a number of different levels. As a program DOD has set an interim goal of privatizing 30,000 units by the year 2000, while determining how much of its inadequate housing will be fixed with this important tool. At a project level, privatization is successful when it meets our leveraging criteria of 3:1 had has life cycle costs at least equal to military construction alternatives. But most importantly, housing privatization is successful when our service members can choose to live in quality affordable housing provided through our new authorities.

UNACCOMPANIED PERSONNEL HOUSING PRIVATIZATION

    Question. What is the status of privitizing unaccompanied personnel housing?

    Answer. Current scoring guidelines from the Office of Management and Budget (OMB) make it prohibitive. Under OMB scoring guidelines, if DOD assigns service members to housing units, the rental payments for the life of the contract are scored fully upfront. Military commanders feel strongly that they must have the ability to assign junior enlisted personnel to unaccompanied housing units.

 Page 226       PREV PAGE       TOP OF DOC    Segment 1 Of 3  
    Question. If privatization will not be used for barracks, is the $5 million appropriated into the fund needed?

    Answer. We are continuing to discuss scoring issues with OMB and still hope to be able to use these funds.

IMPLEMENTATION

    Question. Two years have passed since the new authorities were signed into law, yet no new agreements have been finalized to build or renovate military housing (Fort Carson—units are pending). What are the key reasons for the slow progress?

    Answer. We have made progress in our first two years, but less than we hoped. Primary reasons for the slow progress are:

    DOD had to learn how to deal with the real estate and financial communities, and how to analyze the viability of projects. Until the HRSO gained experience, initial analyses took too long.

    One year into the program we did not have approved scoring guidelines from the Office of Management and Budget. That delay essentially put our work on hold. Once guidelines were established, we had to go back and redo the detailed pro forma analysis to account for changes from the old rules.

    We only wanted to ramp up consistent with our ability to manage the program and ensure the government's long-term financial interests.
 Page 227       PREV PAGE       TOP OF DOC    Segment 1 Of 3  

    Developing loan and loan guarantee instruments was time consuming, because we needed to work with the financial community to translate the concept of loan guarantees into actual documents that would receive favorable, investment grade, financing.

LONG-TERM PRIVATIZATION AGREEMENTS

    Question. How is the Department determining an installation's future mission, military population and family housing requirement when considering a 50-year privatization project?

    Answer. While individual bases may gain or lose missions and associated military units over a 50 year period, the essential core mission of bases will remain essentially the same. Building housing using traditional military construction faces essentially the same challenge of predicting future requirements.

CONTRACTOR PERFORMANCE

    Question. How will DOD ensure that a contractor performs housing management, repairs, maintenance, and improvements in accordance with long-term (50 year) privatization agreements?

    Answer. The structure of each deal determines the contract mechanisms used to oversee contractor performance. Contract management plans and ground leases provide for contract performance over time. Depending on the financial structure of the deal we may also have loan documents, loan guarantees and intercreditor agreements. Each deal will specifically design these mechanisms to work together to provide adequate DOD controls. Also, DOD will require the contractor to include funding in contingency escrow accounts, and DOD will monitor on site maintenance.
 Page 228       PREV PAGE       TOP OF DOC    Segment 1 Of 3  

    Finally, these are basically private housing developments and the primary enforcement factor is the ability of the military members to vote with their feet and leave.

    Question. How will DOD enforce these agreements? Won't it be time consuming and costly?

    Answer. No. The structure of each deal determines the contract mechanisms used to oversee contractor performance. Contract management plans and ground leases provide for contract performance over time. Depending on the financial structure of the deal we may also have loan documents, loan guarantees and intercreditor agreements. Each deal will specifically design these mechanisms to work together to provide adequate DOD controls. Also, DOD will require the contractor to include funding in contingency escrow accounts, and DOD will monitor on site maintenance.

    Finally, these are basically private housing developments and the primary enforcement factor is the ability of the military members to vote with their feet and leave.

50-YEAR CONTRACTS

    Question. Under current authorities, family housing privatization involves government contribution of land, facilities, infrastructure, mortgage guarantees, and differential lease payments to developers and financiers. Wouldn't it be prudent to gain some experience with how well this program works, before making such a large commitment to turn over so many assets for a 50-year term?
 Page 229       PREV PAGE       TOP OF DOC    Segment 1 Of 3  

    Answer. The Services and OSD are working closely and cautiously to find the best ways to implement these new privatization authorities. As we test the authorities, we find that long term, large scale deals offer some of the most powerful leveraging. We are bringing the best private sector expertise to the table to help us apply the authorities wisely as we test these deals. Balancing caution with expeditious improvement of housing for our service members is the challenge we are striving to meet.

PERMANENT AUTHORITIES

    Question. The privatization authorities provide for a five-year test of the initiative. Is it likely that DOD will have enough experience by the end of the five years to recommend that the authorities be made permanent; or, is it more likely that additional time will be needed to continue to test the usefulness of the authorities?

    Answer. We expect to be able to make a recommendation regarding the extension of the authorities next year.

HOUSING ALLOWANCES

    Question. Explain the new housing allowances program DOD began implementing in January 1998.

    Answer. The Basic Allowance for Housing (BAH) is a single monthly payment being phased in that replaces the Variable Housing Allowance (VHA) and Basic Allowance for Quarters (BAQ). The intent is to provide housing compensation based on comparable civilian housing costs that considers both salary and location.
 Page 230       PREV PAGE       TOP OF DOC    Segment 1 Of 3  

    The BAH is designed so that all of the service members of a given grade and family size will have the same monthly out-of-pocket for the same level of housing as they do now. Lower pay grades will have lower out-of-pocket cost than senior grades.

    BAH is based on civilian standards, considering the housing choices made by civilians of comparable income in each location.

    Question. How will this program affect the need for on-base housing, and what is the potential impact from the new allowance program on the privatization initiative?

    Answer. The intent of this program is to reduce the requirement for on-base housing and the new allowance program should financially improve our deals.

SEPARATE ORGANIZATIONS

    Question. DOD has separate organizations responsible for managing the two key components of the military housing program—allowances and government housing. How do these organizations work together and coordinate on matters relating to housing strategy to ensure optimum integration of all initiatives?

    Answer. The DOD housing program is built from the Military Services through the planning, programming and budgeting system, which integrates the varied tools used to provide housing for our service members. The Services are currently preparing detailed plans, for use in the upcoming program review process, to integrate use of DOD's various construction tools in dealing with our inadequate housing by 2010. Additionally, outside the PPBS process, joint working groups, comprised of representatives from all segments of DOD, address the interaction of allowances, local market housing and improved referral in meeting our housing needs. The different DOD organizations coordinate on a regular basis optimize integration of all initiatives.
 Page 231       PREV PAGE       TOP OF DOC    Segment 1 Of 3  

BUILDING PRACTICES

    Question. Under privatization, developers are allowed to use local building practices and standards to construct or revitalize housing available to military members. In what ways could privatized housing be different from that housing built using military construction standards and specifications? Also, how are such differences factored into a cost analysis comparing privatization and military construction alternatives?

    Answer. Under privatization construction costs are born by the developer with the government being responsible primarily for allowances paid to the potential renters. Under military construction alternatives, the government pays the entire construction cost and operations and maintenance costs for the life of the housing. The economic analysis compares costs to the government and, therefore, the differences in construction costs are not directly compared. We require a minimum quality standard in privatization although not the detailed military specifications of military construction. The ability to get more housing through privatization is the primary advantage of this program.

CIVILIANS

    Question. One justification for on-base housing is to provide a secure, military housing community available to members and their families. How will this objective be affected if civilians begin to live on-base?

    Answer. We expect there is little likelihood of civilians living on base is expected to be minimal, primarily because the privatization projects are all targeted in areas that have a great need for housing for military personnel. Our standard of security would be not different no matter who lives there.
 Page 232       PREV PAGE       TOP OF DOC    Segment 1 Of 3  

    Question. What are the concerns associated with civilians renting units located on a military installation, and how is DOD addressing these concerns?

    Answer. Our standards of security will not change. Secure areas for military operations will remain secure.

IMPACT AID

    Question. If housing is not built on federal land, the local school district will lose Impact Aid funding. Are leaders of surrounding communities, especially school superintendents, being consulted early enough in the process to provide input on the project's impact on the entire community?

    Answer. Leaders from surrounding communities are contacted about the projects during the site visits which take place early in the privatization process. Most of our on base projects envision long term leases of government land, in which case the level of impact aid remains the same.

HOUSING REFERRAL OFFICE

    Question. The Navy has adopted an aggressive approach to help families find housing in local communities. I have visited the housing referral office in San Diego—it is quite impressive. What efforts are the other services undertaking to implement enhanced referral service?
 Page 233       PREV PAGE       TOP OF DOC    Segment 1 Of 3  

    Answer. Each Service focuses improvements to referral services where need is greatest. The Navy has emphasized its referral services in San Diego and Norfolk primarily because of the high concentration of service members and the presence of a robust private sector market.

INSTALLATION RELUCTANCE

    Question. Have there been any instances where an installation has been told to do a privatization project when the base is reluctant to do so?

    Answer. I am not aware of any such instances.

HOUSE REVITALIZATION SUPPORT OFFICE

    Question. What are the long range plans for the House Revitalization Support Office?

    Answer. The Department will reevaluate the role of HRSO at the point when the authorities become permanent.

USE OF BASE REALIGNMENT AND CLOSURE FUNDS

    Question. Why is the Department seeking authority to allow the transfer of Base Closure funds into the Family Housing Improvement Fund?
 Page 234       PREV PAGE       TOP OF DOC    Segment 1 Of 3  

    Answer. Privatization on receiving installations from BRAC may be the most cost effective way of housing incoming servicemembers and their families. The Department wants to have that option where it makes sense.

    Question. Is there any prohibition on using direct appropriations to the Family Housing Improvement Fund at BRAC locations?

    Answer. No.

    Question. Have the services and DOD determined any specific locations where the transfer of BRAC funds would be useful?

    Answer. No.

OMB SCORING

    Question. Provide for the record a detailed description of the scoring guidelines approved for the Office of Management and Budget in June of 1997.

    Answer. See attached letter from OMB Director Franklin Raines describing OMB Scoring Guidelines.
    "The Official Committee record contains additional material here."

FAMILY HOUSING PRIVATIZATION
 Page 235       PREV PAGE       TOP OF DOC    Segment 1 Of 3  

INITIAL PROJECTS AT EVERETT AND CORPUS CHRISTI

    Last year, the Navy executed two limited partnership agreements under earlier statutory authorities, at Naval Air Station Corpus Christi, Texas and at Naval Station Everett, Washington.

    Question. The project at Everett, Washington was designed for occupancy by E–5 military personnel. Information available to the Committee indicates that as of January, 1998, 39 percent of the units were occupied by E–6 and above. Are you satisfied with this performance?

    Answer. The Navy is pleased with the performance of the Public Private Venture project at Everett. Its first two Public Private Venture projects at Everett and Corpus Christi/Kingsville were a significant breakthrough in addressing our housing problems within the financial constraints facing the Navy and the Department of Defense. Enlisted Sailors occupy the 185 Country Manor homes at Smokey Point (Everett). A survey of the Sailors and their families at Country Manor found that residents liked the location, floor plan and amenities provided in the homes.

    Question. At Corpus Christi/Kingsville, Texas, 12 percent of the units are occupied by civilians, and 67 percent of the units are occupied by officers. At Corpus Christi/Portland, 20 percent of the units are occupied by civilians and 32 percent are occupied by E–6 and above and by officers. Are you satisfied with this performance?

 Page 236       PREV PAGE       TOP OF DOC    Segment 1 Of 3  
    Answer. The Navy is pleased with the overall performance of the public private venture projects at Kingsville and Corpus Christi. However, there are two concerns.

    First is the preponderance of officers at the Kingsville PPV, where junior officers occupy about 59 percent of the homes. The project targeted rental homes to be affordable for an E5 or E6 with dependents. Nearly all the officers who moved into the Kingsville PPV homes are junior officers (0–1 and 0–2) who earn a Basic Allowance for Housing virtually identical to the E–5 and E–6 pay grades.

    Second is the concern that some of the Portland homes at Corpus Christi are still not filled, and that civilians fill 25 percent of those homes that are occupied. Obviously, Sailors have not been as motivated to move in at this location as they have been at Everett. The Navy believes this is due to a number of factors. The adverse publicity generated during the unfortunate flooding incident, and the fact that the homes at Portland require a larger out-of-pocket expense than other locations are two possible examples. In addition, the project opened for occupancy when Permanent Change of Station moves were minimal, which meant there were fewer sailors who could easily move in. Since sailors cannot reasonably be expected to be willing to get out of their leases and move children out of one school district to another when the project was ready for occupancy, the supply of sailors was insufficient at the initial occupancy window. Therefore, civilians were given short term, one year leases, and it is expected that the percentage of military occupants will greatly increase in one to two years. To avoid this situation in the future, the Navy is considering coordinating the timing of Permanent Change of Station orders with obtaining a building occupancy permit for future PPV projects.

    Question. If there is a misjudgment regarding the target population for which a privatization project is designed, compared with the actual occupants, what can be done to make it right after the fact?
 Page 237       PREV PAGE       TOP OF DOC    Segment 1 Of 3  

    Answer. As we worked through this with the Navy at locations where this has been a problem, there are generally three ways we can adapt:

    (1) Review referral priorities for ways to target the needs of specific paygrades.

    (2)If referral does not work, develop additional housing units for paygrades which remain in need of affordable housing for a given geographic area.

    (3) Consider use of differential lease payments where housing exists but is unaffordable.
    "The Official Committee record contains additional material here."

FAMILY HOUSING IMPROVEMENT FUND

STATUS OF FUNDS

    Question. We have appropriated $47,000,000 to the Family Housing Improvement Fund, and an additional $5,900,000 has been transferred into the Fund, and no projects have been executed under the 1996 authorities. What has been accomplished to date with the funds that have been provided for the Family Housing Improvement Funds?

    Answer. The money has been used to fund project development, for consultant support, and some has been committed to the two Navy limited partnerships. Although NAS Corpus Christi, Texas and NAVSTA Everett, Washington projects were both initiated under the 1995 authorities, they were technically signed under the 1996 Act. A detailed account is as follows:
 Page 238       PREV PAGE       TOP OF DOC    Segment 1 Of 3  

Table 30


    Question. Submit for the record a detailed history and the current status of funds for the Family Housing Improvement Fund, showing the amounts appropriated to date, amounts transferred into the Fund to date, the amounts committed, obligated, and expended to date, and cumulative obligations by object class and subobject class. This detailed history should include, at a minimum, displays of how much has been expended for facilities, equipment, travel, compensation of federal employees, and consultant services.

    Answer. A detailed account is as follows:

Table 31


    No federal employee compensation is funded from the Family Housing Improvement Fund.

    Question. Provide for the record an object class and subobject class display of planned expenditures of the $7,000,000 included in the budget request for fiscal year 1999.

    Answer. This is how we break down the funds we are requesting:

Table 32


    Question. Provide for the record a table that will display the full-time equivalent staff-years, as well as the personnel compensation and benefits, for all Housing Revitalization Support Office personnel for fiscal years 1998 and 1999.

    Answer. HRSO Salaries for all civilian employees are as follows. They are not paid from the Family Housing Improvement Fund.

Table 33


 Page 239       PREV PAGE       TOP OF DOC    Segment 1 Of 3  

    Question. Provide for the record a table that will display the full-time equivalent staff-years, as well as the personnel compensation and benefits, for all personnel of the Military Services assigned to the Housing Revitalization Support Office for fiscal year 1998 and 1999.

    Answer. HRSO Salaries for all military servicemembers are as follows. They are not paid from the Family Housing Improvement Fund.

Table 34



FORT CARSON, COLORADO

BACKGROUND

    The privatization proposal for Fort Carson calls for a whole-base contract in which the developer will (1) revitalize the existing 1,824 on-base family housing units that are conveyed to the developer, (2) construct 840 new family housing units on the installation, (3) own, operate, and maintain all units for 50 years. The developer is expected to invest about $220 million to initially construct and revitalize the units and will recoup this cost, as well as the operating and maintenance costs, from the rents paid by the occupants over the term of the agreement. Military families have first preference in renting the units and will pay rent equal to the member's housing allowance. If military families do not rent the units, vacancies can be rented to civilians. DOD is providing a loan guarantee to cover the risks of base closure, deployment, and downsizing. In the event of a base closure default, the government will be obligated to pay off the loan and assume ownership of the project for disposal.
 Page 240       PREV PAGE       TOP OF DOC    Segment 1 Of 3  

    DOD prepared an economic analysis comparing the long term, or life cycle, costs of the proposed privatization project at Fort Carson with the costs to perform the same project using traditional military construction (MilCon) financing. Under traditional MilCon financing of military housing, DOD pays the initial housing construction or renovation costs, as well as the annual costs to operate, maintain, and manage the units. The Military does not pay monthly housing allowances to the occupants, since occupants of government-owned housing forfeit their allowances.

    Under the proposed privatization alternative, DOD will convey the existing housing units to the developer, set aside funds to cover the value of a loan guarantee, and begin paying monthly housing allowance to the service members who occupy the housing, since the housing is no longer government-owned. DOD estimated that the value of the 1,824 units to be conveyed is $27.6 million and that the cost of the loan guarantee is $9.6 million. In addition, under the proposed project, DOD will continue to pay utilities for military families occupying the housing, and some housing management costs for member referral services and for contract oversight.

    The DOD economic analysis estimated that the proposed privatization alternative would cost the government $639 million (the net present value of all costs over the 50-year term of the agreement). In comparison, the analysis estimated that the government's costs over the same period would be $861 million if traditional MilCon funds were used to finance the project. Thus, the analysis reported that the privatization alternative was about $223 million less costly than the MilCon alternative.

ECONOMIC ANALYSIS
 Page 241       PREV PAGE       TOP OF DOC    Segment 1 Of 3  

    Question. Under the MilCon alternative, the economic analysis assumed that operation and maintenance costs would be $8,658 per unit. Yet, the Army's fiscal year 1999 budget estimate for family housing operations and maintenance costs is $6,073 per unit. Considering that all housing at Fort Carson will be either newly constructed or newly revitalized, what is the basis for the higher estimate for operations and maintenance costs? Also, if the lower amount is more appropriate, how would its use change the economic analysis?

    Answer. The economic analysis is a comparison between military ownership and operation and private ownership and operation as proposed by the selected bidder. The Milcon maintenance calculation is based on an engineering estimate of the government cost to maintain the units in an appropriate manner to meet the bidder's construction and revitalization plan. (The government's utilities costs were included in the Milcon calculation but inadvertently not in the privatization calculation. This error has been corrected.) The results of the revised economic analysis do not change the least cost alternative, privatization. The difference between the engineering estimate and the FY 99 budget estimate occurs because the budget estimate does not fund the cost of maintenance and repair to the standard the contract calls for. Also the budget estimate is an average for all installations in the United States, while the engineering estimate is specifically for Ft. Carson. The Army's O&M estimate does not provide for the same level of operations and maintenance at the level the contractor would. It would take approximately 120% of the sustainment M&R plus major repair and revitalization construction funds to raise the dwelling units to contemporary standards and a 35 year life cycle. The contractor's proposal obtains these goals.

    Question. The economic analysis states that because utilities are provided by the government in each alternative, utility costs are the same in each alternative and are not included in the analysis. Yet, under the MilCon alternative, operations and maintenance costs are included as a cost to the government, and utility costs are included in operations and maintenance costs. No estimate for the cost of utilities is included under the privatization alternative. To be consistent, should an estimate for utilities costs be added to the privatization alternative? Also, if adding an estimate for utilities is appropriate, what would be the impact on the economic analysis?
 Page 242       PREV PAGE       TOP OF DOC    Segment 1 Of 3  

    Answer. There was an error in the economic analysis by including utilities costs in the Milcon alternative. The error has been corrected and the results of the analysis have been corrected. Although savings from privatization are less, it is still cheaper than MILCON.

    Question. Under the privatization alternative, the economic analysis assumes that housing management cost are $300,000 annually. This amount is considerably less than the amount the Army budgets for housing management. What is the basis for the $300,000 estimate for annual housing management costs? More specifically, (1) what functions currently performed by the local housing management office will be eliminated, and (2) does the $300,000 estimate include costs for monitoring contractor performance under the agreement and for providing referral services for members living in the local community?

    Answer. Under the contract, the contractor assumes ownership of the housing transferred from the government and the newly constructed units. The contractor is responsible for all costs of operating the housing with the exception of the cost of utilities. This greatly reduces the government cost of providing this housing to our service members. The only family housing responsibilities that will remain at Ft. Carson are the Community Housing Relocation and Referral Service (CHRRS) and the administration of the proposed contract. There are no operation and maintenance functions left once the property is transferred to the contractor. The $300,000 is the estimated cost of the installation housing portion of the cost to administer the contract and perform the CHRRS function. The cost roughly equates to five person years of effort.

    Question. The economic analysis states that the value of the 1.824 family housing units conveyed to the developer is $27.6 million, or about $15,132 per unit. What is the basis for this estimate?
 Page 243       PREV PAGE       TOP OF DOC    Segment 1 Of 3  

    Answer. The basis for the estimate was calculated from the commercial rental market value estimation method. The method takes the net cash flow of the project and projects its value from a multi-housing rental market basis. Since the rents of the housing are restricted to the value of military housing allowances and the housing resides on Federal land, the property has limited commercial value as a rental property with little or no resale value. Therefore the value of the property being transferred to the private sector is limited to its income potential. The $27.6 million is the estimate of that value using a standard commercial rate of return.

    Question. The existing units are valued at $15,132 per unit, and the operations and maintenance cost per unit is calculated at an optimum of $8,658 per unit or an actual of $6,073 per unit. Does that mean that the Army is expecting the contractor to spend approximately half of the value of an existing unit on annual operations and maintenance for that unit?

    Answer. No. These values are what it would cost the government to maintain the standard. The estimated cost of operations and maintenance for the contractor was based on actual costs for similar housing units in the Colorado Springs rental market. The real value of the transferred property lies in the project's net operating income. The $27.6 million value is a book keeping way to value the transfer of the government property to the contractor.

    Question. If so, what is the breakout of the several components of this operations and maintenance cost?

 Page 244       PREV PAGE       TOP OF DOC    Segment 1 Of 3  
    Answer. The breakout of the contractor's O&M costs remains proprietary information at this time.

    Question. In January 1998, DOD began implementing a new housing allowance program. The new program, which will be phased in over six years, is expected to better match the allowance amount with actual housing costs in each geographic area. Under the new program, civilian housing could become more affordable to military families, especially in high-cost housing areas. How will the new allowance program affect the proposed privatization project at Fort Carson, in particular in the calculation of the housing deficit? Also, to what extent does the economic analysis consider the impact from the new allowance program?

    Answer. The BAH for Ft. Carson is 4.8% higher than the FY97 BAQ/VHA costs which were used in the analysis. The difference does not alter the outcome of the analysis. There will still remain a deficit at Fort Carson.

50-YEAR TERM OF THE PROPOSAL

    Question. The Fort Carson privatization proposal involves a 50-year agreement with an option to extend the agreement for an additional 25 years. However, most installation forecasts of family housing needs over a period of only three to five years. In view of possible changes over time to the installation's mission, its military population, and the availability of civilian housing in the local community, please describe in detail the process and the factors used to determine this 50 year housing requirement at Fort Carson.

    Answer. The mix in the project housing between bedroom count and income level generally reflects the current grade mix of the service members assigned to Ft. Carson. There is a heavier weighting to the junior enlisted. Ft. Carson could experience a significant reduction in troop strength and grade mix without affecting the need for the project housing. It is unlikely that such a radical change in the mix and need of service member requirements would render the project housing unsuitable to meet the needs of the military families. In addition the Ft. Carson project allows other DOD military and DOD civilian personnel in the Ft. Carson area to occupy the project housing in the event there are insufficient military personnel assigned to the base. Lastly, on the lowest priority basis, local civilians will be allowed to occupy the project housing if there are no higher DOD priority personnel available.
 Page 245       PREV PAGE       TOP OF DOC    Segment 1 Of 3  

    Question. Ensuring adequate contractor performance is a concern in most housing agreements. DOD is attempting to ensure that the contractor will perform needed housing repairs, maintenance, and improvements by including maintenance standards, modernization schedules, and required escrow accounts as part of the agreement. However, it appears that enforcing these agreements could be difficult, time consuming, and costly. Specifically, how does the agreement deal with these issues?

    Answer. The contract includes a number of mechanisms to ensure contractor performance in maintaining quality housing. The contractor must maintain reserves and must meet minimum requirements regarding upkeep and repair response times. The Base engineer will monitor housing and the remaining housing referral staff will provide the ability for residents to register complaints not resolved by the contractor. The Army has the ability to tap the reserves if necessary to address legitimate problems in emergency situations. Lastly, this is private housing and soldiers can vote with their feet if housing is not maintained. Any default due to lack of performance puts developer at risk, as government can remove the developer.

    Question. Government-owned housing is provided at Fort Carson because the local community cannot meet the military's family housing needs. And, under the proposed agreement, military occupants of privatized housing will pay rent equal to their housing allowance. In this situation, it appears that the contractor is reasonably assured of having full occupancy at fixed rental rates; and could increase profits by skimping on maintenance and repairs and by cutting costs by hiring less qualified managers and staff. Is DOD confident that repairs and maintenance will be performed in a manner that does not erode the quality of life of the military occupants?
 Page 246       PREV PAGE       TOP OF DOC    Segment 1 Of 3  

    Answer. Yes. The contractor is required to maintain adequate reserves which can be tapped if he fails to maintain the property to the standards of the contract.

    Question. The proposed privatization agreement allows for civilians to rent housing units not rented by military families. Over the 50-year term of the agreement, it appears that the possibility of civilians living on the installation could at some time become a reality. The possibility of civilians living on the installation raises several questions relating to property tax liability and jurisdiction over domestic disputes. What are the concerns associated with civilians living on the installation and how does DOD propose to deal with each of these concerns?

    Answer. It is the responsibility of the Developer to take into account the tax consequences of the project and deal with the Federal and local taxing authorities. The government has no liability for taxes levied against the property or the income derived from the property. The ground lease issued to the developer retains exclusive Federal jurisdiction. Civilians occupying the housing subject to the ground lease will be subject to Federal law and jurisdiction.

AWARD CANCELLATION

    [CLERK'S NOTE.—On April 22, 1998, the Department of the Army canceled the proposed award of the whole-installation capitol venture initiative project at Fort Carson, Colorado. This contract would have been the first exercise of the authority sought by the Department of Defense and enacted in the National Defense Authorization Act for Fiscal Year 1996 on February 10, 1996 (section 2801 of Public Law 104–106, 10 USC 2871). The Army's decision was based upon litigation instituted in the U.S. Court of Federal Claims, and has resulted in a re-examination of the acquisition process. The Army is now studying corrective action alternatives, including a return to best and final offers and re-solicitation.
 Page 247       PREV PAGE       TOP OF DOC    Segment 1 Of 3  

CONGRESSIONAL NOTIFICATIONS

    Question. Submit for the record the Congressional notifications that have been submitted for the projects at Fort Carson, Colorado, Lackland AFB, Texas, and Marine Corps Logistics Base Albany, Georgia.

    Answer. See notification letters on pages immediately following:
    Offset folios 660 to 688 insert here

PUBLIC-PRIVATE VENTURES IN FAMILY HOUSING

    Question. Under current law, can the Department transfer Family Housing funds to the Military Personnel accounts?

    Answer. No. We can not move money after appropriation.

    Question. If not, what execution difficulties can you anticipate?

    Answer. The Military Departments must estimate Family Housing O&M requirements and military personnel allowances based on projected contract award timelines, as part of their budget preparation Predicting when privatization will occur is difficult and without transfer authority, housing costs may be budgeted in the wrong account. For instance, the Army anticipated privatization of Fort Carson would occur near the end of FY 97 and moved $15 million of Family Housing O&M funds to the Military Personnel account during formulation of their FY 98 budget. However, privatization of Fort Carson housing has not yet occurred and the Department has no authority to transfer the funds back to family housing, resulting in $15 million of housing requirements not budgeted in the account bearing the cost. With authority to transfer funds from family housing to military personnel accounts, funds can be moved at the actual time privatization occurs, thus avoiding having funds in the wrong account when budget assumptions for privatization time lines are different from when privatization actually occurs.
 Page 248       PREV PAGE       TOP OF DOC    Segment 1 Of 3  

    Question. In the preparation of this year's budget materials, have there been ''transfers in the estimates'' for fiscal year 1999, moving funds out of the family housing accounts and into the military personnel accounts?

    Answer. Yes. As compared with 1999 column of the 1998 request, the Navy moved $3.88 million from family housing O&M accounts to the Military Personnel appropriation. The Army and Air Force did not move any funds from Family Housing to Military Personnel.

    Question. If so, what amounts from which accounts?

    Answer. The $3.88 million was moved from the following Navy family housing O&M accounts:

Table 35



    Question. Submit for the record the FY 99–03 Defense Planning Guidance requirement with regard to using privatization as a tool to meet family housing needs.

    Answer. The Department established 2010 as its goal for eliminating, as quickly as possible, DOD's inadequate family housing units to provide better living conditions for our servicemembers and their families. The available methods for attaining this goal are revitalization, divestiture through privatization, or demolition. Additionally, the savings realized from leveraging private sector capital should be reinvested, in order to fully take advantage of its potential.

 Page 249       PREV PAGE       TOP OF DOC    Segment 1 Of 3  
    Question. Current statutory authorities expire in February of 2001. Are Departmental efforts being paced with this ''sunset'' date in mind?

    Answer. Yes.

FAMILY HOUSING PRIVATIZATION

THROUGH THE FAMILY HOUSING IMPROVEMENT FUND

[''PUBLIC-PRIVATE VENTURE'' PROGRAM, OR ''PPV'']

[''CAPITAL VENTURE INITIATIVE'' PROGRAM, OR CVI'']

    Question. The Committee has received the notification for the Army's family housing privatization project at Fort Carson, and it is under review. The Navy has executed two projects in Texas and Washington under somewhat different authorities. We have gone into detail about the ongoing privatization efforts during each of our hearings with the Services. I have several questions that I will ask you to answer from the perspective of the Office of the Secretary of Defense:

    Does the Department anticipate that there will be any significant financial savings from privatization as compared with the traditional programs?

    Answer: Yes.

 Page 250       PREV PAGE       TOP OF DOC    Segment 1 Of 3  
    Question. To what extent is privatization becoming a substitute for the traditional family housing program, rather than the supplemental program that Congress originally intended?

    Answer. Privatization is still a supplemental program under the test authorities provided by Congress.

    Question. Is it correct that the cost of the family housing program is being shifted from this bill to the housing allowances subaccounts within the military personnel accounts in the National Security appropriations bill?

    Answer. Only in cases where existing base housing is privatized must the Military Departments predict additional allowances in their programming process. In these cases their programming would also reflect the reduced requirement in the O&M accounts. In many cases housing allowances which are already being paid are used to leverage better housing for our military families, as in the example of the first two Navy projects.

    Question. Is this a ''zero-sum'' game, that is, will there be any savings or is it just a shift of costs?

    Answer. In order to be approved for privatization, projects must leverage our initial investment at least 3:1 and also demonstrate life cycle costs which do not exceed comparable military construction alternatives. Since we can solve our housing problem through privatization much sooner than through traditional military construction, the primary advantage is speed rather than life cycle savings. However, our first projects indicate that privatization also saves the government money over the life of the project as well.
 Page 251       PREV PAGE       TOP OF DOC    Segment 1 Of 3  

    Question. Is there any assumption about the future funding levels for housing allowances?

    Answer. Yes. When we model a project, allowances are increased along with inflation.

    Question. Are the Services or the Department holding up the execution of family housing projects for which funds have been appropriated, pending privatization efforts?

    Answer. Yes. (See attached letter of April 6, 1998 to Chairman Packard on pages immediately following this one.)
     "The Official Committee record contains additional material here."

    Question. Are these funds being withheld to meet OMB scoring costs on mortgage guarantees to developers against base closure, downsizing, or extended deployments?

    Answer. Funds are held to cover costs of executing privatization projects. Depending on the terms of the proposed project, budget scoring for loan guarantees could be one of those costs.


Next Hearing Segment(2)